Taller de Matematicas

February 19, 2018 | Author: Kaiser Rock | Category: Square Root, Set (Mathematics), Exponentiation, Integer, Rational Number
Share Embed Donate


Short Description

Descripción: libro para poder dar un repaso general de las matematicas...

Description

´ TALLER DE MATEMATICAS

´ TALLER DE MATEMATICAS

Autores: J.V. Becerril E., D. Elizarraraz M., R. Herrera A., R. P´ erez F., L.F. Res´ endis O., M. Salazar A., C.A. Ul´ın J., C. Zubieta B. Coordinador: L. F. Res´ endis O.

Departamento de Ciencias B´ asicas Universidad Aut´ onoma Metropolitana Unidad Azcapotzalco

Rector General Enrique Fernández Fassnacht Secretaria General Iris Santacruz Fabila Coordinador General de Difusión Carlos Ortega Guerrero Director de Publicaciones y Promoción Editorial Bernardo Ruiz Subdirectora de Publicaciones Laura González Durán Subdirector de Distribución y promoción editorial Marco Moctezuma

UNIDAD AZCAPOTZALCO Rectora Gabriela Paloma Ibáñez Villalobos Secretario Dario Guaycochea Guglielmi Director de la División de Ciencias Básicas e Inganiería Luis Enrique Noreña Franco Secretaria Académica Ma. de Lourdes Delgado Nuñez Jefe del Departamento de Ciencias Básicas David Elizarraraz Martínez Presidente del Consejo Editorial de la División de CBI Alejandro León Galicia Presidente del Comité Editorial de la División de CBI Lucio Vazquez Briseño Jefa de la Oficina de Producción Editorial y Difusión de Eventos de la División de CBI Rosa Ma. Benítez Mendoza

Taller de Matemáticas Primera edición 2013 Distribución nacional Diseño Gráfico: Juan Manuel Galindo Medina D.R. © 2013, Universidad Autónoma Metropolitana Prolongación Canal de Miramontes 3855, Ex Hacienda San Juan de Dios, Delegación Tlalpan 14387 México, D.F. D.R. © 2013 J.V. Becerril E., D. Elizarraraz M., R. Herrera A., R. Pérez F., L.F. Reséndis O., M. Salazar A., C.A. Ulín J., C. Zubieta B. Unidad Azcapotzalco / División de Ciencias Básicas e Ingeniería / Departamento de Ciencias Básicas Tel. (55) 5318 9011 - 9012, Fax (55) 5394 7385 ISBN de la colección: 978-607-477-934-9 Esta publicación no puede ser reproducida, ni en todo ni en parte, ni registrada o transmitida, por un sistema de recuperación de información, en ninguna forma y por ningún medio, sea mecánico, fotoquímico, electrónico, magnético, electroóptico, por fotocopia o cualquier otro, sin el permiso previo y por escrito, de los editores.

Impreso en México / Printed in Mexico Este material fue dictaminado y aprobado para su publicación por el Comité Editorial de la División de Ciencias Básicas e Ingeniería de la Universidad Autónoma Metropolitana Unidad Azcapotzalco en su sesión del día 28 de febrero del año 2013.

´ PROLOGO

Este libro presenta los temas del curso Taller de Matem´aticas, que forma parte del programa de nivelaci´on acad´emica de la Divisi´on de Ciencias B´asicas e Ingenier´ıa de la Universidad Aut´onoma Metropolitana, unidad Azcapotzalco. Se ha escrito con la intenci´on de ser un texto para el curso Taller de Matem´aticas, asimismo una gu´ıa para el profesor que imparte el curso y para el alumno que lo estudia; principalmente en cuanto a las habilidades operativas, que se desea alcance el estudiante al finalizar el curso. Se presentan de manera accesible, los conceptos necesarios para abordar los ejemplos y ejercicios del libro. En general, el contenido es material que el alumno ya conoce o debiese conocer de sus cursos de secundaria y bachillerato, sin embargo se han adicionado argumentos que ilustran algunos conceptos o que justifican algunos resultados, lo cual favorece que estudiantes que no han cubierto determinados temas en su anterior formaci´on, lo puedan hacer a partir del propio texto. Los ejemplos est´an resueltos con detalle y se espera que el alumno los lea y revise con atenci´on, procurando siempre entender cada uno de los pasos que se han redactado en la soluci´on de los mismos. Un prop´osito m´as es ofrecer al estudiante la oportunidad de resolver suficientes ejercicios que le ayuden a fortalecer lo bien aprendido y a corregir errores debidos a deficiencias en su formaci´on. En vista de lo anterior se han inclu´ıdo las soluciones al final de cada grupo de ejercicios. La abundante cantidad de ´estos tiene por finalidad que el profesor pueda trabajar en clase con equipos de estudiantes y as´ı asignar ejercicios distintos a cada uno de ´estos. La conducci´on del curso de Taller, presupone s´olo una exposici´on sucinta del material por parte del profesor, por ello es deseable que el estudiante lea el material te´orico y los ejemplos resueltos con anticipaci´on para que, en clase, la explicaci´on del profesor se reduzca a aclarar dudas y pasar directamente a la resoluci´on de ejercicios.

En algunas ocasiones, entender bien los ejemplos puede ser suficiente para resolver los ejercicios y el alumno puede omitir la lectura de la teor´ıa, que en general ya ha visto en alguna etapa de su formaci´on escolar. Sin embargo se recomienda leerla, pues as´ı podr´a entender un poco m´as el sustento te´orico de lo que ha aprendido de forma operativa. Aunque el texto ha sido escrito exprofeso para el Taller de Matem´aticas, puede ser usado por estudiantes y profesores de los cursos correspondientes de secundaria y bachillerato. Un lector dedicado se podr´a dar cuenta que es un texto adecuado para un estudio autodidacta. La escritura de este texto ha sido un esfuerzo colegiado de los autores, donde las diversas filosof´ıas personales de ense˜ nanza de las matem´aticas, redacci´on y escritura de textos matem´aticos, conocimiento matem´atico, concepciones sobre los estudiantes y muchos elementos m´as, fueron vertidas en los manuscritos originales y puestas a discusi´on en m´ ultiples reuniones de trabajo. Sabemos que cualquier trabajo es perfeccionable y tambi´en es el caso de este texto. Agradecemos a las autoridades de la Divisi´on de Ciencias B´asicas e Ingenier´ıa de la Universidad Aut´onoma Metropolitana Unidad Azcapotzalco, su inter´es y apoyo para la escritura y edici´on de este libro.

´Indice general 1. Aritm´ etica 1.1. Los n´ umeros reales . . . . . . . . . 1.1.1. Propiedades de los n´ umeros 1.2. Divisibilidad . . . . . . . . . . . . 1.3. Producto de n´ umeros racionales . . 1.4. Fracciones equivalentes . . . . . . 1.5. Fracciones irreducibles . . . . . . . 1.6. Divisi´on de n´ umeros racionales . . 1.7. Suma de n´ umeros racionales . . . .

. . . . reales . . . . . . . . . . . . . . . . . . . . . . . .

. . . . . . . .

. . . . . . . .

. . . . . . . .

. . . . . . . .

. . . . . . . .

. . . . . . . .

. . . . . . . .

. . . . . . . .

´ 2. Algebra 2.1. Notaci´on algebraica . . . . . . . . . . . . . . . . . . . . . 2.2. Exponentes . . . . . . . . . . . . . . . . . . . . . . . . . 2.2.1. Exponentes Enteros . . . . . . . . . . . . . . . . . 2.2.2. Exponentes fraccionarios y radicales . . . . . . . . 2.3. Operaciones algebraicas . . . . . . . . . . . . . . . . . . 2.3.1. Suma y resta . . . . . . . . . . . . . . . . . . . . 2.3.2. Multiplicaci´on . . . . . . . . . . . . . . . . . . . . 2.3.3. Divisi´on . . . . . . . . . . . . . . . . . . . . . . . 2.4. Productos notables . . . . . . . . . . . . . . . . . . . . . 2.4.1. Producto de binomios conjugados . . . . . . . . . 2.4.2. Producto de dos binomios con un t´ermino com´ un 2.4.3. Cuadrado de un binomio . . . . . . . . . . . . . . 2.4.4. Cubo de un binomio . . . . . . . . . . . . . . . . 2.4.5. Tri´angulo de Pascal . . . . . . . . . . . . . . . . . 2.4.6. F´ormula del binomio . . . . . . . . . . . . . . . . 2.5. Factorizaci´on . . . . . . . . . . . . . . . . . . . . . . . . 2.5.1. Factor com´ un . . . . . . . . . . . . . . . . . . . . 11

. . . . . . . .

. . . . . . . . . . . . . . . . .

. . . . . . . .

. . . . . . . . . . . . . . . . .

. . . . . . . .

15 15 16 17 21 22 23 25 28

. . . . . . . . . . . . . . . . .

37 37 39 39 46 59 59 61 67 82 82 85 88 92 95 100 103 103

´ Indice general

12

2.6.

2.7. 2.8.

2.9.

2.5.2. Factorizaci´on por agrupaci´on . . . . . . . . . . . . . . 2.5.3. Diferencia de cuadrados . . . . . . . . . . . . . . . . 2.5.4. Trinomio cuadrado perfecto . . . . . . . . . . . . . . 2.5.5. Trinomio cuadr´atico . . . . . . . . . . . . . . . . . . 2.5.6. Suma y diferencia de cubos . . . . . . . . . . . . . . 2.5.7. Miscel´anea de ejercicios . . . . . . . . . . . . . . . . . Operaciones con fracciones algebraicas . . . . . . . . . . . . 2.6.1. Simplificaci´on de fracciones algebraicas . . . . . . . . 2.6.2. Multiplicaci´on y divisi´on de fracciones algebraicas . . 2.6.3. Suma de fracciones algebraicas . . . . . . . . . . . . . 2.6.4. Otras fracciones algebraicas. . . . . . . . . . . . . . . Racionalizaci´on . . . . . . . . . . . . . . . . . . . . . . . . . Ecuaciones de primer grado . . . . . . . . . . . . . . . . . . 2.8.1. Ecuaciones de primer grado con una inc´ognita . . . . 2.8.2. Sistemas de dos ecuaciones de primer grado con dos inc´ognitas . . . . . . . . . . . . . . . . . . . . . . . . 2.8.3. M´etodo de sustituci´on . . . . . . . . . . . . . . . . . 2.8.4. M´etodo de suma o resta . . . . . . . . . . . . . . . . 2.8.5. Aplicaciones de las ecuaciones de primer grado . . . . Ecuaciones de segundo grado . . . . . . . . . . . . . . . . . . 2.9.1. El m´etodo de factorizaci´on . . . . . . . . . . . . . . . 2.9.2. M´etodo de completaci´on de cuadrados . . . . . . . . 2.9.3. La f´ormula general . . . . . . . . . . . . . . . . . . .

3. Geometr´ıa ´ 3.1. Angulos y rectas paralelas . . . 3.2. Pol´ıgonos . . . . . . . . . . . . . 3.2.1. Tri´angulos . . . . . . . . . 3.2.2. Cuadril´ateros . . . . . . . 3.3. El c´ırculo . . . . . . . . . . . . . 3.3.1. El per´ımetro de un c´ırculo 3.3.2. El ´area de un c´ırculo . . . 3.4. S´olidos . . . . . . . . . . . . . . 3.4.1. Cilindros . . . . . . . . . 3.4.2. Conos . . . . . . . . . . . 3.4.3. Esferas . . . . . . . . . . .

. . . . . . . . . . .

. . . . . . . . . . .

. . . . . . . . . . .

. . . . . . . . . . .

. . . . . . . . . . .

. . . . . . . . . . .

. . . . . . . . . . .

. . . . . . . . . . .

. . . . . . . . . . .

. . . . . . . . . . .

. . . . . . . . . . .

. . . . . . . . . . .

. . . . . . . . . . .

. . . . . . . . . . .

. . . . . . . . . . .

. . . . . . . . . . . . . .

106 109 110 113 125 128 133 133 135 140 146 155 159 159

. . . . . . . .

168 171 175 179 189 189 193 197

203 . 203 . 210 . 210 . 218 . 222 . 222 . 226 . 228 . 228 . 232 . 234

´ Indice general

13

4. Geometr´ıa anal´ıtica 4.1. El plano cartesiano . . . . . . . . . . . . . . . . . . . 4.2. La l´ınea recta . . . . . . . . . . . . . . . . . . . . . . 4.2.1. Inclinaci´on y pendiente de una recta . . . . . 4.2.2. Ecuaci´on de la recta que pasa por dos puntos 4.2.3. Ecuaci´on de la recta punto-pendiente . . . . . 4.2.4. Ecuaci´on de la recta de pendiente-ordenada . 4.2.5. Rectas horizontales . . . . . . . . . . . . . . . 4.2.6. Rectas verticales . . . . . . . . . . . . . . . . 4.3. Ecuaci´on general de la recta . . . . . . . . . . . . . . 4.4. Intersecci´on de rectas . . . . . . . . . . . . . . . . . 4.5. Rectas paralelas . . . . . . . . . . . . . . . . . . . . 4.6. Rectas perpendiculares . . . . . . . . . . . . . . . . 4.7. La circunferencia . . . . . . . . . . . . . . . . . . . . 4.7.1. Ecuaci´on general de la circunferencia . . . . . 4.8. La par´abola vertical . . . . . . . . . . . . . . . . . .

. . . . . . . . . . . . . . .

. . . . . . . . . . . . . . .

. . . . . . . . . . . . . . .

. . . . . . . . . . . . . . .

247 . 247 . 250 . 250 . 253 . 254 . 256 . 257 . 258 . 259 . 261 . 264 . 267 . 270 . 271 . 278

5. Trigonometr´ıa 5.1. Las funciones trigonom´etricas 5.2. El c´ırculo trigonom´etrico . . 5.3. Los ´angulos escuadra . . . . . 5.4. Identidades trigonom´etricas . 5.5. Aplicaciones . . . . . . . . .

. . . . .

. . . . .

. . . . .

. . . . .

. . . . .

. . . . .

. . . . .

. . . . .

. . . . .

. . . . .

. . . . .

. . . . .

. . . . .

. . . . .

. . . . .

. . . . .

. . . . .

. . . . .

285 285 293 302 305 311

Cap´ıtulo 1 Aritm´ etica En este cap´ıtulo se revisan conceptos, propiedades y operaciones entre n´ umeros racionales.

1.1.

Los n´ umeros reales

El conjunto de n´ umeros reales se denota por R. No se da una definici´on rigurosa de este conjunto de n´ umeros, s´olo se recuerdan algunos subconjuntos destacados de n´ umeros reales. El primero de ellos es el conjunto de los n´ umeros naturales, denotado por N, que consta de los n´ umeros que se usan para contar y es N = { 1, 2, 3, . . . } donde los puntos suspensivos indican que la lista continua dando lugar a un conjunto infinito. La necesidad de resolver ecuaciones de la forma x + 1 = 0, o m´as generalmente x + a = 0, para a ∈ N, propici´o la introducci´on de los n´ umeros enteros, denotados por Z, m´as precisamente Z = { . . . , −3, −2, −1, 0, 1, 2, 3, . . . } . a Una fracci´ on es un n´ umero de la forma , donde a es un n´ umero entero b llamado el numerador de la fracci´on y b es otro n´ umero entero, con b ̸= 0 y llamado denominador de la fracci´on. El conjunto de todas las fracciones es el conjunto de los n´ umeros racionales; se denota como a Q = { q = , a, b ∈ Z, b ̸= 0 } . b 15

16

Cap´ıtulo 1.

Aritm´etica

Se observa que N⊂Z⊂Q⊂R. Existen n´ umeros reales que no pueden expresarse en forma de fracci´on, a tales n´ umeros se les llama irracionales y el conjunto de estos n´ umeros se √ denota por I. Los n´ umeros 2 y π son ejemplos de n´ umeros irracionales. Finalmente se tiene el conjunto de los n´ umeros reales como la uni´on de estos dos conjuntos ajenos, es decir: R=Q∪I , con Q ∩ I = ∅.

1.1.1.

Propiedades de los n´ umeros reales

Los n´ umeros reales junto con las operaciones de suma y producto, satisfacen ciertas propiedades que se aplican cuando se opera con ellos, las cuales son conocidas. A continuaci´on se enuncian de manera expl´ıcita algunas de estas propiedades. Para cada a, b ∈ R: Cerradura. Se tiene a + b y a · b ∈ R. Es decir, la suma y el producto de dos n´ umeros reales es nuevamente un n´ umero real. Conmutativa. Se tiene a + b = b + a y a · b = b · a. Esto es, el orden de los t´erminos en la suma y el de los factores en el producto, no altera el resultado. Asociativa. Se tiene (a + b) + c = a + (b + c) y (a · b) · c = a · (b · c). Es decir, el orden de asociaci´on al hacer una suma o un producto da el mismo resultado. Distributiva. Se tiene a · (b + c) = a · b + a · c y (b + c) · a = b · a + c · a. Es decir, el producto se distribuye con respecto a la suma. Existencia de elementos neutros. Existen dos elementos distintos, 0 ∈ R y 1 ∈ R tal que a + 0 = 0 + a = a, a · 1 = 1 · a = a. El n´ umero 0 se llama neutro aditivo y el n´ umero 1 neutro multiplicativo. Existencia de inversos. Para cada a ∈ R existe el elemento −a ∈ R, el inverso aditivo de a, tal que a + (−a) = 0. Si a ̸= 0, existe a−1 ∈ R, el inverso multiplicativo de a, tal que a · a−1 = a−1 · a = 1.

1.2.

Divisibilidad

17

Estas propiedades se ense˜ nan y aprenden de manera intuitiva desde los primeros cursos de matem´aticas. De igual forma en este texto se aplicar´an estas propiedades.

1.2.

Divisibilidad

Sean a, b dos n´ umeros enteros. Se dice que b divide al n´ umero a si existe c ∈ Z tal que a = bc. Se observa entonces que a es divisible por b y por c. Se tiene los siguientes criterios de divisibilidad. Un n´ umero es divisible por 2 si termina en 0, 2, 4, 6 u ´ 8. Un n´ umero es divisible por 3 si al sumar sus cifras se obtiene un n´ umero m´ ultiplo de 3. umero es divisible por 4 si el n´ umero formado por sus dos u ´ltimas Un n´ cifras es m´ ultiplo de 4. umero es divisible por 5 si termina en 0 ´o bien en 5. Un n´ umero es divisible por 10 si termina en cero. Un n´ Para saber si un n´ umero es divisible por n´ umeros diferentes a los mencionados anteriormente es necesario realizar la divisi´on. Ejemplo

1.2.1 El n´ umero 875160 es divisible por 2 porque termina en 0 que es un n´ umero par. Es divisible por 3 porque la suma de sus cifras 8 + 7 + 5 + 1 + 6 + 0 = 27 = 3 · 9 es un m´ ultiplo de 3. Es divisible por 4 porque 60 = 4 × 15, que es m´ ultiplo de 4. Es divisible por 5 y por 10, pues termina en 0. Como las divisiones 875160 875160 875160 = 79560 , = 67320 , = 51480 11 13 17 son exactas tambi´en es divisible por 11, 13 y 17.  Ejercicio 1.2.1 Dados los siguientes n´ umeros determine cu´ales son divisibles por 2, 3, 4, 5, 7, 11 y 13. 1. 1000

2. 4800

7. 49140

8. 5824.

3. 28800

4. 2400 5. 660

6. 27720

18

Cap´ıtulo 1.

Aritm´etica

Soluciones. 1. 2, 4 y 5

2. 2, 3, 4 y 5

3. 2, 3, 4 y 5

4. 2, 3, 4 y 5

5. 2, 3, 4, 5 y 11

6. 2, 3, 4, 5, 7 y 11

7. 2, 3, 4, 5, 7 y 13

8. 2, 4, 7 y 13.

Un n´ umero natural, diferente de 1, se llama primo si s´olo es divisible por si mismo y la unidad. Los primeros n´ umeros primos son 2, 3, 5, 7, 11, 13, 17, 19, . . . y hay una infinidad de ellos. Se recuerda que (n factores)

z }| { a = a · a · a···a , n

es decir, an abrevia el producto de a realizado n veces. Teorema 1.2.1 Todo n´ umero natural a se puede escribir de manera u ´nica como el producto de sus diferentes factores primos, contando multiplicidades. As´ı a = pα1 1 pα2 2 · · · pαnn con p1 , . . . , pn n´ umeros primos distintos, y α1 , . . . , αn ∈ N. Ejemplo

1.2.2 Escribir como producto de sus factores primos al n´ umero

257400. Soluci´on. Para ello se considera el siguiente arreglo y se usan los criterios de divisibilidad 257400 2 se toma mitad 128700 2 se toma mitad 64350 2 se toma mitad 32175 3 se toma tercera 10725 3 se toma tercera 3575 5 se toma quinta 715 5 se toma quinta 143 11 se toma onceava 13 13 se toma treceava 1

1.2.

Divisibilidad

19

Los factores primos aparecen en la segunda columna y sus repeticiones indican su multiplicidad. As´ı, se tiene 257400 = 23 · 32 · 52 · 11 · 13 y esta es la descomposici´on que afirma el teorema anterior.  umeros, escriba su descomposici´on Ejercicio 1.2.2 Dados los siguiente n´ en n´ umeros primos. 1. 42

2. 5400

7. 28875

8. 139425.

3. 1848

4. 31500 5. 15210

6. 13310

Soluciones. 1. 2 · 3 · 7

2. 23 · 33 · 52

5. 2 · 32 · 5 · 132 6. 2 · 5 · 113

3. 23 · 3 · 7 · 11

4. 22 · 32 · 53 · 7

7. 3 · 53 · 7 · 11

8. 3 · 52 · 11 · 132 .

El m´ınimo com´ un m´ ultiplo (m.c.m) de dos n´ umeros a, b ∈ N es el n´ umero m´as peque˜ no que es m´ ultiplo tanto de a como de b. Dos n´ umeros naturales a y b se dicen primos relativos si su m.c.m. es ab; equivalentemente los n´ umeros primos que aparecen en la descomposici´on prima de a son todos diferentes a los que aparecen en la descomposici´on prima de b. Ejemplo

1.2.3 Determinar el m´ınimo com´ un m´ ultiplo de 36 y 15.

Soluci´ on. Los m´ ultiplos de 15 son {15, 30, 45, 60, 75, 90, 105, 120, 135, 150, 165, 180, 195, . . .} y los de 36 son {36, 72, 108, 144, 180, 216, . . .} . De las listas anteriores se observa que el primer m´ ultiplo com´ un, y m´as peque˜ no, de 36 y 15 es 180.  Una manera sistem´atica de obtener el m´ınimo com´ un m´ ultiplo es recurrir a su descomposici´on en factores primos. El siguiente ejemplo lo ilustra. Ejemplo

1.2.4 Obtener el m.c.m. de 194040 y 136500. Se obtiene primero la descomposici´on de los n´ umeros en sus factores primos:

20

Cap´ıtulo 1.

Aritm´etica

194040 136500 2 97020 68250 2 48510 34125 2 24255 34125 3 8085 11375 3 2695 11375 5 539 2275 5 539 455 5 539 91 7 77 13 7 11 13 11 1 13 13 1 1 As´ı 194040 = 23 · 32 · 5 · 72 · 11 y 136500 = 22 · 3 · 53 · 7 · 13. El m´ınimo com´ un m´ ultiplo es el producto de todos los primos que aparecen en las descomposiciones, elevado cada uno de ellos a la m´axima potencia con que aparece. As´ı el m.c.m. buscado es: 23 · 32 · 53 · 72 · 11 · 13 = 63063000 . 

Ejercicio 1.2.3 Dados los siguientes n´ umeros escribir su m´ınimo com´ un m´ ultiplo como producto de n´ umeros primos. 1. 300, 120

2. 300, 24

3. 600, 3000

4. 6300, 2940

5. 25200, 73500

6. 29400, 18900,

7. 76230, 33000

8. 105105, 13013

9. 600, 90, 756

10. 66150, 156, 1260

11. 25200, 1500, 3780

12. 31500, 3900, 7560.

1.3. Producto de n´ umeros racionales

21

Soluciones. 1. 600 = 23 · 3 · 52

2. 600 = 23 · 3 · 52

3. 3000 = 23 · 3 · 53

4. 44100 = 22 · 32 · 52 · 72

5. 882000 = 24 · 32 · 53 · 72

6. 264600 = 23 · 33 · 52 · 72

7. 7623000 = 23 · 32 · 53 · 7 · 112

8. 1366365 = 3 · 5 · 72 · 11 · 132

9. 37800 = 23 · 33 · 52 · 7

10. 1719900 = 22 · 33 · 52 · 72 · 13

11. 378000 = 24 · 33 · 53 · 7

1.3.

12. 2457000 = 23 · 33 · 53 · 7 · 13

Producto de n´ umeros racionales a c y se denota por b d (a) ( c ) a c ´o × b d b d

El producto de los n´ umeros racionales a c · , b d y est´a definido como

(a) ( c ) b

d

=

ac . bd

Se observa que el numerador del producto anterior de fracciones se obtiene multiplicando los numeradores a y c y el denominador del producto de fracciones se obtiene multiplicando los respectivos denominadores b y d. Ejemplo

1.3.1 Calcular los siguientes productos:

( )( ) 3 1 1. 4 5 Soluciones.

( 2.

−2 7

)( )( ) 3 6 5 7

22

Cap´ıtulo 1.

( )( ) 3 1 3 (3)(1) 1. = . = 4 5 (4)(5) 20 ( )( )( ) −2 3 6 −36 36 (−2)(3)(6) 2. = =− . = 7 5 7 (7)(5)(7) 245 245

Aritm´etica



umero entero a es tambi´en un n´ umero racional, debido a que Ya que todo n´ a a = , las multiplicaciones de fracciones por n´ umeros enteros se efectuan 1 como se indica en el siguiente ejemplo. ( ) Ejemplo 2 1.3.2 Calcular (3) . 5 ( ) ( )( ) 2 2 3 2·3 6 (3) = = = . 5 5 1 5·1 5

Soluci´ on.



Fracciones equivalentes

1.4.

Se dice que dos fracciones son equivalentes cuando representan el mismo 1 a 2 c n´ umero racional, por ejemplo y . En general dos fracciones y son 2 4 b d equivalentes si y s´olo si ad = bc. Ejemplo 1.

8 4 y 6 3

1.4.1 Verificar que son fracciones equivalentes: 2.

9 45 y 14 70

Soluciones. 1.

8 4 = 6 3

9 45 2. = 14 70

ya que

8×3 = 4×6 24 = 24 .

ya que

9 × 70 = 45 × 14 630 = 630 .



a a Si se quiere obtener fracciones equivalentes a , basta con multiplicar b b por el n´ umero 1 representado en una de sus fracciones equivalentes, ejemplo: 2 10 −3 1= = = = ··· . 2 10 −3

1.5.

Fracciones irreducibles

Ejemplo

23

1.4.2 Encontrar tres fracciones equivalentes a

5 . 4

Soluci´ on. Se eligen algunas fracciones equivalentes al n´ umero 1, as´ı ( )( ) 5 5 2 10 = . = 4 4 2 8

Luego

5 = 4

( )( ) 5 7 35 . = 4 7 28

5 = 4

( )( ) 5 −3 −15 15 = = . 4 −3 −12 12

5 10 35 15 umero racional. , , , representan el mismo n´ 4 8 28 12

1.5.



Fracciones irreducibles

a Dada la fracci´on con a, b ∈ Z, b ̸= 0, se dice irreducible si a y b son primos b relativos. Se observa que una fracci´on irreducible no se puede simplificar m´as. Ejemplo

1.5.1 Determinar la fracci´on irreducible que representa al n´ umero 30 racional . Mostrar que es equivalente a la original. 42 Soluci´ on. Recurriendo a la descomposici´on en factores primos ( )( )( ) ( ) 30 2·3·5 2 3 5 5 5 = = = (1)(1) = . 42 2·3·7 2 3 7 7 7 Se verifica que

30 5 es equivalente a : 42 7 30 5 = 42 7 (30)(7) = (5)(42) 210 = 210 .

24

Cap´ıtulo 1.

Aritm´etica

30 5 As´ı y representan el mismo n´ umero racional con la diferencia de que 42 7 30 5 pudo simplificarse a y ´este u ´ltimo no puede simplificarse m´as.  42 7 Ejemplo

1.5.2 Encontrar la forma irreducible de

24 . 81

Soluci´ on. Al obtener la descomposici´on en factores primos se tiene ( )( 3) 24 23 · 3 3 2 8 = 4 = = . 3 81 3 3 3 27



Otra manera de obtener la forma irreducible es aplicar los criterios de divisibilidad. Ejemplo

1940400 umero 1.5.3 Simplificar el n´ . Se observa que el denomi693000 nador y el numerador son divisibles por 100, ya que ambos terminan en doble cero. Luego 1940400 19404 9702 3234 1078 154 14 = = = = = = . 693000 6930 3465 1155 385 55 5 En la primera igualdad se dividi´o entre 100, en la segunda igualdad se sac´o mitad, en la tercera y cuarta igualdades se sac´o tercera, en la quinta igualdad se sac´o s´eptima y finalmente se tom´o onceava.  Ejercicio 1.5.1 Efectuar la operaci´on indicada y expresar el resultado en su forma irreducible. ( )( ) ( ) ( ) −2 4 3 11 1. 2. (7) 3. (−2) 5 5 4 10 ( )( ) 4 −3 4. (3) 5 4

( )( )( ) 7 0 −4 5. 4 5 5

( ) 1 7. (0) 2

( ) ( ) 5 −3 8. (3) 6 8

( )( )( ) 1 −3 −9 6. 6 4 5

Obtener tres fracciones equivalentes para las siguientes fracciones. 9. −

1 2

10.

5 4

11. −

7 3

1.6.

Divisi´ on de n´ umeros racionales

25

Soluciones. 1.

−8 25

5. 0

2.

21 4

3. −

6.

9 40

7. 0

11 5

−9 5

4.

8. −

15 16

Para los ejercicios 9, 10 y 11 no se presenta su soluci´on, sin embargo se recuerda que existen una infinidad de fracciones equivalentes para cada n´ umero racional. Ejercicio 1.5.2 Encontrar la forma irreducible de los siguientes n´ umeros racionales 1.

390 210

7.

300300 50050 8. . 13650 392700

2.

600 2200

3.

375 975

4.

630 7245

2 23

5.

3 5

5.

12936 169884 6. 21560 65340

Soluciones. 1.

13 7

1.6.

2.

3 11

3.

5 13

4.

6.

13 5

7.

11 3

8.

13 17

Divisi´ on de n´ umeros racionales a c y , con c ̸= 0, se denota como b d a b o bien c . d

La divisi´on de los n´ umeros racionales a c ÷ b d

En su primera representaci´on se tiene la definici´on

c ad a ÷ = . b d bc Si la divisi´on est´a expresada en la segunda forma, la aplicaci´on de la definici´on se conoce como la regla del emparedado:

26

Cap´ıtulo 1.

Aritm´etica

a b = ad . c bc d Ejemplo

1.6.1 Calcular las siguientes divisiones:

( ) ( ) −2 3 1. ÷ 4 5

5. (

3 10

2 − 5 )(

12 5

)

( ) 5 2. ÷ 4 6 ( )( ) 1 5 3 4 6. ( ) ( ) 6 2 − 7 5

2 3. 3 5 − 4 ( ) 4 − 8 7 7. 11

Soluciones. ( ) ( ) −2 3 (3)(5) 15 ÷ = =− . 1. 4 5 (4)(−2) 8 ( ) ( ) ( ) 5 5 4 (5)(1) 5 2. ÷ 4= ÷ = = . 6 6 1 (6)(4) 24 2 (2)(4) 8 3. 3 = =− . 5 (3)(−5) 15 − 4 7 7 7 4. 4 = 4 = . 8 8 32 1 5. (

3 10

2 − 5 )(

12 5

)

4.

7 4 8

1.6.

Divisi´ on de n´ umeros racionales

27

Se resuelve primero la multiplicaci´on indicada en el denominador ( )( ) 3 12 22 · 32 18 (3)(12) = = . = 2 10 5 (10)(5) 2·5 25 Ahora se realiza la divisi´on 2 5 = (−2)(25) = (−2)(5)(5) = − 5 . 18 (5)(18) (5)(9)(2) 9 25



( )( ) 1 5 5 (5)(35) 175 3 4 6. ( ) ( ) = 12 = =− . 12 6 2 −(12)(12) 144 − − 35 7 5 ( ) ( )( ) 8 4 4 32 − 8 − − 32 7 7 1 7. = = 7 =− . 11 11 11 77 1 1



Ejercicio 1.6.1 Efectuar la operaci´on indicada. Simplificar el resultado. 1. −

3 1 ÷ 4 2

9 ÷ 4 4 ( )( ) 1 2 5 6 ( ) 5. 4 9

2.

2 4. 5 3 − 4 ( )( ) 2 −3 9 5 ) 7. ( ) ( 4 −10 3 6 ( 10.

−3 8 ÷ 4 6

)( ) 4 5

8. (

2 )( ) 7 1 − 6 4

1 4 ( ) 2 (8) 7 6. 6 7 3. 7 ÷

( 9.



2 3

)



 1  4   2  6

28

Cap´ıtulo 1.

Aritm´etica

Soluciones. 1. − 6.

8 3

1.7.

3 2

2.

9 16

3. 28

7.

3 50

8. −

4. −

8 15

48 1 9. − 7 2

5.

3 20

10. −

9 20

Suma de n´ umeros racionales

La suma de fracciones presenta dos casos. El primero es cuando las fracciones tienen el mismo denominador, el segundo es cuando los denominadores son diferentes. Caso 1. Si los n´ umeros racionales tiene el mismo denominador la suma est´a dada como:

a b a+b + = . c c c Caso 2. Si los denominadores son diferentes, la suma se expresa por medio de fracciones equivalentes que tengan el mismo denominador, reduciendo la suma al caso 1.

a c a·d c·b ad + bc + = + = . b d b·d d·b bd Una manera alternativa de conseguir fracciones equivalentes es por medio del m.c.m. de los denominadores. Por ejemplo si m es el m.c.m. de b, d y f se tiene

m m m ·a+ ·c+ ·e a c e b d f + + = . b d f m Los siguientes ejemplos ilustran el proceso.

1.7.

Suma de n´ umeros racionales

Ejemplo

1.

3 5 + 4 4

29

1.7.1 Efectuar las siguientes operaciones. 4 2 − 3 3

2.

Soluciones. Al tener id´enticos denominadores la suma es directa. 1.

3 5 3+5 8 + = = =2. 4 4 4 4

2.

4 2 4−2 2 − = = . 3 3 3 3



Ejemplo

1.7.2 Efectuar las siguientes operaciones. ( ) 2 3 1 5 2 5 2 4. 3 1. + 2. − 3. +3 + 4 3 3 8 5 5 7

6.

3 5 − 4 6

7.

5.

2 5 − 3 6

5 7 − 16 4

Soluciones. 1. Primero se obtienen fracciones equivalentes de cada sumando con el mismo denominador 1 3 3 × = , 4 3 12

5 4 20 × = . 3 4 12

Por lo tanto

1 5 3 20 23 + = + = . 4 3 12 12 12 Alternativamente, dado que el m.c.m. de 3 y 4 es 12 se tiene 12 12 ·1+ ·5 1 5 3·1+4·5 23 3 + = 4 = = . 4 3 12 12 12 2. Se obtienen fracciones equivalentes para cada sumando 2 8 16 × = , 3 8 24

5 3 15 × = . 8 3 24

30

Cap´ıtulo 1.

Aritm´etica

Por lo tanto

2 5 16 15 1 − = − = . 3 8 24 24 24 Alternativamente, dado que el m.c.m. de 3 y 8 es 24 se tiene 24 24 · 2 − ·5 8·2−3·5 1 2 5 3 8 − == = = . 3 8 24 24 24 3. En este ejercicio se considera 3 =

3 . Entonces, 1

2 2 3 (2)(1) + (5)(3) 17 +3= + = = . 5 5 1 5 5 4. Para este ejercicio se sugiere calcular primero la operaci´on indicada entre par´entesis, es decir 2 3 14 + 15 29 + = = , 5 7 35 35 y luego

( )( ) 29 3 87 = . 1 35 35

O bien al aplicar la propiedad distributiva se tiene ( ( ) ( ) ) 2 3 2 3 3 + =3 +3 5 7 5 7 6 9 = + 5 7 42 + 45 = 35 87 = . 35 5. Primero se obtiene la fracci´on equivalente conveniente 2 2 4 × = , 3 2 6 2 5 4 5 1 − = − =− . 3 6 6 6 6

1.7.

Suma de n´ umeros racionales

6.

31

(3)(6) − (4)(5) 18 − 20 2 1 3 5 − = = =− =− . 4 6 24 24 24 12 Ya que el m.c.m de 4 y 6 es 12 se tiene 12 12 ·3− ·5 3·3−2·5 9 − 10 1 3 5 4 6 − = = = =− . 4 6 12 24 12 12

7. Una fracci´on equivalente al segundo sumando es 7 4 28 · = . 4 4 16 As´ı

5 7 5 28 −23 23 − = − = =− . 16 4 16 16 16 16  El siguiente ejemplo muestra nuevamente el algoritmo de la suma de n´ umeros racionales tomando como denominador com´ un el m.c.m. de los denominadores de las fracciones involucradas. Ejemplo

1.7.3 Realizar la suma siguiente 39 129 187 + − . 60 90 225

Soluci´ on. La descomposici´on en n´ umeros primos de cada una de los denominadores es 60 = 22 · 3 · 5

90 = 2 · 32 · 5 ,

225 = 32 · 52 .

El m.c.m. de los tres n´ umeros es 22 · 32 · 52 = 900. As´ı se tiene 39 129 187 + − = 60 90 225

900 900 900 · 39 + · 129 − · 187 60 90 225 900

=

15(39) + 10(129) − 4(187) 900

=

585 + 1290 − 748 1127 = . 900 900

32

Cap´ıtulo 1.

Aritm´etica

Si se recurre a fracciones equivalentes directamente se tiene



39 15 39 = · 60 60 15

=

585 900

129 129 10 = · 90 90 10

=

1290 900

187 4 748 187 = − · = − . 225 225 4 900 

Claramente, al sumar se obtiene el mismo resultado.

Ejercicio 1.7.1 Efectuar las siguientes operaciones. −3 +8 4 ( ) 2 1 5. 3 − 6 3

1 4 − 2 5 ( )( ) −4 1 3 4. + 7 2 4 1.

( 7. (−3)

2 4 1 − + 5 5 5

2.

) 8.

2 +7 5

3.

1 5

1 2 1 − + 2 5 10 ( )( ) 3 4 3 1 6. − + −1 7 7 7 8

3.

Soluciones. 1. − 8.

3 10

2.

29 4

4.

−5 7

5. 0

37 5

Ejercicio 1.7.2 Efectuar las siguientes operaciones.

6.

−15 3 7. 56 5

1.7.

Suma de n´ umeros racionales

( )( ) 4 2 5 3 ( ) 1. 2 (3) 5

( 2.

( )( ) 3 7 4 8 4. 5 −1 4 ( )( )( ) 8 11 5 7. −4 6 9 5 6 10. 5

(

( 4+

3 −1 ÷ 7 4

17.

4 −2 ÷ 5 3

)

( ÷

2 8. 5

23. −

2 ÷ 4 5

(

(

4 3 − 6 8

3 − 2

18. 12 ÷ ( 20. )(

1 − 5

4 2 ÷ 3 3

1 5 )( ) 2 −5 5

) +3

( 6. 2

6 2 − 8 4

9. −

5 8 ÷ 4 7

)

5 4 + 14. 3 6 2 1 − 4 3 ( ) 3 1 16. − ÷ 2 2 4 )

3.

( ) 1 2 ÷ − 3 5 7 2 − + 3 5

]

( )( ) 0 4 11. (4) 9 2

( )( ) 3 9 3 11 19. + − 5 4 4 4 3 1 2 21. + − 22. 5 4 3

)( )( ) 1 8 4 7 2 1 − 3 5

2 4 1 5. −3 + − 7 7 7

))

1 3 + 2 4

3 − 5

[

8 3 − 13. 5 4 1 2 + 5 5 ( ) ( ) 3 1 2 3 15. − ÷ − 5 4 5 4 (

33

12.

3 5 − 2 2 2

)

34

Cap´ıtulo 1.

Aritm´etica

Soluciones. 1.

4 9

7. −

2. − 176 27

8.

18 49

7 60

13.

17 12

14. 14

19.

3 20

20. −

127 25

3.

50 29

9. −

35 32

15. −1 21.

11 60

4.

21 8

5. −

15 7

10.

96 35

11. 0

12. −

16.

5 8

17. −

24 18. 60 25

22. 0

23. −

1 10

6.

1 2 1 2

En lo hecho anteriormente se han usado los par´ entesis ( ) para indicar el producto de dos n´ umeros. Pero los par´entesis no s´olo indican un producto, tambi´en se utilizan para agrupar e indican que lo que est´a contenido dentro de ellos se puede considerar como un solo ente. Otros s´ımbolos de agrupamiento son los corchetes [ ] y las llaves { }. Los siguientes ejemplos y ejercicios abundan sobre el uso de s´ımbolos de agrupamiento. Ejemplo

1.7.4 Efectuar las siguientes operaciones. ( ) [( ) ] ( ) 2 1 3 3 2 3 4 1 1. − + 2. + − ÷ − 3− +7 5 2 4 2 5 4 5 2 3. 500 − {[(6 − 1)8 ÷ 4]3 + [16 ÷ (10 − 2)]} − 5

Soluciones. 1. Primero se calcula la operaci´on dentro del par´entesis, as´ı: ( ) 2 2 2+3 2 5 8 − 25 17 1 3 − + = − = − = =− . 5 2 4 5 4 5 4 20 20 2. Se procede a eliminar los par´entesis desde el interior al exterior [( ) ] ( ) 3 2 3 4 1 + − ÷ − 3− +7= 2 5 4 5 2

1.7.

Suma de n´ umeros racionales

35

[

] ( ) 30 + 8 − 15 4 6 1 = ÷ − − +7 20 5 2 2 [ ] ( ) 23 4 5 = ÷ − +7 20 5 2 115 5 = − +7 80 2 23 5 7 = − + 16 2 1 23 − 40 + 112 = 16 95 = . 16 3. 500 − {[(6 − 1)8 ÷ 4]3 + [16 ÷ (10 − 2)]} − 5 = = 500 − {[(5)8 ÷ 4]3 + [16 ÷ 8]} − 5 = 500 − {[40 ÷ 4]3 + 2} − 5 = 500 − {[10]3 + 2} − 5 = 500 − {30 + 2} − 5 = 500 − 32 − 5 = 463 . Ejercicio 1.7.3 Efectuar las siguientes operaciones: ( ) ( ) 3 3 1. −3 ÷ 7− 5 4 [ ( )] 3 5 2. 5 − (−24 + 3) − − 5 5 {[( ] [( ) ) ( )] } 3 5 7 2 4 3. × ÷3 9− ÷ 2 +3 5 3 2 7 49 4. (40 ÷ 5)5 + (6 ÷ 2)3 + 4 − [(5 × 2) ÷ 10] 5. 800 − {20 − 3 × 4 + 5[18 − (6 − 1)3 + (5 − 2)4]} [( ) ] ( ) 3 7 3 3 8 7 6. × ÷ + × 2 4 2 8 3 3



36

Cap´ıtulo 1.

Aritm´etica

[ ( )] [( ) ] 1 3 2 1 3 3 7. − + − × − −2 2 4 9 2 4 5 Soluciones. 1. −

48 125

5. 717

2. 17 6.

49 12

3. 4 7.

473 360

4. 52

Cap´ıtulo 2 ´ Algebra 2.1.

Notaci´ on algebraica

Se inicia esta secci´on recordando la notaci´on algebraica. Expresiones de la forma: 1 √ 9x − 3x2 − 4x + 10 , 6xy 3 − xy − 5y 2 + 1 + y , 7y se denominan expresiones algebraicas, es decir, son expresiones constitu´ıdas por n´ umeros, literales y signos de operaciones. Las expresiones algebraicas se construyen a partir de sus t´ erminos, los cuales aparecen separados √ por los signos (+) ´o (−). Por ejemplo, la expresi´on 6xy 3 − xy − 5y 2 + 1 1 √ tiene cuatro t´erminos, 6xy 3 , xy, 5y 2 y 1; mientras que la expresi´on 9x− 7y 1 tiene dos t´erminos, 9x y . 7y Un t´ermino consta de un coeficiente y una parte literal . Por ejemplo 1 en el t´ermino 3x2 , 3 es el coeficiente y x2 es la parte literal. En el t´ermino , 7y 1 1 el coeficiente es y la parte literal es . El t´ermino 10 no tiene parte literal 7 y y se llama t´ ermino constante. La parte literal de un t´ermino puede estar elevada a un exponente, por ejemplo, en 5a3 , 5 es el coeficiente y 3 es el exponente de a. Una expresi´on algebraica que contiene un s´olo t´ermino se llama monomio, si tiene dos se llama binomio, la de tres un trinomio, en general, la de dos o m´as t´erminos se llama multinomio. 37

38

Cap´ıtulo 2.

´ Algebra

Una expresi´on algebraica donde las literales est´an u ´nicamente afectadas por exponentes enteros positivos recibe el nombre de polinomio. Por ejemplo 3x2 − 4x + 10 y 2x5 y − 4x3 − xy 2 son polinomios; no as´ı 6xy 3 −



xy − 5y 2 + 1 y 9x −

1 . 7y

Se llaman t´ erminos semejantes aquellos que s´olo difieren en su coeficiente num´erico. Por ejemplo 9x3 y 4 ,

−13x3 y 4 ,

2 3 4 xy , 3



5x3 y 4

son todos t´erminos semejantes. El grado de un monomio es la suma de todos los exponentes de las literales que lo conforman. As´ı el grado de 3x2 es 2, el de 2x5 y es 6 y el de −4x es 1. El grado de un polinomio es el correspondiente al t´ermino de mayor grado cuyo coeficiente sea distinto de cero. Las constantes se consideran polinomios de grado cero. Ejemplo

2.1.1 Determinar el grado de cada uno de los polinomios dados.

1. 6x2 y 4 +3xy 2 −11xy

2. −2xy 3 +5x2 y 2 −8x3 y+x4

Soluci´on 1. Los grados de los t´erminos o monomios 6x2 y 4 , 3xy 2 y 11xy son 6, 3 y 2 respectivamente. Por consiguiente el grado del polinomio es 6. 2. En el polinomio −2xy 3 + 5x2 y 2 − 8x3 y + x4 , todos los t´erminos tienen grado 4, de modo que el grado del polinomio es tambi´en 4. Nuevamente, se usar´an los s´ımbolos de agrupamiento ya mencionados: los par´entesis ( ), los corchetes [ ] y las llaves { }, para indicar que el grupo de t´erminos contenidos dentro de ellos debe ser considerado como un s´olo ente; estos s´ımbolos tambi´en indican una multiplicaci´on.

2.2. Exponentes

2.2.

39

Exponentes

El uso de exponentes es frecuente en el manejo de expresiones algebraicas. En esta secci´on se enuncian las leyes de los exponentes y se presentan diversos ejemplos. Inicialmente se presentan los exponentes enteros y posteriormente los exponentes fraccionarios o radicales.

2.2.1.

Exponentes Enteros

El ´area de un c´ırculo de radio r est´a dada por A = πr2 , de modo que el ´area de un c´ırculo de radio 5 es A = π52 = 25π . Por otro lado el volumen de un cubo, de arista l, est´a dado por V = l3 . As´ı que el volumen de un cubo cuya arista es igual a 4 es V = 43 = 64. En estos dos casos se ha hecho uso de los exponentes de la siguiente manera 52 = 5 · 5 = 25 y 43 = 4 · 4 · 4 . Sean a cualquier n´ umero real y n un entero positivo. Se define la n-´ esima potencia del n´ umero a como (n factores)

z }| { a = a · a · a···a . n

(2.2.1)

El n´ umero a se llama base y el n´ umero n exponente. Se extiende la definici´on previa de la siguiente forma a0 = 1 , Ejemplo

5

1. 3

7.

1 − 2

1 an

con a ̸= 0 .

2.2.1 Calcular las siguientes potencias:

4

(

a−n =

2. 2 )−4

3. 6

0

( )4 1 4. 2

5.

5−3

6. (−2)−5

40

Cap´ıtulo 2.

´ Algebra

Soluci´ on. 1. 34 = 3 · 3 · 3 · 3 = 81 .

2. 25 = 2 · 2 · 2 · 2 · 2 = 32 . ( )4 1 1 1 1 1 1 4. = · · · = . 2 2 2 2 2 16

0

3. 6 = 1 . 5. 5−3 =

1 1 = . 3 5 125

6. (−2)−5 = ( 7.

1 − 2

1 1 1 = = − . (−2)5 −32 32

)−4 =(

1

1 )4 = 1 = 16 . 1 − 16 2



A partir de la definici´on de exponente, se pueden establecer una serie de propiedades, denominadas leyes de los exponentes, las cuales se enuncian a continuaci´on. Sean a y b n´ umeros reales y m y n n´ umeros enteros. I. El producto de dos potencias con la misma base es

am an = am+n . II. El cociente de dos potencias con base igual es

am = am−n n a

(a ̸= 0) .

III. La potencia de otra potencia es

(am )n = amn

(a ̸= 0) .

2.2. Exponentes

41

IV. La potencia de un producto es

con ab ̸= 0 si m < 0 .

(ab)m = am bm V. La potencia de un cociente es

( a )m b

am = m b

con b ̸= 0, y a ̸= 0 si m ≤ 0 .

Ejemplo

2.2.2 Simplificar las expresiones siguientes, escribiendo el resultado final sin exponentes negativos. −6

1. 6 · 6

2. z · z

85 5. −3 8

x3 · x−5 6. x−7

3

4

9. 85 · (8−3 )2

2

10.

3. x

−2

·x ·x 11

−9

7. y −3 ÷ (y −10 · y 6 )

(x−3 )−3 (x4 )−2

2−4 4. 2 8. (5−3 )−7

11. (x2 )2 ÷ (x−5 )−5

Soluciones. En los siguientes ejercicios se aplica la primera ley de los exponentes. 1. 63 · 64 = 63+4 = 67 . 2. z 2 · z −6 = z 2+(−6) = z −4 =

1 . z4

3. x−2 · x11 · x−9 = x−2+11 · x−9 = x9 · x−9 = x0 = 1 . En los siguientes ejercicios se aplican la primera y segunda ley de los exponentes. 4.

2−4 1 = 2−4−1 = 2−5 = 5 . 2 2

5.

85 = 85−(−3) = 88 . 8−3

42

Cap´ıtulo 2.

6.

´ Algebra

x3−5 x−2 x3 · x−5 = = = x−2−(−7) = x5 . x−7 x−7 x−7 y −3 = y −3+4 = y . y −4 En los siguientes ejercicios se aplican la primera, segunda y tercera ley de los exponentes.

7. y −3 ÷ (y −10 · y 6 ) = y −3 ÷ (y −10+6 ) = y −3 ÷ (y −4 ) =

8. (5−3 )−7 = 5(−3)(−7) = 521 . 9. 85 · (8−3 )2 = 85 · 8−6 = 85−6 = 8−1 =

1 . 8

(x−3 )−3 x(−3)(−3) x9 10. = (4)(−2) = −8 = x9−(−8) = x17 . 4 −2 (x ) x x 11. (x2 )2 ÷ (x−5 )−5 =

x2(2) x(−5)(−5)

=

x4 1 = x4−25 = x−21 = 21 . 25 x x

Ejemplo



2.2.3 Simplificar las expresiones siguientes, eliminando par´entesis y exponentes negativos. ( )3 (ab2 )−3 4x 2 −4 3 1. (2x y ) 2. 3. (a2 b)−5 5y ( 4. y

4

x y3

)−5

7. (a−1 − b−1 )−1

( 5.

8.

x5 9

)2 ÷ (3x−2 )−2

6.

(3x−4 )2 (x3 y 2 )4

a−2 + b−2 (ab)−2

Soluciones. Se aplican las leyes de los exponentes 1. (2x2 y −4 )3 = 23 (x2 )3 (y −4 )3 = 8x6 y −12 =

8x6 . y 12

2. (ab2 )−3 a−3 (b2 )−3 a−3 b−6 a−3 b−6 = = = · = a−3−(−10) b−6−(−5) (a2 b)−5 (a2 )−5 b−5 a−10 b−5 a−10 b−5 a7 7 −1 = ab = . b

2.2. Exponentes

( 3.

4x 5y (

4. y

4

)3 =

x y3

43

64x3 . 125y 3

)−5 = y4

−5 x−5 y 19 4 x −5 4−(−15) −5 19 = y = x y = x y = . (y 3 )−5 y −15 x5

5. (

6.

x5 9

)2

(x5 )2 1 x10 1 ÷ = ÷ 2 (−2)(2) 2 −2 2 2 9 (3x ) 9 3x 10 10 −4 x 1 (x )(9x ) x10−4 1 = ÷ = = = x6 2 −4 2 2−1 9 9x 9 ·1 9 9

÷ (3x−2 )−2 =

(3x−4 )2 32 x(−4)(2) 9x−8 9 = = = . (x3 y 2 )4 (x3 )4 (y 2 )4 x12 y 8 x20 y 8

7. En primer lugar se simplifica la expresi´on que se encuentra dentro del par´entesis y se obtiene (

(a

−1

−1 −1

−b )

)−1 ( )−1 1 1 b−a = − = a b ab −1 (b − a) ab = = . (ab)−1 b−a

8. Como a0 = b0 = 1 se tiene a−2 + b−2 a−2 b−2 = + = a−2+2 b2 + a2 b−2+2 = b2 + a2 . (ab)−2 (ab)−2 (ab)−2



Ejercicio 2.2.1 Simplificar las siguientes expresiones. Expresar el resulta-

44

Cap´ıtulo 2.

´ Algebra

do sin par´entesis y sin exponentes negativos. 1. (94 )3

2. (52 )8

3. (z 5 )2

4. (x7 )6

5. (−x2 )5

6. (−x3 )4

7. x7 · x6

8. b2 · b4

9. y −6 · y 11

10. z 3 · z −10

11. z −2 · z 4

12. (2x)3 x−5

13. (3x)−4 x7

14. (2x)2 (2x−1 )4

15.

16. (x3 y 2 z)2 (xz 2 )5

17. (2xy 3 )4 (x5 z)3

18. (y −3 w2 )−5

19. (ab−3 )−1

20. (x2 yw4 )−2 (xyw)6

21. (pq 5 r)2 (q −1 r)−2

(42 )3 22. 163

(54 )2 23. 57

( )−2 1 24. ÷ 9−5 9

( )4 1 25. ÷ 6−3 6

26.

x3 x−4

27.

z −2 z −13

x4 (5x−1 )3 4

28.

(y 3 )2 y4

29.

x−12 (x2 )6

30.

(c−2 )9 (c6 )−3

31.

(r−7 )2 (r3 )3

32.

(−y 4 )2 (−y)−3

33.

(−z −2 )−3 (−z 2 )−4

34.

(a3 b2 )−3 (ab)4

35.

(xy −3 )−2 (x2 y)3

36.

(−2xy)5 x5 y

(−xy 2 z)−2 37. x−3 y 2 z −1

(−6x)2 38. −6x2

40. 7x3 (x2 + 7x−2 )

41. 5x4 (x−3 − 9x)

(2a2 b)−2 39. (−4ab3 )2

2.2. Exponentes

45

42. x6 (4x2 + 2x − 5x−3 )

43. 3x−6 (4x8 − 4x6 + x2 )

44. (5−3 + x−3 )−1

45. [(5x)−2 + (5y)−2 ]−1

46. x−1 ÷ (x + x−1 )−1

47. (xy)−1 (x−1 + y −1 )−1

48. (x

−1

+y )

( 50. x

( )( ) ( )2 6 5 5 49. − y 12y 2y

−1 −1

−4

9 3x

)−1

(

1 − − 4x

52.

7 4 + −4 15y 20y −3

54.

1 1 − 5y −5 2y −5

)2 51.

3 3 + −3 −3 2y 4y ( 2 4) ( ) xy 5 9 55. ÷ ÷ 6 y x4 53.

(

a−2 a 56. − 5 2 4a 2a ( 58.

6x 3 − 20y 3 13xy

57. x

−7

y3 3xy ÷ 4 4x

) ( 3 ) 4 x 1 −2 +x ÷ + −3 x 4 6x

Soluciones. 1. 912

2. 516

3. z 10

4. x42

5. −x10

6. x12

7. x13

8. b6

9. y 5 13.

x3 81

10. z −7 14.

64 x2

11. z 2 15.

125x 4

12.

8 x2

16. x11 y 4 z 12

)

46

Cap´ıtulo 2.

y 15 w10

b3 a

x2 y 4 w2

17. 16x19 y 12 z 3

18.

21. p2 q 12

22. 1

23. 5

24. 97 28. y 2

19.

20.

25.

1 6

26. x7

27. z 11

29.

1 x24

30. 1

31.

1 r23

32. −y 11

35.

y3 x8

36. −32y 4

39.

1 40. 7x5 + 49x 64a6 b8

33. −z 14 37.

x y6z

34.

1 13 a b10

38. −6

41. 5x − 45x5

42. 4x8 + 2x7 − 5x3

43. 12x2 − 12 +

44.

125x3 x3 + 125

45.

25x2 y 2 x2 + y 2

46.

47.

1 y+x

48.

xy y+x

49. −

50.

16 − 3x 48x3

51.

39x2 − 30y 2 130xy 3

52.

7y 4 + 3y 3 15

53.

9y 3 4

54. −

55.

3y 5 10x2

58.

48x + 12 5x5

56. −

2.2.2.

1 4a4

57.

3y 5 10

12 x2 y 2

´ Algebra

3 x4

x2 + 1 x2 15 4y 2

Exponentes fraccionarios y radicales

En la secci´on anterior se estudiaron s´olo exponentes enteros. Sin embargo, las leyes de los exponentes tambi´en son v´alidas para exponentes fraccionarios.

2.2. Exponentes

47

Para ello es necesario tomar en cuenta las siguientes definiciones y consideraciones. Sean n un n´ umero natural par y a ≥ 0 un n´ umero real. Se dice que el n´ umero b es la ra´ız n-´ esima principal de a si bn = a y b ≥ 0. Se escribe 1 b = an . Si n es un entero positivo impar y a es un n´ umero real arbitrario, entonces 1 n b es la ra´ız n-´ esima de a si b = a. Se sigue denotando b = a n . N´otese que las ra´ıces impares est´an definidas para todos los n´ umeros reales a, mientras que las ra´ıces pares u ´nicamente se definen cuando a es no 1 negativo y en este caso −a n tambi´en es una ra´ız n-´esima de a. Ejemplo

2.2.4 Calcular las siguientes ra´ıces.

1

1

1

1

1

1. 64 3 2. 81 4 3. (−32) 5 4. (15625) 6 5. (−100) 4 1

6. 1 n , n un entero positivo 7.

1

(−1) n , n un entero positivo impar

Soluci´ on. Aplicando la definici´on de ra´ız n-´esima. 1

1. 64 3 = 4, ya que 43 = 64 . 1

2. 81 4 = 3, porque 34 = 81 . 1

3. (−32) 5 = −2, ya que (−2)5 = −32 . 1

4. (15625) 6 = 5, porque 56 = 15625 . 1

5. (−100) 4 no existe puesto que las ra´ıces n-´esimas con n par u ´nicamentes est´an definidas para n´ umeros no negativos. 1

6. 1 n = 1, ya que 1n = 1 para todo entero positivo. 1

7. (−1) n = −1, porque (−1)n = −1 para todo entero positivo impar.  1

Tambi´en se denota a n como 1 2

√ n

a. El s´ımbolo

√ n



se denomina ra´ız n-

por a y se le llama la√ra´ız ´ esima. Si n = 2, a se representa simplemente √ 1 1 3 3 cuadrada de a. An´alogamente a = a es la ra´ız c´ ubica de a; a 4 = 4 a es

48

Cap´ıtulo 2.

´ Algebra

la ra´ız cuarta de a y as´ı sucesivamente. Los resultados del ejemplo anterior (2.2.4), pueden reescribirse utilizando esta notaci´on: √ √ √ √ 6 1. 3 64 = 4 . 2. 4 81 = 3 . 3 5 −32 = −2 . 4. 15625 = 5 . √ 5. 4 −100 no existe. 6. 7.

√ n √ n

1=1,

con n un entero positivo.

−1 = −1 ,

para n un entero positivo impar.

La potencia fraccionaria de un n´ umero real a se define a continuaci´on. Sea n un entero positivo y m un entero diferente de cero. Entonces la potencia fraccionaria m de a es n m

1

a n = (a n )m . En la definici´on anterior se observa que si n es par, a debe ser mayor o igual que cero, y que si m es negativo, a debe ser distinto de cero. Ejemplo

2.2.5 Utilizando la definici´on calcular: 2. 9− 2

5

1

1. 4 2

3. 256− 4 3

Soluci´ on. 5

2. 9− 2 = (9 2 )−1 = 3−1 =

1

1

1. 4 2 = (4 2 )5 = 25 = 32 .

1

1 . 3

1 . 64  Se puede generalizar el resultado de 2 del ejemplo anterior como sigue:

3. 256− 4 = (256 4 )−3 = 4−3 = 3

1

a− n = (a n )−1 = 1

esto es

1

1 1

an

,

1 1 a− n = √ . n a Existe un procedimiento alternativo para calcular una potencia fraccionaria. Se enuncia este resultado en el siguiente teorema:

2.2. Exponentes

49 m

Teorema 2.2.1 Si a n existe, entonces: m

1

a n = (am ) n . Ejemplo 3

1. 25 2

2.2.6 Calcular de dos maneras distintas: 3

2. 81 4

Soluci´ on. 1. Una primera manera es 3

1

25 2 = (25 2 )3 = 53 = 125 y otra forma es 3

1

1

1

25 2 = (253 ) 2 = ((52 )3 ) 2 = (56 ) 2 = 53 = 125 . 2. Ahora bien 3

1

81 4 = (81 4 )3 = 33 = 27 y por otro lado 3

1

1

1

81 4 = (813 ) 4 = ((34 )3 ) 4 = (312 ) 4 = 33 = 27 . m



De este ejemplo se concluye que es m´as f´acil calcular a n utilizando la m 1 m 1 definici´on a n = (a n )m , que utilizando la forma a n = (am ) n . Se observa ahora el hecho de que con estas definiciones, se sigue que las leyes de los exponentes vistas en la secci´on anterior tambi´en son v´alidas para exponentes fraccionarios. Si p y q son n´ umeros racionales, a y b son n´ umeros reales, y las potencias involucradas est´an bien definidas, entonces: I´.El producto de dos potencias con base igual es

ap aq = ap+q .

50

Cap´ıtulo 2.

´ Algebra

II´. El cociente de dos potencias con base igual es

ap = ap−q . aq III´. La potencia de otra potencia es

(ap )q = apq . IV´. La potencia de un producto es

(ab)p = ap bp . V´. La potencia de un cociente es

( a )p b

=

ap . bp

Al emplear estas leyes se debe tener presente que en cualquier potencia, si el exponente es negativo, la base no debe ser cero; y que si el exponente contiene una ra´ız par, entonces la base no debe ser negativa. Tres casos particulares de las leyes de los exponentes son utilizadas con frecuencia utilizando la notaci´on de radicales y se les conoce como las leyes de los radicales. III´´. El m−´esimo radical de un n−´esimo radical es

√ m

√ n

a=

√ a.

mn

IV´´. El n−´esimo radical de un producto es

√ √ √ n n ab = n a b . V´´. El n−´esimo radical de un cociente es

2.2. Exponentes

51

√ n

Ejemplo

√ n a a = √ . n b b

2.2.7 Simplificar. 13

2 3

1. 7 · 7 4

2. 6

− 52

·6

2

3.

4.

5

10 4

21

x5 5. x3 9.



2

√ 4 7

a5 b 3

√ 13.

7. (8− 9 )− 3

11

6. (45 ) 7

3

√ 7

−6 − 12

10. (7x y )

√√ 3 14. 64x12 y 30

89

Soluciones. 2

2

14

1. 74 · 7 3 = 74+ 3 = 7 3

5 5 1 1 2. 6− 2 · 62 = 6− 2 +2 = 6− 2 = √ 6 13

3.

10 4

= 10 4 − 4 = 102 = 100 13

5 4

10

3− 2 3

4.

3

=

3 2

5

1 3

3 −( −3 ) 2 2

=

1 1 = 3 3 27

21 √ 21 6 x5 5 5. 3 = x 5 −3 = x 5 = x6 x 11

11

55

6. (45 ) 7 = 45 7 = 4 7

7. (8− 9 )− 3 = 8(− 9 )(− 3 ) = 8 27 8

2

a2 b =

√ √ 3 3 a2 b

2

8.

√ 3

8

16

11.

3− 2 3

10 4

8

8.

3

32 √ 3

a2 b

√ 4a 9b

10

12.

5

a3 b5

52

Cap´ıtulo 2.

9.



a5 b3 =

´ Algebra

√ √ a5 b 3

4 1 1 4 1 1 1 2 y3 10. (7x 7 y −6 )− 2 = 7− 2 x( 7 )(− 2 ) y (−6)(− 2 ) = 7− 2 x− 7 y 3 = √ √ 7 7 x2

√ 11.

√ √ √ √ 2 a 4a 4a 4 a = √ = √ √ = √ 9b 9b 9 b 3 b

√ 12.

5

√ 13. 14.

3

10 3

a b5 √ 7

89 =

√√ 3

Ejemplo

√ √ 5 10 1 2 10 3 (a 3 ) 5 a3 a3 a2 = √ = = = 1 5 5 b b b (b5 ) 5 √

21

89

64x12 y 30 =

√ 6

64x12 y 30 =

√ 6

√ √ 6 64 x12 6 y 30 = 2x2 y 5



64 2.2.8 Determinar r tal que √ = 4r . 5 16

Soluci´on. Se expresa el numerador y el denominador como potencia de 4, es decir 13 64 43 43 43 3− 25 √ = 1 = =45. 1 = 2 = 4 5 2 16 16 5 (4 ) 5 45

Por consiguiente r = Ejemplo ( 1.



2.2.9 Evaluar:

48 1+ 121

Soluci´ on.

13 . 5

) 21

( 2.

125y 6 8

)− 43

2.2. Exponentes

53

1. Se realiza la suma indicada ( )1 ( ) 1 ( 2 ) 12 48 2 169 2 13 1+ = = 121 121 112 1 [( ) ] 2 2 13 = 11 ( )(2) 12 13 = 11 ( )1 13 = 11 13 = 11

se usa la ley V´

se aplica la ley III´

2. (

125y 6 8

)− 34

(

)− 43 53 y 6 = 23 [( )3 ]− 43 5y 2 = 2 ( 2 )−4 5y = 2 ( )4 2 = 5y 2 24 = (5y 2 )4 16 = 625y 8

se aplica la ley V

se emplea la ley III

se usa la ley V

se aplica la ley IV

 Ejemplo

2.2.10 Simplificar la expresi´on siguiente. 4k · 9k · 5k · 6k 8k · 93k/2 · 10k

54

Cap´ıtulo 2.

´ Algebra

Soluci´ on. En este caso es conveniente expresar todas las bases en t´erminos de sus factores primos. 4k · 9k · 5k · 6k (22 )k · (32 )k · 5k · (2 · 3)k = 8k · 93k/2 · 10k (23 )k · (32 )3k/2 · (2 · 5)k 22k · 32k · 5k · 2k · 3k = 23k · 33k · 2k · 5k 2k 3k 2 3 = 3k 3k 2 3 = 22k−3k

por las leyes III y IV

simplificando factores comunes

= 2−k 1 = k 2 Ejemplo



2.2.11 Simplificar la expresi´on siguiente √ √ 128 − 18 √ 3 8

Soluci´ on. Primero se observa que los n´ umeros dentro de los radicales pueden expresarse como el producto de dos n´ umeros, tales que uno de ellos tiene ra´ız cuadrada exacta. √ √ √ √ √ = = = 8 128 64 · 2 64 2 √ √ √ √ √2. √18 = √9 · 2 = √9√2 = 3√2. 8 = 4·2 = 4 2 = 2 2. Luego √

√ √ √ 8 2−3 2 128 − 18 √ √ = 3 8 3(2 2) √ 5 2 = √ 6 2 5 = 6



2.2. Exponentes

55

Ejemplo

2.2.12 Simplificar ( √ ) √ √ √ √ √ √ 4 3 4 1. x7 16x − x 2. 3 x(9 x5 + x2 )

√ 3.

x − 10x2 √ 7 x

Soluciones. 1. √ 4

( x7

√ 4

√ ) √ √ √ √ √ 4 4 16x − x = x7 ( 16 4 x − 4 x) 7

1

7

1

1

= x 4 (2x 4 − x 4 ) = x4 · x4 = x2 2. √ 3

√ √ 5 2 1 3 x(9 x5 + x2 ) = x 3 (9x 2 + x 3 ) 1

5

1

2

= 9x 3 · x 2 + x 3 · x 3 17

= 9x 6 + x √ 6 = 9 x17 + x 3. √

1

x − 10x2 x 2 − 10x2 √ = 1 7 x x7 1 1 = (x 2 − 10x2 )(x− 7 ) = x 2 (x− 7 ) − 10x2 (x− 7 ) 1

5

1

1

13

= x 14 − 10x 7

Se concluye esta secci´on con las siguientes observaciones: √ √ √ √ (i) a2 + b2 ̸= a + b y a + b ̸= a + b √ (ii) ( n a)n = a, donde a ≥ 0 si n es par



56

Cap´ıtulo 2.

´ Algebra

Ejercicio 2.2.2 En los ejercicios 1 a 6 determinar el valor de k para que se cumpla la igualdad. √



√ 1. 27 3 = 3k

2.

3 = 3k 81



√ √ 3 4. 4 8 2 = 2k

5.



3.

3

9 = 3k 27

√√ 4

3 = 9k

3

6.



3 = 3k

En los ejercicios 7 a 19 evaluar la expresi´on dada.

7.



100

8.

√ 10. 13.

3

10 27

2+



11.

16. (16

√ 5

√ 81

−243

12.

· 81 )

1 10

−0. 064 1

( )− 85 1 18. 36 6

− 73

2 9

√ 3

13 36

15. (0. 09)− 2

5

5 2

1+

9.

14. (16)− 4

(−2)2 −5

√ 3

1 4

17. 27 · 9

19. 8 3 ÷ 81− 4 1

3

En los ejercicios 20 a 42 simplificar cada expresi´on. ( 10

20. (32x ) √ 23.

3

26. (x

4 5

21.

125w6 8z 3

1/3

·x

24.

−1/2 3

)

64x3 27

√ 4

) 43

22. (16x4 y −20 ) 4 3

√ 3 x5/3 8x1/3 −6 −1/2

27. (64x )

25. (x−2/7 · x4/3 )4

÷ (27x )

6 2/3

x2/5 y −7/3 28. −8/5 4/3 x y

2.2. Exponentes

57

(

w11/6 z −7/6 w−1/6 z −1/12

)12

a10/7 b5/6 29. −2/7 1/3 x y

30.

(ab3 )−1/2 (a2 b)1/3 31. (a−5 b4 )−1/20

32. (3a3 b4 )1/7 (9−2 ab−1 )−8/7 √ √ 34. 3 50 − 10 32

4x4/3 2x5/9 33. ÷ 3/25 y 1/5 y √ √ 35. 5 63 + 9 28

√ √ 36. 7 45 + 2 20

√ √ 6 27 − 5 3 √ 37. 12

√ √ 38. −6 3 −24 − 2 3 −128

39. z 4/5 · z −2/3 · (z 3 )−1/8 ·

41.

1

40. x1/2 · y −3/4 ·

(z 2/15 )4

8k · 9k/3 · 25k · 63k 4k/2 · 9k/2 · 103k

42.

( y )5/9 x

·

x73/18 y −79/36

202k · 28k · 30k · 12k 63k · 85k/3 · 15k

43. Determinar si las siguientes igualdades son verdaderas o falsas.

(a) (d) (g)

(h)

√ √

12 =



7+



5

16 + 9 = 4 + 3 = 7

(b)



12 =





(e) xm · xn = xmn

√ √ √ 4 x3x= 7x





x2 = x, para todo n´ umero real x

2

(c) (f )



−36 = −6

xm = xm/n xn

58

Cap´ıtulo 2.

´ Algebra

Soluciones. 1. k =

7 2

2. k = −

5. k =

1 8

6. k =

7 2

3. k = −

1 24

10.

4 3

11. −3

13. 2

14.

1 32

15.

17.

1 34/21

18. 69/8

256x4 81

22.

25. x88/21

26.

21.

a10/7 b5/6 x2/7 29. y 1/3 33.

2x7/9 y 2/25

37.

13 2

40. x4 y 2

8x3 y 15 1 x1/2

23 6

√ 8. 3 3 3 12. −

10 3

16.

2 5

3 4

20. 16x8

19. 54 23.

5w2 2z

24. 2x19/36

27.

1 72x

28.

x2 y 11/3 33

w24 30. z 13

1 31. 1/12 29/30 a b

3 7 b12/7 32. a5/7

√ 34. −25 2

√ 35. 33 7

36. 55/2

√ √ 38. 12 3 3 + 8 3 2

41.

4. k =

7. 10

7 6

9.

1 3

22k 38k/3 5k

39.

42.

1 z 31/40 52k 24k 3k

43. (a) Falsa

(b) Verdadera

(c) Falsa

(d) Falsa

(e) Falsa

(f ) Falsa

(g)

(h) Falsa

Falsa

2.3. Operaciones algebraicas

2.3.

59

Operaciones algebraicas

2.3 Operaciones algebraicas En esta secci´on se estudian las operaciones con expresiones algebraicas.

2.3.1.

Suma y resta

La suma o resta de expresiones algebraicas se realiza reduciendo t´erminos semejantes. Las siguientes pautas resultan u ´tiles al efectuar operaciones. 1. Eliminar s´ımbolos de agrupamiento de la siguiente manera: a) Si al s´ımbolo le antecede un signo positivo, equivale a multiplicar por +1 todos los t´erminos del interior, por lo cual los t´erminos permanecen iguales, en particular conservan su signo. b) Si el s´ımbolo es antecedido por un signo negativo, equivale a multipicar por −1 todos los t´erminos del interior y as´ı, al quitarlo, los t´erminos cambian de signo. c) Si existen varios s´ımbolos de agrupamiento anidados, es decir, uno dentro del otro, se van eliminando a partir del m´as interno, conforme a las dos reglas antes descritas. Tambi´en es v´alido hacerlo a partir del m´as externo. 2. Reducir t´erminos semejantes. Es decir, sumar o restar los coeficientes de t´erminos semejantes. A menudo se reagrupan los t´erminos de la expresi´on de tal manera que los t´erminos semejantes aparezcan juntos.

Ejemplo

2.3.1 Efectuar las siguientes operaciones.

1. (7x2 + 6x − 7) + (8x2 − 2x) − (4x2 + 10x − 11) 2. 10x3 − [6x3 − (2x − 4) + (−4x3 + x2 − 9x − 7)] Soluciones.

60

Cap´ıtulo 2.

´ Algebra

1. Se observa que el signo (−) que antecede al tercer par´entesis afecta no s´olo al primer t´ermino de la expresi´on dentro de ´el, sino a todos los t´erminos en su interior. (7x2 +6x − 7) + (8x2 − 2x) − (4x2 + 10x − 11) = 7x2 + 6x − 7 + 8x2 − 2x − 4x2 − 10x + 11 = 7x2 + 8x2 − 4x2 + 6x − 2x − 10x − 7 + 11

se eliminan los par´ entesis se agrupan t´ erminos se reducen t´ erminos semejantes

= 11x2 − 6x + 4 2. Este ejemplo presenta anidamiento en la agrupaci´on. Se eliminan primero los s´ımbolos m´as internos 10x3 − [6x3 − (2x − 4) + (−4x3 + x2 − 9x − 7)] = 10x3 − [6x3 − 2x + 4 − 4x3 + x2 − 9x − 7] = 10x3 − 6x3 + 2x − 4 + 4x3 − x2 + 9x + 7 = 10x3 − 6x3 + 4x3 − x2 + 2x + 9x − 4 + 7

se eliminan los par´ entesis se elimina el corchete se agrupan t´ erminos se reducen t´ erminos semejantes

= 8x3 − x2 + 11x + 3  Ejemplo

2.3.2 Realizar las siguientes operaciones

1. Sumar 7x3 y 2 + 3x2 y − 8y + 1

11 + 4y − 10x2 y − 9x3 y 2 .

y

2. Restar 6 − 2x − 8xy − 10xy 2 + 7x2 y 4

de

15x2 y 4 + 3xy 2 − 8xy + x .

3. Restar − Soluciones.

√ 2√ √ x + 8 xy − 6 3y 3

de

√ 1√ √ x − 7 xy + 4 3y . 3

2.3. Operaciones algebraicas

61

1. La suma requerida es 7x3 y 2 + 3x2 y − 8y + 1 + (11 + 4y − 10x2 y − 9x3 y 2 )

se elimina el par´ entesis

= 7x3 y 2 + 3x2 y − 8y + 1 + 11 + 4y − 10x2 y − 9x3 y 2

se agrupan t´ erminos semejantes

= 7x3 y 2 − 9x3 y 2 + 3x2 y − 10x2 y − 8y + 4y + 1 + 11

se reducen t´ erminos semejantes

= −2x3 y 2 − 7x2 y − 4y + 12 2. La resta es 15x2 y 4 + 3xy 2 − 8xy + x − (6 − 2x − 8xy − 10xy 2 + 7x2 y 4 ) = 15x2 y 4 + 3xy 2 − 8xy + x − 6 + 2x + 8xy + 10xy 2 − 7x2 y 4 = 15x2 y 4 − 7x2 y 4 + 3xy 2 + 10xy 2 − 8xy + 8xy + x + 2x − 6 = 8x2 y 4 + 13xy 2 + 3x − 6

se elimina el par´ entesis se agrupan t´ erminos semejantes se reducen t´ erminos semejantes

3. Ahora se calcula √ √ 2√ 1√ √ √ x−7 xy + 4 3y − (− x + 8 xy − 6 3y) 3 3 √ √ 1√ 2√ √ √ = x − 7 xy + 4 3y + x − 8 xy + 6 3y 3 3 √ √ 1√ 2√ √ √ x+ x − 7 xy − 8 xy + 4 3y + 6 3y 3 3 √ √ √ = x − 15 xy + 10 3y =

2.3.2.

se elimina el par´ entesis se agrupan t´ erminos semejantes erminos se reducen t´ semejantes

Multiplicaci´ on

La multiplicaci´on de expresiones algebraicas se estudia por casos.



62

Cap´ıtulo 2.

´ Algebra

Multiplicaci´ on de monomios El caso m´as simple corresponde a la multiplicaci´ on de monomios. El procedimiento usa los siguientes pasos. 1. Se multiplican los coeficientes, aplicando las reglas de los signos: (+)(+) = + ,

(+)(−) = − ,

(−)(+) = − ,

(−)(−) = + .

2. Se escribe a continuaci´on el producto de todas las variables distintas que se encuentren en los monomios a multiplicar aplicando las leyes de los exponentes. Se recuerda que si una variable no presenta exponente, ´este es igual a 1.

Ejemplo

2.3.3 Efectuar las multiplicaciones

1. 12(−7x3 y 4 z)

2. (−4a5 b2 )(−8ab6 c3 )

√ √ 3. (9 xy 3 )(2x2 y 4 )(−4x3/2 y) Soluciones. 1. 12(−7x3 y 4 z) = 12(−7)(x3 y 4 z) = −84x3 y 4 z 2. (−4a5 b2 )(−8ab6 c3 ) = (−4)(−8)a5 b2 ab6 c3 = 32a6 b8 c3 3. √ √ (9 xy 3 )(2x2 y 4 )(−4x3/2 y) = −72(x1/2 y 3 )(x2 y 4 )(x3/2 y 1/2 ) = −72x4 y 15/2



Multiplicaci´ on de un monomio por un multinomio En este caso se aplica la propiedad distributiva. Se multiplica el monomio por todos los t´erminos que constituyen el multinomio, en la forma descrita previamente y todos los resultados obtenidos se suman algebraicamente. Ejemplo

2.3.4 Efectuar la multiplicaci´on indicada y simplificar.

2.3. Operaciones algebraicas

1. −2x2 y(x3 + 7xy − y 3 ) 2. 8ab2 c(6ab − 9a3 bc4 + 5) ( )( ) x 5y 2 4 3. 3x y − y x √ √ √ 4. −2 wz 2 (−3 w + 7wz − 4 z) Soluciones. 1. −2x2 y(x3 + 7xy − y 3 ) = (−2x2 y)x3 + (−2x2 y)(7xy) − (−2x2 y)(y 3 ) = −2x5 y − 14x3 y 2 + 2x2 y 4 2. 8ab2 c(6ab − 9a3 bc4 + 5) = (8ab2 c)(6ab) − (8ab2 c)(9a3 bc4 ) + (8ab2 c)(5) = 48a2 b3 c − 72a4 b3 c5 + 40ab2 c 3. ( )( ) ( )( 2 4) ( )( ) 3x y 5y x 5y x x 2 4 3x y − = − y x y 1 y x 2 4 (x)(3x y ) (x)(5y) = − (y)(1) (y)(x) 3 3 = 3x y − 5 4. √ √ √ −2 wz 2 (−3 w + 7wz − 4 z) √ √ √ √ √ = (−2 wz 2 )(−3 w) + (−2 wz 2 )(7wz) − (−2 wz 2 )(4 z) √ = 6wz 2 − 14w3/2 z 3 + 8 wz 5/2 

63

64

Cap´ıtulo 2.

´ Algebra

Multiplicaci´ on de dos multinomios Consiste en aplicar nuevamente la propiedad distributiva. Se multiplica cada uno de los t´erminos del primer multinomio por el segundo multinomio, y se suman algebraicamente los productos obtenidos. Ejemplo

2.3.5 Multiplicar los polinomios y simplificar.

1. (3x − 5)(x2 − 4x + 7)

2. (6x − y + 4z)(x − 2y + z)

Soluciones. 1. Se indica en cada paso la operaci´on a realizar o la propiedad a utilizar. (3x−5)(x2 − 4x + 7) = 3x(x2 − 4x + 7) − 5(x2 − 4x + 7) = (3x)(x2 ) − (3x)(4x) + (3x)(7) + (−5)(x2 ) − (−5)(4x) + (−5)(7) = 3x3 − 12x2 + 21x − 5x2 + 20x − 35 = 3x3 − 12x2 − 5x2 + 21x + 20x − 35 = 3x3 − 17x2 + 41x − 35

propiedad distributiva monomios por un multinomio propiedad distributiva

se agrupan t´ erminos semejantes se reducen t´ erminos semejantes

2. (6x − y + 4z)(x − 2y + z) = 6x(x − 2y + z) − y(x − 2y + z) + 4z(x − 2y + z) = (6x)(x) − (6x)(2y) + (6x)(z) + (−y)(x) − (−y)(2y) + (−y)(z) + (4z)(x) − (4z)(2y) + 4z(z) = 6x2 − 12xy + 6xz − xy + 2y 2 − yz + 4xz − 8yz + 4z 2 = 6x2 + 2y 2 + 4z 2 − 12xy − xy + 6xz + 4xz − yz − 8yz = 6x2 + 2y 2 + 4z 2 − 13xy + 10xz − 9yz

propiedad distributiva monomios por un multinomio propiedad distributiva

se agrupan t´ erminos semejantes se reducen t´ erminos semejantes

2.3. Operaciones algebraicas

65

 Ejemplo

2.3.6 Efectuar las operaciones indicadas.

√ √ √ √ 1. (2 x − 3 y)(4 x − 5 y) ( ) 2 2 2. 3x − (−4x3 − 6x) x ( )( ) x 4y 2 3. 8xy − 2xy + y x 4. (3a2 b − 4ab2 )(a2 − ab + b2 ) 5. ab2 [a4 − (2ab + a3 b2 )] + 7a2 b[a3 b − (3b2 − a2 b3 )] 6. 3a{a3 − 2a[4a − 6(a2 − 4)]} + (2a − 3)(4 − a) Soluciones. 1. √ √ √ √ (2 x − 3 y)(4 x − 5 y) √ √ √ √ = 8x − 10 x y − 12 x y + 15y √ = 8x − 22 xy + 15y

propiedad distributiva se reducen t´ erminos semejantes

2. Se multiplican los multinomios para obtener ( ) 2 2 3x − (−4x3 − 6x) = (3x2 − 2x−1 )(−4x3 − 6x) x = −12x5 − 18x3 + 8x2 + 12

propiedad distributiva

3. Se multiplican los multinomios entre s´ı ( )( ) x 4y 2 8xy − 2xy + = (8xy − xy −1 )(2xy 2 + 4x−1 y) y x = 16x2 y 3 + 32y 2 − 2x2 y − 4

propiedad distributiva

66

Cap´ıtulo 2.

´ Algebra

4. Se multiplican los multinomios entre s´ı (3a2 b − 4ab2 )(a2 − ab + b2 ) = 3a4 b − 3a3 b2 + 3a2 b3 − 4a3 b2 + 4a2 b3 − 4ab4 = 3a4 b − 7a3 b2 + 7a2 b3 − 4ab4

propiedad distributiva se reducen t´ erminos semejantes

5. ab2 [a4 − (2ab + a3 b2 )] + 7a2 b[a3 b − (3b2 − a2 b3 )] = ab2 [a4 − 2ab − a3 b2 ] + 7a2 b[a3 b − 3b2 + a2 b3 ] = a5 b2 − 2a2 b3 − a4 b4 + 7a5 b2 − 21a2 b3 + 7a4 b4 = a5 b2 + 7a5 b2 − 2a2 b3 − 21a2 b3 − a4 b4 + 7a4 b4

se eliminan los par´ entesis monomios por multinomios se agrupan t´ erminos semejantes se reducen t´ erminos semejantes

= 8a5 b2 − 23a2 b3 + 6a4 b4 6. 3a{a3 − 2a[4a − 6(a2 − 4)]} + (2a − 3)(4 − a) = 3a{a3 − 2a[4a − 6a2 + 24]} + 8a − 2a2 − 12 + 3a = 3a{a3 − 8a2 + 12a3 − 48a} − 2a2 + 11a − 12 = 3a{13a3 − 8a2 − 48a} − 2a2 + 11a − 12 = 39a4 − 24a3 − 144a2 − 2a2 + 11a − 12

se multiplica y eliminan los par´ entesis se multipica y se eliminan corchetes se reducen t´ erminos semejantes se multiplica y agrupan t´ erminos semejantes se reducen t´ erminos semejantes

= 39a4 − 24a3 − 146a2 + 11a − 12  Un caso particular en la multiplicaci´on de multinomios es cuando los factores son polinomios, en una misma literal. El algoritmo usado es semejante al de la multiplicaci´on de n´ umeros reales, como se ilustra en el siguiente ejemplo. Ejemplo

2.3.7 Multiplicar los polinomios 2x3 − x2 + 5x − 1 y x2 − 3x + 2.

2.3. Operaciones algebraicas

67

Soluci´ on. Se escribe un arreglo vertical para realizar la multiplicaci´on, ordenando los t´erminos en forma decreciente, con respecto al grado. As´ı 2x3

5

2x 2x5

−6x4 −x4 −7x4

4x3 +3x3 +5x3 +12x3

−x2 x2 −2x2 −15x2 −x2 −18x2

+5x −3x +10x +3x

−1 +2 −2

+13x

−2

por tanto (2x3 − x2 + 5x − 1)(x2 − 3x + 2) = 2x5 − 7x4 + 12x3 − 18x2 + 13x − 2 . Se usar´a la notaci´on f (x) = an xn + an−1 xn−1 + · · · + a2 x2 + a1 x + a0 con a0 , a1 , . . . , an n´ umeros reales, para denotar polinomios en x.

2.3.3.

Divisi´ on

La divisi´on, al igual que la multiplicaci´on, tambi´en se aborda por casos y se enfatiza el caso de los polinomios. Divisi´ on de monomios Este caso ha sido ya ilustrado en parte cuando se estudiaron las leyes de exponentes. El procedimiento sugerido es el siguiente. 1. Se dividen los coeficientes, aplicando la regla de los signos: + =+, +

+ =−, −

− =−, +

− =+. −

2. Se aplican las leyes de los exponentes a las literales iguales que aparezcan en el numerador y en el denominador. Si el exponente obtenido en alguna literal a es igual a cero, se recuerda que a0 = 1. Las literales que s´olo aparecen una vez permanecen invariantes.

68

Cap´ıtulo 2.

Ejemplo 1.

´ Algebra

2.3.8 Realizar las divisiones indicadas.

32x4 y 3 z 2 −8x2 y 6

2.

−3a2 b5 z −27a7 b5 z 3

3.

9p8 q 4 r10 5p7 q 2 r10

Soluciones. 32x4 y 3 z 2 4x2 z 2 1. = − −8x2 y 6 y3 2.

−3a2 b5 z 1 = −27a7 b5 z 3 9a5 z 2

3.

9p8 q 4 r10 9pq 2 9 = = pq 2 7 2 10 5p q r 5 5



on de un polinomio entre un monomio Divisi´ Se divide cada uno de los t´erminos del polinomio entre el monomio divisor, de acuerdo con el procedimiento antes descrito y se suman algebraicamente los resultados obtenidos de cada divisi´on. Ejemplo 1.

2.3.9 Efectuar la divisi´on dada.

15x6 − 7x3 5x

2.

9x3 + 6x2 − 3x + 12 −3x3

8a4 − 10a2 b2 + 14b − 6 3. 2ab2 Soluciones. 15x6 − 7x3 15x6 7x3 7 1. = − = 3x5 − x2 5x 5x 5x 5 2. 9x3 + 6x2 − 3x + 12 9x3 6x2 3x 12 = + − + 3 3 3 3 −3x (−3x ) (−3x ) (−3x ) (−3x3 ) 2 1 4 = −3 − + 2 − 3 x x x

2.3. Operaciones algebraicas

69

3. 8a4 10a2 b2 14b 6 8a4 − 10a2 b2 + 14b − 6 = − + − 2 2 2 2 2ab 2ab 2ab 2ab 2ab2 4a3 7 3 = 2 − 5a + − 2 b ab ab Ejemplo 1.



2.3.10 Realizar las divisiones.

3z 2 − 4z + 5 √ 2 z

2.

y 3 + 6y 2 − 12y − 8 √ 4y y

Soluciones. En ambos ejemplos el denominador tiene potencia fraccionaria, el procedimiento descrito se aplica nuevamente, ya que, est´a justificado por las leyes de los exponentes. 1. 3z 2 − 4z + 5 3z 2 − 4z + 5 3z 2 4z 5 √ = = − 1/2 + 1/2 1/2 1/2 2z 2z 2z 2z 2 z

=

3z 3/2 5 − 2z 1/2 + 1/2 2 2z

=

√ 5 3z 3/2 −2 z+ √ 2 2 z

2. y 3 + 6y 2 − 12y − 8 y 3 + 6y 2 − 12y − 8 = √ 4y y 4y 3/2 y3 6y 2 12y 8 = 3/2 + 3/2 − 3/2 − 3/2 4y 4y 4y 4y 1 6 3 2 = y 3/2 + y 1/2 − 1/2 − 3/2 4 4 y y 1 3√ 3 2 = y 3/2 + y − √ − 3/2 4 2 y y



70

Cap´ıtulo 2.

´ Algebra

Divisi´ on de un polinomio entre otro polinomio En una divisi´on de polinomios, el polinomio del numerador se llama dividendo y el polinomio del denominador, divisor . El siguiente ejemplo describe el procedimiento para realizar la divisi´on de un polinomio entre otro polinomio y es conocido como divisi´ on larga. Ejemplo

2.3.11 Dividir −14x2 + 17 + 4x3 entre −1 + 2x.

Soluci´on. En este caso −14x2 + 17 + 4x3 es el dividendo y 2x − 1 es el divisor y la divisi´on es −14x2 + 17 + 4x3 . −1 + 2x Antes de realizar la divisi´on los t´erminos del dividendo y del divisor se escriben en orden decreciente de las potencias de x y se escribe cero en las potencias faltantes. As´ı, la divisi´on se escribe ahora como −14x2 + 17 + 4x3 4x3 − 14x2 + 0x + 17 = . −1 + 2x 2x − 1 El arreglo para realizar la divisi´on es semejante a la divisi´on num´erica donde la notaci´on posicional se sustituye por el orden de los exponentes. Divisor

−→ 2x − 1



4x3 4x3

− + − +

2x2 14x2 2x2 12x2 12x2

+ + + − − +

6x − 3 ←− Cociente 0x + 17 ←− Dividendo ←− 1ra fila 0x + 17 ←− 2da fila 6x ←− 3ra fila 6x + 17 ←− 4ta fila 6x − 3 ←− 5ta fila 14 ←− Residuo

Los detalles de la divisi´on que aparecen en el arreglo anterior, se explican a continuaci´on. a) Primero se dividen las potencias m´aximas del dividendo y del divisor, es decir, 4x3 4x3 = = 2x2 2x 2x y as´ı se obtiene el primer t´ermino del cociente.

2.3. Operaciones algebraicas

71

b) Se multiplica el cociente 2x2 por el divisor 2x−1 y se sustrae este resultado del dividendo. Esto est´a indicado en la primera fila. El resultado de esta resta est´a en la segunda fila. c) Ahora el nuevo dividendo es el resultado de la segunda fila −12x2 + 0x + 17. d) Se repiten los pasos anteriores hasta obtener un dividendo de grado menor al del divisor. Aqu´ı se termina la divisi´on y al u ´ltimo dividendo se le denomina residuo. A continuaci´on se describe como termina la divisi´on. El segundo t´ermino del cociente se obtiene al dividir −12x2 = −6x . 2x Se multiplica el segundo t´ermino del cociente −6x por el divisor 2x − 1 y se sustrae del dividendo obtenido anteriormente, esto est´a es la tercera fila. El resultado de esta sustracci´on es el nuevo dividendo y est´a en la cuarta fila y es −6x + 17. El tercer t´ermino del cociente se obtiene al dividir −6x + 17 = −3 . 2x Se multiplica el tercer t´ermino del cociente −3 por el divisor y se sustrae del u ´ltimo dividendo y aparece en la quinta fila. El resultado de esta sustracci´on es el residuo con un t´ermino constante igual a 14. Como se trata de un polinomio de grado cero y ´este es menor que el grado uno del divisor, el procedimiento de la divisi´on concluye. El cociente de la divisi´on es 2x2 −6x−3 con residuo 14. La respuesta puede escribirse en la forma 4x3 − 14x2 + 17 14 = 2x2 − 6x − 3 + . 2x − 1 2x − 1 La divisi´on larga justifica la siguiente igualdad Residuo Dividendo = Cociente + , Divisor Divisor o equivalentemente Dividendo = (Cociente)(Divisor) + Residuo ,

72

Cap´ıtulo 2.

´ Algebra

donde el residuo tiene grado menor que el grado del divisor. La u ´ltima igualdad permite comprobar la respuesta de una divisi´on entre polinomios. En el ejemplo anterior se verifica la igualdad 4x3 − 14x2 + 17 = (2x2 − 6x − 3)(2x − 1) + 14 = 4x3 − 2x2 − 12x2 + 6x − 6x + 3 + 14 = 4x3 − 14x2 + 17

Ejemplo



2.3.12 Dividir 5x4 − 3x3 + 2x2 − 6x + 4 entre x2 − 3x + 2.

Soluci´on. En este caso 5x4 − 3x3 + 2x2 − 6x + 4 es el dividendo y x2 − 3x + 2 es el divisor y la divisi´on es 5x4 − 3x3 + 2x2 − 6x + 4 . x2 − 3x + 2 Se recurre a una divisi´on larga

x − 3x + 2 2

4

5x − 5x4

− 3x + 15x3 12x3 − 12x3 3

+ − − + −

5x2 2x2 10x2 8x2 36x2 28x2 28x2

+ 12x − 6x

+ 28 + 4

− 6x + 4 − 24x − 30x + 4 + 84x − 56 54x − 52

y as´ı 54x − 52 5x4 − 3x3 + 2x2 − 6x + 4 = 5x2 + 12x + 28 + 2 . 2 x − 3x + 2 x − 3x + 2



De manera formal el procedimiento utilizado en la divisi´on larga es denominado algoritmo de la divisi´ on, el cual se expresa en el siguiente resultado.

2.3. Operaciones algebraicas

73

Teorema 2.3.1 [Algoritmo de la divisi´ on]. Sean f (x) y d(x) ̸= 0 dos polinomios, con el grado de f (x) mayor o igual al grado de d(x). Entonces, existen dos polinomios u ´nicos, q(x) y r(x) tales que f (x) = d(x)q(x) + r(x), o equivalentemente f (x) r(x) = q(x) + d(x) d(x) donde r(x) tiene grado menor que el grado de d(x). Si el divisor d(x) es un polinomio de grado uno de la forma d(x) = x − c, entonces, de acuerdo con el algoritmo de la divisi´on, el grado del residuo es cero, es decir, es una constante r(x) = k. As´ı que la primera expresi´on del teorema anterior se transforma en f (x) = (x − c)q(x) + k y al sustituir x = c, en la igualdad anterior, se obtiene el valor del polinomio en x = c, a saber f (c) = (c − c)q(c) + k = k . Este resultado se escribe de la siguiente forma Teorema 2.3.2 [Teorema del Residuo]. Si un polinomio f (x) se divide entre un polinomio lineal x − c, el residuo r es el valor de f (x) en x = c, es decir, f (c) = r. Ejemplo la divisi´on

2.3.13 Aplicar el teorema del residuo para obtener el residuo de 4x3 + 5x2 − 10 . x−2

Soluci´ on. Sea f (x) = 4x3 + 5x2 − 10. De acuerdo con el teorema del residuo, el residuo r es el valor del polinomio f (x) en x = 2. As´ı que r = f (2) = 4(23 ) + 5(22 ) − 10 = 42.  En realidad el problema inverso es de m´as utilidad, es decir calcular el valor en el punto x = c del polinomio f (x), obteniendo el residuo de la divisi´on de f (x) entre x − c. El m´ etodo de divisi´ on sint´ etica simplifica considerablemente el trabajo para efectuar dicha divisi´on. A continuaci´on se presenta el algoritmo para realizar esta divisi´on sint´etica y los ejemplos posteriores ilustran el m´etodo.

74

Cap´ıtulo 2.

´ Algebra

Algoritmo de la divisi´on sint´etica para calcular: an xn + an−1 xn−1 + · · · + a1 x + a0 entre x − c . a) Se escribe el siguiente arreglo y se pone cero en el coeficiente de cualquier potencia faltante del polinomio c⌋ an an−1 an−2 · · ·

a1 a0

an b) Se multiplica an por c y el producto can y se anota abajo de an−1 , justo encima de la l´ınea horizontal. Luego se calcula la suma b1 = an−1 + can y se coloca debajo de la l´ınea. c⌋ an an−1 an−2 an−3 · · · can cb1 cb2 · · · an

b1

b2

b3 · · ·

a1

a0

cbn−2 cbn−1 bn−1

r

c) Se multiplica b1 por c y el producto cb1 se escribe abajo de an−2 . Despu´es se obtiene la suma b2 = an−2 + cb1 y se anota abajo de la l´ınea. d) Se contin´ ua este proceso hasta obtener la suma final r = a0 + cbn−1 . Los n´ umeros an , b1 , b2 , · · · bn−2 , bn−1 son los coeficientes del cociente q(x); es decir q(x) = an xn−1 + b1 xn−2 + · · · + bn−2 x + bn−1 y r es el residuo. Ejemplo

2.3.14 Aplicar el teorema del residuo para determinar f (−2) y el cociente de f (x) = x5 + 2x3 − 8x − 12

entre

x+2 .

2.3. Operaciones algebraicas

75

Soluci´ on. En este caso se aplica la divisi´on sint´etica. −2⌋

0 2 0 −8 −12 −2 4 −12 24 −32

1

1 −2 6 −12

16 −44

En la u ´ltima fila los primeros cinco n´ umeros corresponden a los coeficientes de las diversas potencias del cociente q(x) y el n´ umero final es el residuo, que se obtiene al dividir f (x) entre x − (−2) = x + 2. Por consiguiente f (−2) = −44 y q(x) = x4 − 2x3 + 6x2 − 12x + 16 . Ejemplo



2.3.15 Aplicar el teorema del residuo para determinar f (c) si

1. f (x) = −2x6 + 21x4 − 10x2 − 30x y c = 3. 2. f (x) = x3 − x2 − 6x − 24 y c = 4. Soluciones. 1. Se usa divisi´on sint´etica. 3⌋ −2

0 21 0 −10 −30 −6 −18 9 27 51

−2 −6

3

9

17

21

Luego f (3) = 63. 2. Nuevamente por divisi´on sint´etica se tiene 4⌋

1 −1 −6 −24 4 12 24 1

3

6

0

0 63 63

76

Cap´ıtulo 2.

´ Algebra

Por lo tanto f (4) = 0 y x = 4 es una ra´ız de la ecuaci´on x3 − x2 − 6x − 24 = 0, es decir, al sustituir x = 4 en la anterior igualdad se tiene 43 − 42 − 6(4) − 24 = 0 . Como el residuo es cero, se tiene en particular la factorizaci´on x3 − x2 − 6x − 24 = (x − 4)(x2 + 3x + 6).  Ejercicio 2.3.1 Efectuar la operaci´on indicada y simplificar su respuesta a la m´ınima expresi´on. 1. (20x − 11y + 13) + (9y − 4x − 2) 2. (5x2 − 4x − 7) − (9 − 6x − 8x2 ) 3. (2b3 − 9b2 + 6b − 18) − (23 + 11b − 15b2 − 8b3 ) √ √ √ √ 4. (7 z − 2 w) + (9 z + 3 w) √ √ √ √ 5. (6 b + 16a) − ( 64a − 9 b) 6. (10xy 2 − 3xy + 9y 3 ) + (4x3 − 11xy − 5x2 y + 21y 3 + 6xy 2 ) √ √ 7. (7 ab + 12) − (3 − 9ab) 8. (x3 − 3)(2x2 + 7) 9. (5x3 + 8y)(4y 3 − 2y 2 + 3) ( ) 7 2 10. − 5x (9x − 2x4 ) x ( )( ) 6y x 2 11. 4x y − 12xy − x 3y 12. 3x2 + 4(y 2 − 6z) − 3(5x − 8y + 11z)

2.3. Operaciones algebraicas

13. 3x − 2 − 4[9 − 7(6x − 4y)] 14. 6{a2 − 4[a + 7(11 − 9a)]} − (1 − 3a) 15. 6x2 − {3x2 − 7[2y − 5(2x2 − 4y)] + 4} 16. (x + 2)(x2 − 6x + 5) 17. (x + 1)(x + 3)(x − 1) 18. (4x2 y 3 )(−3xy 5 − 2x2 y 3 ) 19. (6a2 b)(−3ac + 2ab2 )(3b2 + 7) 20. (x + 2y)(x2 − 5xy + y 2 ) − (x + y)2 (x − 3y) 21. 2a2 b[a3 − (6ab − 3b3 )] − 3ab[a4 − (4a2 b + 9ab3 ) − 1] 22.

3x6 y 2 z 5 −12x4 y 2 z

23.

−24p3 q 2 r 2p5 q

24.

4ab3 − 5ab2 c + 10a3 b2 2a3 bc

25.

7x4 − 9x3 3x5

26.

8x5 − 6x3 + 12x2 + 10 2x2

w2 − 13w + 1 √ 27. w 28.

t4 + 16t3 − 8t + 4 √ 4t2 t

9xy 2 − 27x2 y x2 y 3 + 4x3 y 2 29. + 3xy 2x2 y 2

77

78

Cap´ıtulo 2.

30.

´ Algebra

8y 4 − 4x2 y 2 24y 3 − 16x2 y − 2xy 3 4xy 2 En los ejercicios 31 a 40 simplificar por medio de la divisi´on larga.

31. (x2 − 6x + 8) ÷ (x − 4) 32. (16x2 − 14x + 3) ÷ (2x − 1) 33. (x2 + 4) ÷ (x − 2) 34. (t3 + 1) ÷ (t + 1) 35. (2z 3 + z 5 − 3z − 2) ÷ (z 2 − 3z + 1) 36. (2t3 − 3t2 + 4t + 6) ÷ (2t + 1) 37. (6x3 + 11x2 − 19x + 5) ÷ (3x − 2) 38. (27x3 + x − 2) ÷ (3x2 − x) 39. (6x5 + 4x4 + x3 ) ÷ (x3 − 2) 40. (5x6 − x5 + 10x4 + 3x2 − 2x + 4) ÷ (x2 + x − 1) En los problemas 41 a 45, utilizar la divisi´on sint´etica para calcular el cociente y el residuo de la divisi´on de f (x) entre el polinomio lineal indicado. x−

41. f (x) = 16x2 + 12x + 2; 42. f (x) = x3 − 2x2 + 5; 43. f (x) = x4 + 81;

x+1

x−3

44. f (x) = x5 + 56x2 − 4; 45. f (x) = x6 − 64;

1 4

x+4

x−2

En los ejercicios 46 a 49 usar la divisi´on sint´etica y el teorema del residuo para calcular f (c), para el valor de c dado. 46. f (x) = 3x2 − 7x + 6; 47. f (x) = 2x4 − 5x2 + 19;

c = −4 c=

1 2

2.3. Operaciones algebraicas

79

48. f (x) = x6 + 2x5 − 3x4 − 4x + 6;

c = −3

49. f (x) = 2x7 − 4x5 + 2x3 − x − 12;

c=6

Soluciones. 1. 16x − 2y + 11 2. 13x2 + 2x − 16 3. 10b3 + 6b2 − 5b − 41 √ √ 4. 16 z + w √ √ 5. −4 a + 15 b 6. 16xy 2 − 14xy + 30y 3 + 4x3 − 5x2 y √ 7. 10 ab + 9 8. 2x5 + 7x3 − 6x2 − 21 9. 20x3 y 3 − 10x3 y 2 + 15x3 + 32y 4 − 16y 3 + 24y 10. 63 − 59x3 + 10x6 4 11. 48x3 y 2 − x3 − 72y 2 + 2 3 12. 3x2 + 4y 2 − 15x + 24y − 57z 13. 171x − 112y − 38 14. 6a2 + 1491a − 1849 15. −67x2 + 154y − 4 16. x3 − 4x2 − 7x + 10 17. x3 + 3x2 − x − 3 18. −12x3 y 8 − 8x4 y 6 19. −54a3 b3 c + 84a3 b3 − 126a3 bc + 36a3 b5

80

Cap´ıtulo 2.

20. −4xy 2 + 5y 3 − 2x2 y 21. −a5 b + 33a2 b4 + 3ab 22. −

x2 z 4 4

12qr p2 ( 2 ) ( ) ( ) b 5 b b + 24. 2 − 5 a2 c 2 a2 c

23. −

25.

7 3 − 2 3x x

26. 4x3 − 3x + 6 +

5 x2



√ 1 w3 − 13 w + √ w √ √ t3 2 1 28. + 4 t − √ − 2√ 4 t3 t t 27.

29. −7x + 30.

7y 2

2x 2y − y x

31. x − 2 32. 8x − 3 33. x + 2 +

8 x−2

34. t2 − t + 1 35. z 3 + 3z 2 + 10z + 27 + 36. t2 − 2t + 3 +

3 2t + 1

68z − 29 − 3z + 1

z2

´ Algebra

2.3. Operaciones algebraicas

37. 2x2 + 5x − 3 − 38. 9x + 3 +

81

1 3x − 2

4x − 2 3x2 − x

39. 6x2 + 4x + 1 +

12x2 + 8x + 2 x3 − 2

40. 5x4 − 6x3 + 21x2 − 27x + 51 + 41. f (x) = 16x + 16 +

6 x−

42. f (x) = x2 − 3x + 3 +

51 − 80x x2 + x − 1

1 4

2 x+1

43. f (x) = x3 + 3x2 + 9x + 27 +

162 x−3

44. f (x) = x4 − 4x3 + 16x2 − 8x + 32 −

132 x−4

45. f (x) = x5 + 2x4 + 4x3 + 8x2 + 16x + 32

82

Cap´ıtulo 2.

2.4.

´ Algebra

Productos notables

En la multiplicaci´on de expresiones algebraicas hay algunas que por su utilidad se denominan productos notables. Estos productos son importantes, tanto al desarrollar una expresi´on algebraica como en el sentido opuesto, es decir, escribir mediante un producto de factores a una expresi´on algebraica dada. Los productos notables a tratar son: producto de binomios conjugados, producto de dos binomios con un t´ermino com´ un, cuadrado de un binomio, cubo de un binomio y la n-´esima potencia de un binomio.

2.4.1.

Producto de binomios conjugados

Dos binomios de la forma (a+b) y (a−b) se llaman binomios conjugados. Por ejemplo, son binomios conjugados: (2x + 3m) y (2x − 3m); (5u2 − 4w3 ) y (5u2 + 4w3 ); (−p + 6q) y (6q + p) . Al multiplicar los binomios conjugados (a + b) y (a − b) se obtiene: (a + b)(a − b) = a(a − b) + b(a − b) = a2 − ab + ba − b2 = a2 − ab + ab − b2 = a2 − b2 . Esto es:

(a + b)(a − b) = (a − b)(a + b) = a2 − b2 . Luego, el producto de dos binomios conjugados es igual a una diferencia de cuadrados: el cuadrado del primer t´ermino menos el cuadrado del segundo. Ejemplo

2.4.1 Aplicar la f´ormula del producto de binomios conjugados

2.4. Productos notables

83

para calcular cada producto y simplificar el resultado. 1. (a + 3)(a − 3)

2. (5 − x2 )(5 + x2 )

3. (2x + 3y)(2x − 3y)

4. (x3 − 5x)(x3 + 5x)

( 5.

2 x − + x 2

)(

x 2 + 2 x

) 6. (xn − y m )(xn + y m ) 8. (2 −

7. (−p2 q 3 + p3 q 2 )(p2 q 3 + p3 q 2 ) √ √ √ √ 9. ( x + h − x)( x + h + x)



x − 3)(2 +



x − 3)

10. (a + b + c)(a + b − c)

Soluciones. 1. (a + 3)(a − 3) = a2 − 32 = a2 − 9 . 2. (5 − x2 )(5 + x2 ) = 52 − (x2 )2 = 25 − x4 . 3. (2x + 3y)(2x − 3y) = (2x)2 − (3y)2 = 4x2 − 9y 2 . 4. (x3 −5x)(x3 +5x) = (x3 )2 −(5x)2 = x6 −25x2 . 5. Primero se permutan los sumandos del primer factor, ´esto es: ( )( ) ( )( ) 2 x x 2 x 2 x 2 − + + = − + x 2 2 x 2 x 2 x ( x ) 2 ( 2 )2 x 2 4 = − = − 2 . 2 x 4 x 6. (xn − y m )(xn + y m ) = (xn )2 − (y m )2 = x2n − y 2m . 7. Se permutan los sumandos del primer factor, as´ı: (−p2 q 3 + p3 q 2 )(p2 q 3 + p3 q 2 ) = (p3 q 2 − p2 q 3 )(p3 q 2 + p2 q 3 ) = (p3 q 2 )2 − (p2 q 3 )2 = p6 q 4 − p4 q 6 . 8. (2 −



x − 3)(2 +



√ x − 3) = 22 − ( x − 3)2 = 4 − (x − 3) = 4−x+3=7−x .

84

Cap´ıtulo 2.

´ Algebra

√ √ √ √ √ √ 9. ( x + h − x)( x + h + x) = ( x + h)2 − ( x)2 = (x + h) − x = h . 10. Primeramente se agrupa convenientemente (a + b + c)(a + b − c) = [(a + b) + c][(a + b) − c] = (a + b)2 − c2 = (a2 + 2ab + b2 ) − c2 = a2 + b2 − c2 + 2ab .  Ejercicio 2.4.1 Aplicar la f´ormula del producto de binomios conjugados para calcular cada multiplicaci´on y simplificar el resultado. 1. (a + 1)(a − 1)

2. (8 − b)(8 + b)

3. (3x + 4)(3x − 4)

√ √ √ √ 4. ( x + 1 − x)( x + 1 + x)

√ √ 5. ( x2 + 1 + x)( x2 + 1 − x)

6. (−3x2 − 2y 3 )(−3x2 + 2y 3 )

7. (1 − p2 )(1 + p2 )

√ √ √ √ 8. (2 x + 3 y)(2 x − 3 y)

9. (x2 − 2x − 3)(x2 + 2x + 3)

10. (5x3 y 4 − 6x4 y 3 )(5x3 y 4 + 6x4 y 3 ) (

11. (3x + 2x − 1)(3x − 2x + 1) 2

2

12.

2x3 3 − 3 3 2x

)(

2x3 3 + 3 3 2x

13. (xn/2 y n + xn y n/2 )(−xn/2 y n + xn y n/2 ) √ √ √ √ 14. ( x2 − 2x + 6 − x2 + 2x − 6)( x2 − 2x + 6 + x2 + 2x − 6) √ √ 15. ( 9x + 19 − 6x − 1)( 9x + 19 + 6x + 1) Soluciones. 1. a2 − 1

2. 64 − b2

3. 9x2 − 16

4. 1

5. 1

6. 9x4 − 4y 6

7. 1 − p4

8. 4x − 9y

9. x4 − 4x2 − 12x − 9

10. 25x6 y 8 − 36x8 y 6

)

2.4. Productos notables

85

9 4x6 − 6 9 4x

11. 9x4 − 4x2 + 4x − 1

12.

13. x2n y n − xn y 2n

14. 12 − 4x

15. 18 − 3x − 36x2

2.4.2.

Producto de dos binomios con un t´ ermino com´ un

Dos binomios de la forma (x + m) y (x + n) tienen el t´ermino com´ un x. Se considera aqu´ı el caso m diferente de n; el caso de igualdad, que corresponde al cuadrado de un binomio, se trata en la secci´on siguiente. Por ejemplo, son binomios de esta forma: (x + 5) y (x − 3); (p2 − 8) y (p2 − 3); (3a + 1) y (3a − 2) . Al multiplicar los binomios (x + m) y (x + n) se obtiene: (x + m)(x + n) = x(x + n) + m(x + n) = x2 + xn + mx + mn = x2 + mx + nx + mn = x2 + (m + n)x + mn . Esto es:

(x + m)(x + n) = x2 + (m + n)x + mn . As´ı, el producto de dos binomios que tienen un t´ermino com´ un es igual a: el cuadrado del t´ermino com´ un, m´as la suma de los t´erminos diferentes por el t´ermino com´ un, m´as el producto de los t´erminos diferentes. Ejemplo

2.4.2 Aplicar la f´ormula del producto de binomios con un t´ermino com´ un para calcular cada multiplicaci´on y simplificar el resultado. 1. (x + 5)(x + 3)

2. (x + 2)(x − 4)

3. (x − 6)(x + 10)

4. (x − 2)(x − 1)

5. (a2 + 8)(a2 − 5)

6. (p3 − 1)(p3 − 3)

7. (2m − 5)(2m + 3) 9. (−3y + x)(2y + x)

8. (9 + xn )(xn − 8) 10. (xn − 3y m )(−2y m + xn )

86

Cap´ıtulo 2.

´ Algebra

Soluciones. 1. (x + 5)(x + 3) = x2 + (5 + 3)x + (5)(3) = x2 + 8x + 15 . 2. (x+2)(x−4) = x2 +(2−4)x+(2)(−4) = x2 +(−2)x+(−8) = x2 −2x−8 . 3. (x − 6)(x + 10) = x2 + (−6 + 10)x + (−6)(10) = x2 + (4)x + (−60) = x2 + 4x − 60 . 4. (x − 2)(x − 1) = x2 + (−2 − 1)x + (−2)(−1) = x2 + (−3)x + (2) = x2 − 3x + 2 . 5. (a2 + 8)(a2 − 5) = (a2 )2 + (8 − 5)a2 + (8)(−5) = a4 + (3)a2 + (−40) = a4 + 3a2 − 40 . 6. (p3 − 1)(p3 − 3) = (p3 )2 + (−1 − 3)p3 + (−1)(−3) = p6 + (−4)p3 + (3) = p6 − 4p3 + 3 . 7. (2m − 5)(2m + 3) = (2m)2 + (−5 + 3)(2m) + (−5)(3) = 4m2 + (−2)(2m) + (−15) = 4m2 − 4m − 15 . 8. Reordenando el primer factor se tiene: (9 + xn )(xn − 8) = (xn + 9)(xn − 8) = (xn )2 + (9 − 8)xn + (9)(−8) = x2n + (1)xn + (−72) = x2n + xn − 72 . 9. Primeramente se reacomodan los dos factores, as´ı (−3y + x)(2y + x) = (x − 3y)(x + 2y) = x2 + (−3y + 2y)x + (−3y)(2y) = x2 + (−y)x + (−6y 2 ) = x2 − xy − 6y 2 .

2.4. Productos notables

87

10. Se reescribe el segundo factor (xn − 3y m )(−2y m + xn ) = (xn − 3y m )(xn − 2y m ) = (xn )2 + (−3y m − 2y m )xn + (−3y m )(−2y m ) = x2n + (−5y m )xn + (6y 2m ) = x2n − 5xn y m + 6y 2m .



Ejercicio 2.4.2 Usando la f´ormula para el producto de binomios con un t´ermino en com´ un calcular cada producto. 1. (x + 8)(x − 2)

2. (x − 5)(x + 4)

3. (x − 9)(x − 7)

4. (a + 1)(a − 6)

5. (m + 10)(m + 9)

6. (x2 − 3)(x2 + 1)

7. (c3 + 2)(c3 − 5)

8. (am − 6)(am + 7)

9. (x − 3y)(x − 5y)

10. (x + 2y)(9y + x)

11. (a2 − 1)(a2 − 2)

12. (a2 − b2 )(a2 − 2b2 )

13. (xn + 11)(xn + 1)

14. (3x − 4)(5 + 3x) 15. (xn + 7y n )(−5y n + xn ) Soluciones. 1. x2 + 6x − 16

2. x2 − x − 20

3. x2 − 16x + 63

4. a2 − 5a − 6

5. m2 + 19m + 90

6. x4 − 2x2 − 3

7. c6 − 3c3 − 10

8. a2m + am − 42

9. x2 − 8xy + 15y 2

10. x2 + 11xy + 18y 2

11. a4 − 3a2 + 2

12. a4 − 3a2 b2 + 2b4

13. x2n + 12xn + 11

14. 9x2 + 3x − 20

15. x2n + 2xn y n − 35y 2n

88

Cap´ıtulo 2.

2.4.3.

´ Algebra

Cuadrado de un binomio

Cuando se eleva al cuadrado la suma o diferencia de dos t´erminos se tiene el cuadrado de un binomio. Al desarrollar el cuadrado de una suma de t´erminos se obtiene: (a + b)2 = (a + b)(a + b) = a(a + b) + b(a + b) = a2 + ab + ba + b2 = a2 + ab + ab + b2 = a2 + 2ab + b2 . Esto es:

(a + b)2 = a2 + 2ab + b2 .

Es decir, el cuadrado de una suma de dos t´erminos, es igual al cuadrado del primer t´ermino, m´as el doble producto de los dos t´erminos, m´as el cuadrado del segundo t´ermino Al desarrollar el cuadrado de una diferencia de dos t´erminos se obtiene: (a − b)2 = (a − b)(a − b) = a(a − b) − b(a − b) = a2 − ab − ba + b2 = a2 − ab − ab + b2 = a2 − 2ab + b2 . Por lo tanto:

(a − b)2 = a2 − 2ab + b2 .

As´ı, el cuadrado de una diferencia de dos t´erminos, es igual al cuadrado del primer t´ermino, menos el doble producto de los dos t´erminos, m´as el cuadrado del segundo t´ermino. Cada uno de los trinomios obtenidos: a2 + 2ab + b2 o bien a2 − 2ab + b2 es denominado trinomio cuadrado perfecto.

2.4. Productos notables

89

Ejemplo

2.4.3 Utilizando las f´ormulas del cuadrado de un binomio, obtener el desarrollo de cada expresi´on. 1. (a + 5)2 4.

(x 3

+

y )2 2

7. (a2 − b2 )2

2. (3 − b)2

3. (2x − 3y)2

5. (x2 − 1)2

6. (x3 + 2)2

8. (p2 + 2p)2

9. (xn − y m )2

10. (3x4 y 3 − 2x3 y 4 )2

11. (axn+1 + bxn )2

13. (x − y − z)2

14. [(a − b) − (x − y)]2

12. [x + (y − z)]2

15. (−a + b − c + d)2 Soluciones. 1. (a + 5)2 = a2 + 2a(5) + (5)2 = a2 + 10a + 25 . 2. (3 − b)2 = 32 − 2(3)b + b2 = 9 − 6b + b2 . 3. (2x − 3y)2 = (2x)2 − 2(2x)(3y) + (3y)2 = 4x2 − 12xy + 9y 2 . 4. (x

( x )2 ( x ) ( y ) ( y )2 x2 xy y 2 y )2 = +2 + = + + + 3 2 3 3 2 2 9 3 4 1 2 1 1 2 = x + xy + y . 9 3 4

5. (x2 − 1)2 = (x2 )2 − 2(x2 )(1) + (1)2 = x4 − 2x2 + 1 . 6. (x3 + 2)2 = (x3 )2 + 2(x3 )(2) + 22 = x6 + 4x3 + 4 . 7. (a2 − b2 )2 = (a2 )2 − 2a2 b2 + (b2 )2 = a4 − 2a2 b2 + b4 . 8. (p2 + 2p)2 = (p2 )2 + 2p2 (2p) + (2p)2 = p4 + 4p3 + 4p2 . 9. (xn − y m )2 = (xn )2 − 2xn y m + (y m )2 = x2n − 2xn y m + y 2m .

90

Cap´ıtulo 2.

´ Algebra

10. (3x4 y 3 − 2x3 y 4 )2 = (3x4 y 3 )2 − 2(3x4 y 3 )(2x3 y 4 ) + (2x3 y 4 )2 = 9x8 y 6 − 12x7 y 7 + 4x6 y 8 . 11. (axn+1 + bxn )2 = (axn+1 )2 + 2(axn+1 )(bxn ) + (bxn )2 = a2 x2(n+1) + 2abx(n+1)+n + b2 x2n = a2 x2n+2 + 2abx2n+1 + b2 x2n . 12. En este caso, no se elimina el par´entesis interior y se desarrolla [x + (y − z)]2 = x2 + 2x(y − z) + (y − z)2 = x2 + 2xy − 2xz + (y 2 − 2yz + z 2 ) = x2 + y 2 + z 2 + 2xy − 2xz − 2yz . 13. Primero se asocia convenientemente [x − y − z]2 = [x − (y + z)]2 = x2 − 2x(y + z) + (y + z)2 = x2 − 2xy − 2xz + (y 2 + 2yz + z 2 ) = x2 + y 2 + z 2 − 2xy − 2xz + 2yz . 14. Aqu´ı no se eliminan los par´entesis interiores y se desarrolla [(a + b)−(x − y)]2 = (a + b)2 − 2(a + b)(x − y) + (x − y)2 = (a2 + 2ab + b2 ) − 2(ax − ay + bx − by) + (x2 − 2xy + y 2 ) = a2 + 2ab + b2 − 2ax + 2ay − 2bx + 2by + x2 − 2xy + y 2 = a2 + b2 + x2 + y 2 + 2ab − 2ax + 2ay − 2bx + 2by − 2xy . En los ejemplos 12, 13 y 14 se observa que el cuadrado de un multinomio es igual a la suma de los cuadrados de cada t´ermino, m´as la suma de todos los dobles productos, cuidando los signos. 15. Aplicando la observaci´on anterior (−a + b − c + d)2 = a2 + b2 + c2 + d2 − 2ab + 2ac − 2ad − 2bc + 2bd − 2cd .



2.4. Productos notables

91

Ejercicio 2.4.3 Utilizando las f´ormulas del cuadrado de un binomio, obtener el desarrollo de cada expresi´on. ( )2 1 2 3. (3x − 2y)2 1. (a + 3) 2. x− 2 4. (4x + 5y)2 ( 7.

x 2 + 2 x

6. (1 − x3 )2

5. (x2 + 1)2 (

)2 8.

3m 2n + 2 3

)2 9. (x4 − 3)2

10. (4 − x2 )2

√ √ 11. ( x + y)2

√ √ 12. (2 x − 3 y)2

13. (3x2 + 4x3 )2

14. (5x3 − 6x5 )2

15. (pn − 1)2

16. (1 + q r )2

17. (1 − 3x5 y 6 z 3 )2

18. (5a4 b2 c3 + 1)2

19. (x3 y 5 − x2 y 4 )2

20. (x6 y 8 + x9 y 7 )2

21. (pn − q m )2

22. (pn + pm )2

23. (xn+1 − xn−1 )2

24. (y 2m + y 2m−1 )2

25. (x + y + z)2

26. (a − b + c)2

27. (x2 − 2x + 1)2

28. (1 + 2x − x2 )2

29. (xn y m − xm y n )2

30. (axn+1 y n−1 + bxn−1 y n+1 )2 Soluciones. 1 4

1. a2 + 6a + 9

2. x2 − x +

3. 9x2 − 12xy + 4y 2

4. 16x2 + 40xy + 25y 2

5. x4 + 2x2 + 1

6. x6 − 2x3 + 1

7.

4 x2 +2+ 2 4 x

9. x8 − 6x4 + 9

8.

4 9 2 m + 2mn + n2 4 9

10. x4 − 8x2 + 16

92

Cap´ıtulo 2.

√ 11. x + 2 xy + y

√ 12. 4x − 12 xy + 9y

13. 9x4 + 24x5 + 16x6

14. 25x6 − 60x8 + 36x10

15. p2n − 2pn + 1

16. q 2r + 2q r + 1

´ Algebra

17. 1 − 6x5 y 6 z 3 + 9x10 y 12 z 6

18. 25a8 b4 c6 + 10a4 b2 c3 + 1

19. x6 y 10 − 2x5 y 9 + x4 y 8

20. x12 y 16 + 2x15 y 15 + x18 y 14

21. p2n − 2pn q m + q 2m

22. p2n + 2pn+m + p2m

23. x2n+2 − 2x2n + x2n−2

24. y 4m + 2y 4m−1 + y 4m−2

25. x2 + y 2 + z 2 + 2xy + 2xz + 2yz

26. a2 + b2 + c2 − 2ab + 2ac − 2bc

27. x4 − 4x3 + 6x2 − 4x + 1

28. x4 − 4x3 + 2x2 + 4x + 1

29. x2n y 2m − 2xn+m y n+m + x2m y 2n 30. a2 x2n+2 y 2n−2 + 2abx2n y 2n + b2 x2n−2 y 2n+2 .

2.4.4.

Cubo de un binomio

Cuando se eleva al cubo la suma o diferencia de dos t´erminos, se tiene el cubo de un binomio. Al desarrollar el cubo de una suma se obtiene: (a + b)3 = (a + b)(a + b)2 = a(a + b)2 + b(a + b)2 = a(a2 + 2ab + b2 ) + b(a2 + 2ab + b2 ) = a3 + 2a2 b + ab2 + a2 b + 2ab2 + b3 = a3 + 3a2 b + 3ab2 + b3 . Esto es:

2.4. Productos notables

93

(a + b)3 = a3 + 3a2 b + 3ab2 + b3 . Es decir, el cubo de una suma de dos t´ erminos, es igual a: el cubo del primer t´ermino, m´as el triple producto del cuadrado del primer t´ermino por el segundo, m´as el triple producto del primer t´ermino por el cuadrado del segundo, m´as el cubo del segundo t´ermino. Al calcular el cubo de una diferencia se obtiene: (a − b)3 = (a − b)(a − b)2 = a(a − b)2 − b(a − b)2 = a(a2 − 2ab + b2 ) − b(a2 − 2ab + b2 ) = a3 − 2a2 b + ab2 − a2 b + 2ab2 − b3 = a3 − 3a2 b + 3ab2 − b3 . Luego:

(a − b)3 = a3 − 3a2 b + 3ab2 − b3 . Es decir, el cubo de una diferencia de dos t´ erminos, es igual a: el cubo del primer t´ermino, menos el triple producto del cuadrado del primer t´ermino por el segundo, m´as el triple producto del primer t´ermino por el cuadrado del segundo, menos el cubo del segundo t´ermino. Ejemplo

2.4.4 Mediante las f´ormulas del cubo de un binomio, desarrollar las expresiones siguientes. 2. (5 − b)3

1. (a + 2)3 ( 4.

3x 2y − 2 3

)3

7. (x2 y 3 − x3 y 2 )3 10. (x2 + 2x)3

3. (2x + 3y)3 (

1 x+ x

)3

5. (x − 1)

6.

8. (an + bn )3

√ √ 9. ( 3 x − 3 y)3

2

3

94

Cap´ıtulo 2.

´ Algebra

Soluciones. 1. (a + 2)3 = a3 + 3a2 (2) + 3a(2)2 + (2)3 = a3 + 6a2 + 12a + 8 . 2. (5 − b)3 = 53 − 3(5)2 b + 3(5)b2 − b3 = 125 − 75b + 15b2 − b3 . 3. (2x + 3y)3 = (2x)3 + 3(2x)2 (3y) + 3(2x)(3y)2 + (3y)3 = 8x3 + 3(4x2 )(3y) + 3(2x)(9y 2 ) + 27y 3 = 8x3 + 36x2 y + 54xy 2 + 27y 3 . 4. (

3x 2y − 2 3

)3

( ) ( )2 )3 ( )2 ( ) 3x 3x 3x 2y 2y = −3 +3 2 2 3 2 3 ( )3 2y − 3 ( )( 2) ( 2)( ) 3 3x 9x 27x 2y 4y 8y 3 = −3 +3 − 8 4 3 2 9 27 27 3 9 2 8 = x − x y + 2xy 2 − y 3 . 8 2 27 (

5. (x2 − 1)3 = (x2 )3 − 3(x2 )2 (1) + 3(x2 )(1)2 − (1)3 = x6 − 3x4 + 3x2 − 1 . ( )3 ( )2 ( )3 ( ) 1 1 1 1 3 1 3 2 6. x + = x + 3x + 3x + = x3 + 3x + + 3 . x x x x x x 7. (x2 y 3 − x3 y 2 )3 = (x2 y 3 )3 − 3(x2 y 3 )2 (x3 y 2 ) + 3(x2 y 3 )(x3 y 2 )2 − (x3 y 2 )3 = x6 y 9 − 3(x4 y 6 )(x3 y 2 ) + 3(x2 y 3 )(x6 y 4 ) − x9 y 6 = x6 y 9 − 3x7 y 8 + 3x8 y 7 − x9 y 6 . 8. (an + bn )3 = (an )3 + 3(an )2 (bn ) + 3(an )(bn )2 + (bn )3 = a3n + 3a2n bn + 3an b2n + b3n .

2.4. Productos notables

95

9. √ √ √ √ √ √ √ √ ( 3 x − 3 y)3 = ( 3 x)3 − 3( 3 x)2 ( 3 y) + 3( 3 x)( 3 y)2 − ( 3 y)3 √ √ √ √ 3 = x − 3 x2 3 y + 3 3 x 3 y 2 − y √ √ = x − 3 3 x2 y + 3 3 xy 2 − y . 10. (x2 + 2x)3 = (x2 )3 + 3(x2 )2 (2x) + 3(x2 )(2x)2 + (2x)3 = x6 + 6x5 + 12x4 + 8x3 .



Ejercicio 2.4.4 Mediante las f´ormulas del cubo de un binomio, desarrollar las expresiones siguientes. ( )3 x 3 3 3 1. (a − 1) 2. (x + 2) 3. − 4. (1 − b)3 3 x 5. (x2 + 3)3 9. (an + xn )3

6. (4 − x3 )3

7. (3x2 − 2x)3

8. (xn − 1)3

10. (xn/3 + y n/3 )3

Soluciones. 1. a3 − 3a2 + 3a − 1 3.

2. x3 + 6x2 + 12x + 8

x3 9 27 −x+ − 3 27 x x

4. 1 − 3b + 3b2 − b3

5. x6 + 9x4 + 27x2 + 27

6. 64 − 48x3 + 12x6 − x9

7. 27x6 − 54x5 + 36x4 − 8x3

8. x3n − 3x2n + 3xn − 1

9. a3n + 3a2n xn + 3an x2n + x3n

2.4.5.

√ √ 10. xn + 3 3 x2n y n + 3 3 xn y 2n + y n

Tri´ angulo de Pascal

El tri´angulo de Pascal no es un producto notable, pero est´a relacionado con los dos productos anteriores. Al calcular (a ± b)n para n = 0, 1, 2, 3 y 4

96

Cap´ıtulo 2.

´ Algebra

se tiene que (a ± b)0 (a ± b)1 (a ± b)2 (a ± b)3 (a ± b)4

=1 =a±b = a2 ± 2ab + b2 = a3 ± 3a2 b + 3ab2 ± b3 = a4 ± 4a3 b + 6a2 b2 ± 4ab3 + b4 .

Estos desarrollos se pueden expresar de la siguiente manera (a ± b)0 (a ± b)1 (a ± b)2 (a ± b)3 (a ± b)4

=1 = 1a1 b0 ± 1a0 b1 = 1a2 b0 ± 2a1 b1 + 1a0 b2 = 1a3 b0 ± 3a2 b1 + 3a1 b2 ± 1a0 b3 = 1a4 b0 ± 4a3 b1 + 6a2 b2 ± 4a1 b3 + 1a0 b4 .

Observando estos desarrollos se infiere, para el desarrollo de (a ± b)n , lo siguiente:

el n´ umero de t´erminos es n + 1 y todos de grado n; el primer t´ermino es an b0 y el u ´ltimo a0 bn , ambos con coeficiente 1;

de t´ermino a t´ermino, el exponente de a se reduce en 1; mientras que el exponente de b aumenta en 1, seg´ un la regla an−k bk para k = 0, 1, 2, . . . , n. pues (a ± b)n = (a ± b)n−1 (a ± b). Para los coeficientes de los t´erminos intermedios se obtiene, para (a + b)n , el siguiente arreglo triangular de n´ umeros.

2.4. Productos notables

(a + b)0 : (a + b)1 :

1 1

(a + b)2 :

1

(a + b)3 :

1

(a + b)4 :

1

(a + b)5 :

1

(a + b)6 :

1

97

+

−→ 1 ↓ + −→ 2 ↓ + −→ 3 ↓ + −→ 4 ↓ + −→ 5 ↓ 6

+

−→ 1 ↓ + + −→ 3 −→ 1 ↓ ↓ + + + −→ 6 −→ 4 −→ 1 ↓ ↓ ↓ + + + + −→ 10 −→ 10 −→ 5 −→ 1 ↓ ↓ ↓ ↓ 15 20 15 6

1

Se observa en el arreglo, que cada rengl´on se obtiene del rengl´on anterior, siguiendo la operatividad indicada por las flechas entre dos t´erminos horizontales consecutivos. A este arreglo triangular de n´ umeros se le denomina tri´ angulo de Pascal. Para (a − b)n , el tri´angulo de Pascal inicia siempre con el coeficiente +1 y el signo − se va alternando con el signo + hasta finalizar. Si n es par el u ´ltimo coeficiente es 1 y si es impar es −1. Se recomienda emplear el tri´angulo de Pascal cuando el exponente n no es grande. Otra manera de visualizar el tri´angulo de Pascal es la siguiente:

1 1 1 1 1

@ R

@ R

4

@ R

3

2



@ R

@ R

6

1 3

1

@ R

4

1 1

98

Cap´ıtulo 2.

Ejemplo

´ Algebra

2.4.5 Mediante el tri´angulo de Pascal, obtener el desarrollo de:

1. (2x + y)5

2. (x − 3y)5

5. (3x + 2y)4

6. (2x − 3y)4

3. (x2 + y 2 )6

4. (a3 − b3 )6

Soluciones. 1. (2x + y)5 = 1(2x)5 y 0 + 5(2x)4 y + 10(2x)3 y 2 + 10(2x)2 y 3 + 5(2x)y 4 +1(2x)0 y 5 = (25 x5 ) + 5(24 x4 )y + 10(23 x3 )y 2 + 10(22 x2 )y 3 + 5(2x)y 4 + y5 = 32x5 + 80x4 y + 80x3 y 2 + 40x2 y 3 + 10xy 4 + y 5 . 2. (x − 3y)5 = 1(x)5 (3y)0 − 5x4 (3y) + 10x3 (3y)2 − 10x2 (3y)3 + 5x(3y)4 − 1x0 (3y)5 = x5 − 5x4 (3y) + 10x3 (9y 2 ) − 10x2 (27y 3 ) + 5x(81y 4 ) − 243y 5 = x5 − 15x4 y + 90x3 y 2 − 270x2 y 3 + 405xy 4 − 243y 5 . 3. (x2 + y 2 )6 = 1(x2 )6 (y 2 )0 + 6(x2 )5 (y 2 ) + 15(x2 )4 (y 2 )2 + 20(x2 )3 (y 2 )3 + 15(x2 )2 (y 2 )4 + 6(x2 )(y 2 )5 + 1(x2 )0 (y 2 )6 = x12 + 6x10 y 2 + 15x8 y 4 + 20x6 y 6 + 15x4 y 8 + 6x2 y 10 + y 12 . 4. (a3 − b3 )6 = 1(a3 )6 (b3 )0 − 6(a3 )5 (b3 ) + 15(a3 )4 (b3 )2 − 20(a3 )3 (b3 )3 + 15(a3 )2 (b3 )4 − 6(a3 )(b3 )5 + 1(a3 )0 (b3 )6 = a18 − 6a15 b3 + 15a12 b6 − 20a9 b9 + 15a6 b12 − 6a3 b15 + b18 .

2.4. Productos notables

99

5. (3x + 2y)4 = 1(3x)4 (2y)0 + 4(3x)3 (2y) + 6(3x)2 (2y)2 + 4(3x)(2y)3 + 1(3x)0 (2y)4 = 34 x4 + 4(27x3 )(2y) + 6(9x2 )(4y 2 ) + 4(3x)(8y 3 ) + 24 y 4 = 81x4 + 216x3 y + 216x2 y 2 + 96xy 3 + 16y 4 . 6. (2x − 3y)4 = 1(2x)4 (3y)0 − 4(2x)3 (3y) + 6(2x)2 (3y)2 − 4(2x)(3y)3 + 1(2x)0 (3y)4 = 24 x4 − 4(8x3 )(3y) + 6(4x2 )(9y 2 ) − 4(2x)(27y 3 ) + 34 y 4 = 16x4 − 96x3 y + 216x2 y 2 − 216xy 3 + 81y 4  Ejercicio 2.4.5 Mediante el tri´angulo de Pascal, obtener el desarrollo de: 1. (3x − y)5

2. (x + 2y)5

3. (xn − y n )4

5. (a2 − b)7

6. (a + b2 )7

7.

( 9.

2x +

y )6 2

(x 2

)5 +2

4. (x5 + y 5 )4 8. (xn − y n )6

10. (a2 − b2 )8

Soluciones. 1. 243x5 − 405x4 y + 270x3 y 2 − 90x2 y 3 + 15xy 4 − y 5 2. x5 + 10x4 y + 40x3 y 2 + 80x2 y 3 + 80xy 4 + 32y 5 3. x4n − 4x3n y n + 6x2n y 2n − 4xn y 3n + y 4n 4. x20 + 4x15 y 5 + 6x10 y 10 + 4x5 y 15 + y 20 5. a14 − 7a12 b + 21a10 b2 − 35a8 b3 + 35a6 b4 − 21a4 b5 + 7a2 b6 − b7 6. a7 + 7a6 b2 + 21a5 b4 + 35a4 b6 + 35a3 b8 + 21a2 b10 + 7ab12 + b14 7.

1 5 5 4 x + x + 5x3 + 20x2 + 40x + 32 32 8

100

Cap´ıtulo 2.

´ Algebra

8. x6n − 6x5n y n + 15x4n y 2n − 20x3n y 3n + 15x2n y 4n − 6xn y 5n + y 6n 9. 64x6 + 96x5 y + 60x4 y 2 + 20x3 y 3 +

1 15 2 4 3 5 x y + xy + y 6 4 8 64

10. a16 −8a14 b2 +28a12 b4 −56a10 b6 +70a8 b8 −56a6 b10 +28a4 b12 −8a2 b14 +b16

2.4.6.

F´ ormula del binomio

El tri´angulo de Pascal resulta poco pr´actico para calcular el desarrollo de (x + y)n , cuando n es grande. En este caso se usa la f´ ormula del Binomio, la cual es tema de un curso posterior. Por ello, aqu´ı s´olo se induce esta f´ormula al observar las f´ormulas de (a + b)n de la secci´on anterior y tambi´en (a + b)5 = a5 + 5a4 b + 10a3 b2 + 10a2 b3 + 5ab4 + b5 = a5 + 5a5−1 b1 + 10a5−2 b2 + 10a5−3 b3 + 5a5−4 b4 + b5 .

(2.4.2)

En los desarrollos mencionados se nota lo siguiente Son n + 1 t´erminos y todos de grado n. ´ltimo es bn , ambos con coeficiente 1. El primer t´ermino es an y el u ultimo es nabn−1 . El segundo t´ermino es nan−1 b y el pen´ Cada uno de los t´erminos es de la forma αk an−k bk , donde αk es el coeficiente, k = 0, 1, 2, . . . , n. En la f´ormula 2.4.2 se observa: 5 · α0 4·5 4 · α1 ; α2 = 10 = = ; 1 2 2 3 · 10 3 · α2 2 · 10 2 · α3 α3 = 10 = = ; α4 = 5 = = ; 3 3 4 4 1·5 1 · α4 α5 = 1 = = . 5 5

α0 = 1 ; α1 = 5 =

Es decir, el coeficiente de cada t´ermino, excepto el primero, se puede determinar por medio del exponente y el coeficiente del t´ermino anterior. En general, si (a + b)n = an + nan−1 b + · · · + αk an−k bk + αk+1 an−(k+1) bk+1 + · · · + nabn−1 + bn

2.4. Productos notables

101

entonces αk+1 = Ejemplo

(n − k)αk k+1

para k = 0, 1, 2, . . . , (n − 1).

2.4.6 Obtener el desarrollo de: 2. (a − b)10

1. (x + y)10

3. (a2 + 2b)6

Soluciones. 1. (x + y)10 = α0 x10 y 0 + α1 x9 y 1 + α2 x8 y 2 + α3 x7 y 3 + α4 x6 y 4 + α5 x5 y 5 + α6 x4 y 6 + α7 x3 y 7 + α8 x2 y 8 + α9 xy 9 + α10 x0 y 10 , donde α0 = α10 = 1 y α1 = α9 = 10. Adem´as: α2 =

9α1 9(10) = =5; 1+1 2

α4 =

6α4 7α3 7(120) 6(210) = = 210 ; α5 = = = 252; 3+1 4 4+1 5

α6 =

5α5 5(252) 4α6 4(210) = = 210 ; α7 = = = 120; 5+1 6 6+1 7

α8 =

3(120) 3α7 = = 45 . 7+1 8

α3 =

8α2 8(45) = = 120; 2+1 3

Por lo tanto (x + y)10 = x10 + 10x9 y 1 + 45x8 y 2 + 120x7 y 3 + 210x6 y 4 + 252x5 y 5 + 210x4 y 6 + 120x3 y 7 + 45x2 y 8 + 10xy 9 + y 10 . 2. Para este caso se utiliza el desarrollo anterior y se alternan los signos (a − b)10 = a10 − 10a9 b1 + 45a8 b2 − 120a7 b3 + 210a6 b4 − 252a5 b5 + 210a4 b6 − 120a3 b7 + 45a2 b8 − 10ay 9 + b10 .

102

Cap´ıtulo 2.

´ Algebra

3. (a2 + 2b)6 = α0 (a2 )6 (2b)0 + α1 (a2 )5 (2b)1 + α2 (a2 )4 (2b)2 + α3 (a2 )3 (2b)3 + α4 (a2 )2 (2b)4 + α5 (a2 )1 (2b)5 + α6 (a2 )0 (2b)6 donde α0 = α6 = 1 y α1 = α5 = 6 y α2 =

5α1 5(6) = 15 ; = 1+1 2

α4 =

3α3 3(20) = = 15 . 3+1 4

α3 =

4α2 4(15) = 20; = 2+1 3

As´ı (a2 + 2b)6 = a12 + 6(a10 )(2b) + 15(a8 )(4b2 ) + 20(a6 )(8b3 ) + 15(a4 )(16b4 ) + 6(a2 )(32b5 ) + 64b6 = a12 + 12a10 b + 60a8 b2 + 160a6 b3 + 240a4 b4 + 192a2 b5 + 64b6 .  Ejercicio 2.4.6 Obtener el desarrollo de: 2 7

1. (a + b )

2.

(x 2

)5 +2

3. (x − y ) n

n 6

( 4.

y )6 2x + 2

5. (a2 − b2 )8 Soluciones. 1. a7 + 7a6 b2 + 21a5 b4 + 35a4 b6 + 35a3 b8 + 21a2 b10 + 7ab12 + b14 2.

1 5 5 4 x + x + 5x3 + 20x2 + 40x + 32 32 8

3. x6n − 6x5n y n + 15x4n y 2n − 20x3n y 3n + 15x2n y 4n − 6xn y 5n + y 6n 4. 64x6 + 96x5 y + 60x4 y 2 + 20x3 y 3 +

15 2 4 3 5 1 x y + xy + y 6 4 8 64

5. a16 −8a14 b2 +28a12 b4 −56a10 b6 +70a8 b8 −56a6 b10 +28a4 b12 −8a2 b14 +b16

2.5. Factorizaci´ on

2.5.

103

Factorizaci´ on

En el tema anterior, se estudi´o c´omo obtener el desarrollo de ciertos productos, sin efectuar expl´ıcitamente la multiplicaci´on indicada. Se parti´o de un producto de factores conocidos y se vi´o como obtener una expresi´on algebraica equivalente, mediante un desarrollo sencillo. Esto es, se transit´o el camino Factores conocidos → Expresi´on algebraica. En esta secci´on se muestra como recorrer la ruta opuesta: Expresi´on algebraica conocida → Descomposici´on en factores, lo anterior s´olo se hace para algunas expresiones algebraicas. A este proceso se le llama factorizaci´ on y se trata por casos, como ocurri´o con el tema de productos notables. Es u ´til recordar: (i) Si a = b entonces b = a. (ii) Si mn = p, se dice que m y n son factores de p. Se ejemplifican este par de afirmaciones. 1. La igualdad x(x − 2) = x2 − 2x permite ver que la expresi´on algebraica x2 − 2x puede ser escrita como x(x − 2), donde x y (x − 2) son factores de x2 − 2x. 2. Ya que (x+3)(x−5) = x2 −2x−15, se tiene x2 −2x−15 = (x+3)(x−5), donde (x+3) y (x−5) son factores de la expresi´on algebraica x2 −2x−15. A continuaci´on se estudian los casos m´as comunes de factorizaci´on.

2.5.1.

Factor com´ un

Debido a la propiedad distributiva se tiene la identidad x(a + b − c) = xa + xb − xc , al desarrollar el producto del factor x por la suma algebraica indicada. As´ı se obtiene una suma algebraica de productos, donde cada t´ermino contiene el factor x. En consecuencia, se dice que x es un factor com´ un a todos los t´erminos de la expresi´on algebraica xa + xb − xc. As´ı por la propiedad distributiva se tiene la factorizaci´on

104

Cap´ıtulo 2.

´ Algebra

xa + xb − xc = x(a + b − c) . con lo cual se factoriza a la expresi´on algebraica xa + xb − xc, ya que se expresa como un producto de factores, a saber: el factor com´ un x y el factor (a + b − c). N´otese que este otro factor (a + b − c) se obtiene dividiendo a cada t´ermino de la suma algebraica xa + xb − xc, entre su factor com´ un x, es decir xa + xb − xc xa xb xc = + − =a+b−c . x x x x Se observa tambi´en que el factor (a + b − c) es un multinomio que ya no tiene un factor com´ un expl´ıcito. Una expresi´on algebraica puede tener un factor com´ un, varios factores comunes o carecer de factores comunes, por ejemplo 6x2 y − 10x3 z + 14x4 t tiene por factores comunes a: 2, x, x2 , 2x y 2x2 . En el proceso de factorizar, cuando hay varios factores comunes, se debe considerar al factor com´ un mayor , el cual se obtiene mediante el producto del factor com´ un num´erico m´as grande, multiplicado por cada una de las literales comunes y elevada al m´ınimo exponente con el que aparece en todos los t´erminos de la expresi´on algebraica dada. En la expresi´on 6x2 y − 10x3 z + 14x4 t se tiene: 6x2 y = 2 · 3 · x2 · y;

10x3 z = 2 · 5 · x3 · z

y

14x4 t = 2 · 7 · x4 · t

por lo cual el factor com´ un mayor es 2x2 . Por ´esto se considera a 2x2 como el factor com´ un de la expresi´on 6x2 y − 10x3 z + 14x4 t. El otro factor de este polinomio es 6x2 y − 10x3 z + 14x4 t 6x2 y 10x3 z 14x4 t = − + = 3y − 5xz + 7x2 t . 2x2 2x2 2x2 2x2 Luego, la factorizaci´on del polinomio propuesto es 6x2 y − 10x3 z + 14x4 t = 2x2 (3y − 5xz + 7x2 t) . Tambi´en es importante tener presente que un factor com´ un puede tener m´as de un t´ermino. Por ejemplo (x + y) es un factor com´ un de la suma algebraica a(x + y) − b(x + y) + c(x + y) as´ı, se puede afirmar que a(x + y) − b(x + y) + c(x + y) = (x + y)(a − b + c)

2.5. Factorizaci´ on

Ejemplo

105

2.5.1 Factorizar las expresiones algebraicas siguientes.

1. ax2 y − 2axy + 3axy 2

2. 4x3 y 2 + 8x4 y 3 − 12x5 y 4

3. x2 (a − 2) + 5(a − 2)

4. 108a3 b3 x − 90a3 b3 x2 + 36a2 b4 x3

5. m2 (x2 + 2x + 3) + m(x2 + 2x + 3) − (x2 + 2x + 3) Soluciones. 1. ax2 y − 2axy + 3axy 2 = (axy)x − (axy)2 + (axy)3y = (axy)(x − 2 + 3y) . 2. 4x3 y 2 + 8x4 y 3 − 12x5 y 4 = = = =

22 x3 y 2 + 23 x4 y 3 − 22 3x5 y 4 (22 x3 y 2 )1 + (22 x3 y 2 )2xy − (22 x3 y 2 )3x2 y 2 (22 x3 y 2 )(1 + 2xy − 3x2 y 2 ) 4x3 y 2 (1 + 2xy − 3x2 y 2 ) .

3. x2 (a − 2) + 5(a − 2) = (a − 2)(x2 + 5) . 4. 108a3 b3 x − 90a3 b3 x2 + 36a2 b4 x3 = 22 33 a3 b3 x − 2(3)2 5a3 b3 x2 + 22 32 a2 b4 x3 = (32 2a2 b3 x)2a3 − (32 2a2 b3 x)5ax + (32 2a2 b3 x)2bx2 = (18a2 b3 x)6a − (18a2 b3 x)5ax + (18a2 b3 x)2bx2 = (18a2 b3 x)(6a − 5ax + 2bx2 ) . 5. m2 (x2 + 2x + 3) + m(x2 + 2x + 3) − (x2 + 2x + 3) = (x2 + 2x + 3)(m2 + m − 1) . Ejercicio 2.5.1 Factorizar las expresiones algebraicas siguientes. 1. ax2 − 2axy + 3ay 2

2. −4x2 − 12x − 20

3. ax2 y 3 + 2abx2 y 3 − 3acx2 y 3

4. x(x + 1) − 2(x + 1)

5. 36x3 y 2 − 60x4 y 3 + 84x5 y 4

6. a(x − 1)2 − b(x − 1)2 + (x − 1)2



106

Cap´ıtulo 2.

´ Algebra

7. 24a3 b2 + 48a3 b3 − 120a4 b3 − 72a5 b4 8. (x − 2)(x + 5) − (x − 2)(x − 3) + (2x − 5)(x − 2) 9. 600m5 n3 + 450m4 n4 − 300m3 n5 10. x2 (a2 − a + 1) − (a2 − a + 1)x + 2(a2 − a + 1) Soluciones. 1. a(x2 − 2xy + 3y 2 )

2. −4(x2 + 3x + 5)

3. ax2 y 3 (1 + 2b − 3c)

4. (x + 1)(x − 2)

5. 12x3 y 2 (3 − 5xy + 7x2 y 2 )

6. (x − 1)2 (a − b + 1)

7. 24a3 b2 (1 + 2b − 5ab − 3a2 b2 )

8. (x − 2)(2x + 3)

9. 150m3 n3 (4m2 + 3mn − 2n2 )

2.5.2.

10. (x2 − x + 2)(a2 − a + 1)

Factorizaci´ on por agrupaci´ on

Existen expresiones algebraicas que no tienen un factor com´ un, pero que pueden ser factorizadas despu´es de llevar a cabo una adecuada agrupaci´on de t´erminos. Se agrupan o asocian parcialmente los t´erminos que contienen un factor com´ un, para as´ı obtener una expresi´on algebraica que tenga un factor com´ un. La forma de agrupar o asociar los t´erminos puede no ser u ´nica. Ejemplo

2.5.2 Factorizar las expresiones algebraicas siguientes:

1. a2 x + b2 x + a2 y + b2 y

2. 10x2 − 15xy + 8x − 12y

3. ax2 + ay 2 + bz 2 − bx2 − az 2 − by 2 Soluciones. 1. a2 x + b2 x + a2 y + b2 y es una expresi´on algebraica que no tiene un factor com´ un. Se observa que los dos primeros t´erminos a2 x y b2 x tienen el

2.5. Factorizaci´ on

107

factor com´ un x, mientras que los dos u ´ltimos t´erminos a2 y y b2 y tienen el factor com´ un y. Agrupando a los t´erminos mencionados se tiene que: a2 x + b2 x + a2 y + b2 y = (a2 x + b2 x) + (a2 y + b2 y) = x(a2 + b2 ) + y(a2 + b2 ) donde se tiene el factor com´ un (a2 + b2 ), por lo que a2 x + b2 x + a2 y + b2 y = (a2 + b2 )(x + y) . Otra manera de factorizar esta expresi´on se tiene si agrupamos los t´erminos primero y tercero a2 x y a2 y que tienen el factor com´ un a2 , 2 2 as´ı como los t´erminos segundo y cuarto b x y b y que tienen el factor com´ un b2 . De esta forma se tiene que: a2 x + b2 x + a2 y + b2 y = (a2 x + a2 y) + (b2 x + b2 y) = a2 (x + y) + b2 (x + y) donde se tiene el factor com´ un (x + y), por lo que a2 x + b2 x + a2 y + b2 y = (x + y)(a2 + b2 ) = (a2 + b2 )(x + y) . 2. La expresi´on algebraica 10x2 −15xy+8x−12y no tiene un factor com´ un. Pero los dos primeros t´erminos si tienen factor com´ un, as´ı como los dos u ´ltimos. Se agrupa de esta manera 10x2 − 15xy + 8x − 12y = (10x2 − 15xy) + (8x − 12y) = 5x(2x − 3y) + 4(2x − 3y) = (2x − 3y)(5x + 4) . Tambi´en se puede factorizar agrupando los t´erminos primero y tercero, as´ı como los t´erminos segundo y cuarto para obtener 10x2 − 15xy + 8x − 12y = (10x2 + 8x) + (−15xy − 12y) = 2x(5x + 4) − 3y(5x + 4) = (5x + 4)(2x − 3y) .

108

Cap´ıtulo 2.

´ Algebra

3. La expresi´on algebraica ax2 + ay 2 + bz 2 − bx2 − az 2 − by 2 no tiene un factor com´ un, pero asociando los t´erminos primero, segundo y quinto, as´ı como los t´erminos tercero, cuarto y sexto se obtiene ax2 + ay 2 + bz 2 − bx2 − az 2 − by 2 = (ax2 + ay 2 − az 2 ) + (bz 2 − bx2 − by 2 ) = (ax2 + ay 2 − az 2 ) + (−bx2 − by 2 + bz 2 ) = a(x2 + y 2 − z 2 ) − b(x2 + y 2 − z 2 ) = (x2 + y 2 − z 2 )(a − b) . Otra manera es agrupando por separado los t´erminos que contienen x2 , los que contienen y 2 , as´ı como los que contienen z 2 . A saber, ax2 + ay 2 + bz 2 − bx2 − az 2 − by 2 = (ax2 − bx2 ) + (ay 2 − by 2 ) + (bz 2 − az 2 ) = (ax2 − bx2 ) + (ay 2 − by 2 ) + (−az 2 + bz 2 ) = x2 (a − b) + y 2 (a − b) − z 2 (a − b) = (a − b)(x2 + y 2 − z 2 ) .  Ejercicio 2.5.2 Factorizar las expresiones algebraicas siguientes: 1. ax2 + bx2 + ay 2 + by 2

2. a3 m + b2 n − b2 m − a3 n

3. 6x2 y − x − 2y + 3x3

4. 15x2 y 2 − 12y 3 + 4xy − 5x3

5. 6x3 y 3 − 4xy 2 + 6 − 9x2 y 6. x2 (a + 1)2 − a2 x2 + y 2 (a + 1)2 − a2 y 2 7. −ax2 m − bx2 m − ay 2 m − by 2 m 8. y 3 (a − 2)2 − 4ax2 − x2 (a − 2)2 + 4ay 3 9. a2 x2 − b2 y 2 − c2 x2 + a2 y 2 − b2 x2 − c2 y 2 10. x3 + 3x + 2xy − 2x2 − x2 y − 3y

2.5. Factorizaci´ on

109

Soluciones. 1. (x2 + y 2 )(a + b)

2. (m − n)(a3 − b2 )

3. (3x2 − 1)(x + 2y)

4. (5x2 − 4y)(3y 2 − x)

5. (3x2 y − 2)(2xy 2 − 3)

6. (2a + 1)(x2 + y 2 )

7. −m(a + b)(x2 + y 2 )

8. (a2 + 4)(y 3 − x2 )

9. (a2 − b2 − c2 )(x2 + y 2 )

2.5.3.

10. (x − y)(x2 − 2x + 3)

Diferencia de cuadrados

En la secci´on de productos notables se mostr´o que: (a + b)(a − b) = a2 − b2 . Ahora en factorizaci´on se destaca que:

a2 − b2 = (a + b)(a − b) , o sea, una diferencia de cuadrados es igual al producto de dos binomios conjugados . Se hace notar que en√la diferencia a2 − b2 , el minuendo es a2 y una de sus ra´ıces cuadradas es a2 = a, que es el t´ermino com´ un de los binomios conjugados (a√+ b) y (a − b); adem´as, el sustraendo es b2 y una de sus ra´ıces cuadradas es b2 = b, que es el t´ermino de los binomios que difiere en signo. Son estas observaciones las que se aplican para factorizar una diferencia de cuadrados. Ejemplo

2.5.3 Factorizar las expresiones algebraicas siguientes:

1. 4x2 − 9y 2

2. 16x2 y 4 − 25

3. −1 + a2n

4. x2 − 7

5. x4 − 1 Soluciones. 1. 4x2 − 9y 2 = (2x)2 − (3y)2 = (2x + 3y)(2x − 3y) . 2. 16x2 y 4 − 25 = (4xy 2 )2 − (5)2 = (4xy 2 − 5)(4xy 2 + 5) .

110

Cap´ıtulo 2.

´ Algebra

3. −1 + a2n = a2n − 1 = (an )2 − (1)2 = (an + 1)(an − 1) . √ √ √ 4. x2 − 7 = (x)2 − ( 7)2 = (x − 7)(x + 7) . 5. En este ejemplo se observa que en la primera factorizaci´on, el segundo factor (x2 − 1) tambi´en se factoriza por diferencia de cuadrados. x4 − 1 = (x2 )2 − (1)2 = (x2 + 1)(x2 − 1) = (x2 + 1)(x2 − 12 ) = (x2 + 1)[(x + 1)(x − 1)] = (x2 + 1)(x + 1)(x − 1) .



Ejercicio 2.5.3 Factorizar las expresiones algebraicas siguientes: x2 y 2 − 4 9

1. x2 − 16t2

2.

4 5. − + 25t2 x4 9

6. 4x2 − 5

9. (a + 3)2 − 36

3. x2n − y 2n

4. 100a4 − 81b2

7. x4 − 16

8. −9x2 y 4 z 6 + 1

10. x8 − 1

Soluciones.

(x

y ) (x y ) − 3 2 3

1. (x + 4t)(x − 4t)

2.

3. (xn + y n )(xn − y n )

4. (10a2 + 9b)(10a2 − 9b)

( 5.

2 5tx + 3

)(

2

2 5tx − 3

)

2

7. (x2 + 4)(x + 2)(x − 2) 9. (a + 9)(a − 3)

2.5.4.

2

+

6. (2x +



5)(2x −



5)

8. (1 + 3xy 2 z 3 )(1 − 3xy 2 z 3 ) 10. (x4 + 1)(x2 + 1)(x + 1)(x − 1)

Trinomio cuadrado perfecto

Se recuerda que el cuadrado de un binomio es: (a + b)2 = a2 + 2ab + b2

y

(a − b)2 = a2 − 2ab + b2 .

Ahora, para el proceso de factorizaci´on se tiene que:

2.5. Factorizaci´ on

111

a2 ± 2ab + b2 = (a ± b)2 , es decir, un trinomio cuadrado perfecto es igual al cuadrado de un binomio. Se observa en esta u ´ltima igualdad que: El trinomio est´a ordenado con respecto a la literal a de mayor a menor exponente y con respecto a la literal b de menor a mayor exponente. Las ra´ıces cuadradas con signo positivo del primer y del u ´ltimo t´ermino del trinomio generan los t´erminos a y b del binomio. El signo del segundo t´ermino del binomio es el del doble producto. Son estas observaciones las que se aplican para factorizar un trinomio cuadrado perfecto. Ejemplo

2.5.4 Factorizar las expresiones algebraicas siguientes.

1. x2 − 8x + 16 2. 12y + 4 + 9y 2 n m 2n 2m 4. 40a b + 25a + 16b 5. 4 − 4x2 + 4

3. 4x2 + 12xy + 9y 2

Soluciones. 1. El trinomio x2 − 8x + 16 est´a ordenado de exponente mayor a menor con respecto a la literal x. Las ra´ıces cuadradas con signo positivo del primero y u ´ltimo t´ermino son: √ √ x2 = x y 16 = 4 . El doble producto de las ra´ıces obtenidas es 2(x)(4) = 8x, que es precisamente el t´ermino intermedio del trinomio dado. Por lo tanto, el trinomio dado es un trinomio cuadrado perfecto con factorizaci´on: x2 − 8x + 16 = x2 − 2(x)(4) + 42 = (x − 4)2 . 2. El trinomio 9y 2 + 12y + 4 est´a ordenado de exponente mayor a menor con respecto a la literal y. Las ra´ıces cuadradas con signo positivo del primero y u ´ltimo t´ermino son: √ √ 9y 2 = 3y 4=2. y

112

Cap´ıtulo 2.

´ Algebra

El doble producto de las ra´ıces obtenidas es 2(3y)(2) = 12y, que es precisamente el t´ermino intermedio del trinomio dado. Por lo tanto, el trinomio dado es un trinomio cuadrado perfecto con factorizaci´on: 9y 2 + 12y + 4 = (3y)2 + 2(3y)(2) + 22 = (3y + 2)2 . 3. El trinomio 4x2 +12xy+9y 2 est´a ordenado de exponente mayor a menor con respecto a la literal x, y con respecto a la literal y, de exponente menor a exponente mayor. Las ra´ıces cuadradas con signo positivo del primero y u ´ltimo t´ermino son: √ √ y 4x2 = 2x 9y 2 = 3y . El doble producto de las ra´ıces obtenidas es 2(2x)(3y) = 12xy, que es precisamente el t´ermino intermedio del trinomio dado. Por lo tanto, el trinomio dado es un trinomio cuadrado perfecto y adem´as: 4x2 + 12xy + 9y 2 = (2x)2 + 2(2x)(3y) + (3y)2 = (2x + 3y)2 es la factorizaci´on pedida. 4. El trinomio 40an bm + 25a2n + 16b2m no est´a ordenado con respecto a ninguna de las literales. Se ordena con respecto a la literal a, reescribi´endolo de la siguiente manera: 25a2n + 40an bm + 16b2m . Las ra´ıces cuadradas con signo positivo del primero y u ´ltimo t´ermino del trinomio ya ordenado son: √ √ 25a2n = 5an y 16b2m = 4bm . El doble producto de las ra´ıces es: 2(5an )(4bm ) = 40an bm , que es precisamente el t´ermino intermedio del trinomio dado. Por lo tanto: 25a2n +40an bm +16b2m = (5an )2 +2(5an )(4bm )+(4bm )2 = (5an +4bm )2 . 5. El trinomio x4 − 4x2 + 4 est´a ordenado, de exponente mayor a menor, con respecto a la u ´nica literal x. Las ra´ıces cuadradas con signo positivo del primero y u ´ltimo t´ermino son: √ √ x4 = x2 y 4=2.

2.5. Factorizaci´ on

113

El doble producto de las ra´ıces es: 2(x2 )(2) = 4x2 , que es el t´ermino intermedio, con signo negativo, del trinomio dado. Por lo tanto, se tiene un trinomio cuadrado perfecto y su factorizaci´on es: x4 − 4x2 + 4 = (x2 )2 − 2(x2 )(2) + (2)2 = (x2 − 2)2 .  Ejercicio 2.5.4 Factorizar las expresiones algebraicas siguientes. 1. 25x2 − 30x + 9

2. 12a + 4 + 9a2

3. 25y 2 + 9x6 − 30x3 y

4. 1 + x4 − 2x2

5. y 6 + 9 + 6y 3

6.

9x2 − 6x + 4 4

7. x2n − 4xn + 4

8. 1 + 2x5 + x10

9.

1 + a2 − 2 a2

√ 10. y n + xn + 2 xn y n Soluci´ on 1. (5x − 3)2

2. (3a + 2)2 (

3

2

5. (y + 3) ( 9.

)2 1 −a a

2.5.5.

6.

3 x−2 2

3. (5y − 3x3 )2

4. (x2 − 1)2

7. (xn − 2)2

8. (1 + x5 )2

)2

10. (xn/2 + y n/2 )2

Trinomio cuadr´ atico

Se trata ahora de factorizar el trinomio cuadr´atico ax2 + bx + c , en factores reales y lineales de x, donde a, b y c son n´ umeros reales y a ̸= 0. El 2 n´ umero b − 4ac se llama discriminante. En la siguiente secci´on se muestra que el trinomio cuadr´atico ax2 + bx + c se puede factorizar, usando n´ umeros reales, cuando el discriminante es positivo o cero. Si el discriminante es menor que cero no se puede factorizar usando n´ umeros reales.

114

Cap´ıtulo 2.

´ Algebra

Antes de ver la forma general para factorizar, se aborda la factorizaci´on de dos casos particulares del trinomio. En estos casos se procede por tanteo y se restringe a una factorizaci´on que involucra n´ umeros enteros. El caso general justifica estos procedimientos. En ambos casos se supone que la expresi´on ax2 + bx + c s´ı se puede factorizar, es decir, su discriminante es mayor o igual que cero. • Primer caso. Cuando a = 1 y por tanto el trinomio es x2 + bx + c. Se buscan dos n´ umeros enteros m y n tales que mn = c y m + n = b. Existiendo este par de n´ umeros, se sustituyen m + n en lugar de b y mn en lugar de c, para luego factorizar x2 + bx + c = x2 + (m + n)x + mn = (x + m)(x + n) . Por lo tanto, si m y n son n´ umeros enteros tales que su producto es c y su suma es b, entonces el trinomio se factoriza como (x + m)(x + n). El m´etodo aqu´ı expuesto se utiliza como lo muestran los ejemplos siguientes. Ejemplo

2.5.5 Factorizar los trinomios cuadr´aticos

1. x2 + 5x + 6

2. x2 − 7x + 12

3. x2 − 2x − 8

4. x2 + 2x + 2 Soluciones. 1. Comparando x2 + 5x + 6 con ax2 + bx + c se tiene: a = 1, b = 5 y c = 6. Su discriminante es b2 − 4ac = 52 − 4(1)(6) = 25 − 24 = 1 > 0, luego s´ı se puede factorizar. M´as a´ un, dos n´ umeros m y n tales que: mn = 6 y m + n = 5, son m = 2 y n = 3. Por lo tanto, x2 + 5x + 6 = (x + m)(x + n) = (x + 2)(x + 3) . 2. Comparando x2 − 7x + 12 con ax2 + bx + c se tiene: a = 1, b = −7 y c = 12. Dos n´ umeros m y n tales que: mn = 12 y m + n = −7, son m = −4 y n = −3. Por lo tanto, x2 − 7x + 12 = (x + m)(x + n) = [x + (−4)][x + (−3)] = (x − 4)(x − 3) .

2.5. Factorizaci´ on

115

El trinomio se pudo factorizar porque su discriminante es b2 − 4ac = (−7)2 − 4(1)(12) = 49 − 48 = 1 > 0. 3. Para x2 − 2x − 8 se tiene: a = 1, b = −2 , c = −8 y b2 − 4ac = 36 > 0. Los dos n´ umeros cuyo producto es −8 y suma es −2 son −4 y 2. Por lo tanto x2 − 2x − 8 = (x − 4)(x + 2) . 4. En x2 + 2x + 2 se tiene a = 1, b = 2 y c = 2. Su discriminante es b2 − 4ac = 22 − 4(1)(2) = 4 − 8 = −4 < 0, luego esta expresi´on no se puede factorizar usando n´ umeros reales.  • El segundo caso es cuando a ̸= 1 en el trinomio ax2 + bx + c. Se multiplica y divide al trinomio por a a(ax2 + bx + c) a2 x2 + abx + ac (ax)2 + b(ax) + ac = = . a a a Al escribir ax = u se tiene: u2 + bu + ac ax2 + bx + c = a y se procede a factorizar u2 + bu + ac como en el primer caso. As´ı, se determinan m y n tales que u2 + bu + ac = (u + m)(u + n), y se factoriza como u2 + bu + ac (u + m)(u + n) (ax + m)(ax + n) ax2 + bx + c = = = . a a a ´ltima expresi´on.  Finalmente se simplifica la u ax2 + bx + c =

Ejemplo

2.5.6 Factorizar los trinomios cuadr´aticos

1. 2x2 + 7x − 15

2. 6x2 − x − 2

3. 5t2 − 6t + 4

Soluciones. 1. Comparando 2x2 + 7x − 15 con ax2 + bx + c se tiene que a = 2, b = 7 y c = −15. Su discriminante es b2 − 4ac = 72 − 4(2)(−15) = 169 > 0, luego se puede factorizar. As´ı 22 x2 + 2(7x) − 30 2(2x2 + 7x − 15) = 2 2 (2x)2 + 7(2x) − 30 = . 2

2x2 + 7x − 15 =

116

Cap´ıtulo 2.

´ Algebra

Al poner 2x = u se tiene: 2x2 + 7x − 15 =

u2 + 7u − 30 (u + 10)(u − 3) = , 2 2

donde se obtuvieron m = 10, n = −3 tales que mn = −30, m + n = 7. Al sustituir el valor u = 2x y simplificar (u + 10)(u − 3) (2x + 10)(2x − 3) = 2 2 2x + 10 = · (2x − 3) = (x + 5)(2x − 3) . 2

2x2 + 7x − 15 =

2. Para 6x2 − x − 2 con ax2 + bx + c se tiene que a = 6, b = −1, c = −2. Su discriminate es b2 − 4ac = (−1)2 − 4(6)(−2) = 49 > 0, entonces se puede factorizar. Aplicando directamente el m´etodo, se tiene 6x2 − x − 2 =

6(6x2 − x − 2) 62 x2 + 6(−x) − 6(2) = 6 6

=

u2 − u − 12 (6x)2 − 1(6x) − 12 = 6 6

=

(u − 4)(u + 3) 1 = (6x − 4)(6x + 3) 6 6

=

1 6 (2)(3x − 2)(3)(2x + 1) = (3x − 2)(2x + 1) 6 6

= (3x − 2)(2x + 1) . por lo tanto 6x2 − x − 2 = (3x − 2)(2x + 1). 3. De 5x2 − 6x + 4 se tiene que a = 5, b = −6, c = 4. Su discriminate es b2 − 4ac = (−6)2 − 4(5)(4) = −44 < 0, entonces no se puede factorizar usando n´ umeros reales.  • El caso general para factorizar al trinomio es la completaci´ on de cuadrados. Se estudia aqu´ı una forma general para factorizar al trinomio cuadr´atico 2 ax + bx + c, cuando b2 − 4ac ≥ 0. Este m´etodo se conoce como completar el cuadrado y consiste en un procedimiento para transformar al

2.5. Factorizaci´ on

117

2 trinomio cuadr´ ( )atico ax + bx + c en una expresi´on algebraica de la forma 2 a (x + p) − q , donde p y q son n´ umeros que se deben determinar. Es decir ( ) ax2 + bx + c = a (x + p)2 − q . (2.5.3)

Ya que (

b x± 2

)2

( ) ( )2 b b2 b = x ± 2x = x2 ± bx + , + 2 2 4 2

b2 b2 y x2 − bx + son los trinomios cuadrados perfectos 4 4 2 asociados a los binomios cuadr´aticos x +bx y x2 −bx, respectivamente. Luego, para obtener el trinomio cuadrado perfecto asociado a los binomios x2 ± bx b2 se debe sumar el n´ umero , que es el cuadrado de la mitad del coeficiente 4 de x. A este procedimiento se le denomina: completar el cuadrado en el binomio x2 ± bx. entonces x2 + bx +

Ejemplo

2.5.7 Determinar el n´ umero que debe ser sumado para completar el cuadrado perfecto asociado en cada binomio. 1. x2 − 10x

4 2. u2 + u 3

Soluciones. 2 1. Para completar ( )el2 cuadrado en el binomio x − 10x se debe sumar el 10 n´ umero: − = (−5)2 = 25. As´ı: 2

x2 − 10x + 25 = (x − 5)2 . 4 2. Para completar el cuadrado en el binomio u2 + u se debe sumar el 3 ( ( ))2 ( )2 1 4 2 4 n´ umero: = = . As´ı: 2 3 3 9 ( )2 4 4 2 2 u + u+ = u+ .  3 9 3

118

Cap´ıtulo 2.

´ Algebra

En lo hecho anteriormente se ha completado el cuadrado en binomios de la forma x2 ± bx, donde el coeficiente de x2 es la unidad y no hay un t´ermino constante. Ahora, para completar el cuadrado en un trinomio cuadr´atico de la forma x2 + bx + c, se completa el cuadrado del binomio x2 + bx, sin preocuparse por el t´ermino constante. As´ı tambi´en, para completar el cuadrado en un trinomio cuadr´atico ax2 + bx + c, primero se debe considerar el n´ umero a como factor com´ un y escribir la factorizaci´on ( ) b c 2 2 ax + bx + c = a x + x + a a b para luego completar el cuadrado del binomio x2 + x, sumando el t´ermino: a [ ( )]2 ( )2 1 b b = . 2 a 2a Finalmente se recuerda que si se suma y resta un t´ermino en una expresi´on algebraica dicha expresi´on no se modifica. El procedimiento general para completar el trinomio cuadrado perfecto es el siguiente: [ ] b c 2 2 ax + bx + c = a x + x + a a [ ( [ ])2 ( [ ])2 ] b 1 b 1 b c = a x2 + x + + − a 2 a a 2 a [ ( )2 ( )2 ] b b c b = a x2 + x + + − a 2a a 2a [( ] )2 b c b2 =a x+ + − 2 2a a 4a [( ] )2 b 4ac − b2 =a x+ + 2a 4a2 [( ] )2 b b2 − 4ac =a x+ − 2a 4a2

2.5. Factorizaci´ on

119

El u ´ltimo rengl´on da los valores expl´ıcitos de p y q en la ecuaci´on (2.5.3). Retomando el objetivo de factorizar al trinomio cuadr´atico ax2 + bx + c, se analiza la igualdad [( ] )2 2 b b − 4ac ax2 + bx + c = a x + − . (2.5.4) 2a 4a2 Teorema 2.5.1 El trinomio cuadr´atico ax2 + bx + c se puede factorizar en factores lineales con coeficientes reales si y s´olo si el discriminante b2 − 4ac es mayor o igual que cero. Demostraci´on. Se supone primero que b2 − 4ac ≥ 0. 2 1. Si(b2 − 4ac )2= 0, entonces se tiene la factorizaci´on ax + bx + c = b a x+ , que dice que la expresi´on original es un trinomio cuadra2a do perfecto.

2. Si b2 − 4ac > 0, entonces

b2 − 4ac umero > 0. Si se denota por r2 al n´ 4a2

b2 − 4ac se tiene entonces que 4a2 [( ] [( ] )2 )2 2 b b − 4ac b ax2 + bx + c = a x + − =a x+ − r2 . 2a 4a2 2a

positivo

Esto muestra que la expresi´on original es el producto de a por una diferencia de cuadrados, la cual se puede factorizar mediante un producto de binomios conjugados. Se tiene en este caso que: [( ) ] [( ) ] b b 2 ax + bx + c = a x + −r x+ +r . 2a 2a b2 − 4ac b2 − 4ac < 0 y − > 0. 4a2 4a2 b2 − 4ac Si se denota por d2 al n´ umero positivo − se tiene entonces que 4a2 [( ] [( ] )2 )2 2 b b − 4ac b ax2 + bx + c = a x + − =a x+ + d2 . 2a 4a2 2a Si b2 − 4ac < 0, entonces

120

Cap´ıtulo 2.

´ Algebra

Esto muestra que la expresi´on original es el producto de a por una suma de cuadrados, la cual no puede ser factorizada con n´ umeros reales. En este caso el trinomio cuadr´atico ax2 +bx+c no puede ser factorizado mediante factores reales.  En los siguientes ejemplos aplican el m´etodo mostrado. Ejemplo

2.5.8 Factorizar los trinomios cuadr´aticos siguientes.

1. x2 − 4x + 1 4. 9x2 + 42x + 49

3. 6x2 − 17x + 12 6. 4x2 − 4x − 11

2. x2 + 3x + 5 5. 3x2 − 4x + 5

Soluciones. 1. Identificando x2 − 4x + 1 con ax2 + bx + c se tiene que a = 1, b = −4 y c = 1. El discriminante es b2 − 4ac = (−4)2 − 4 (1) (1) = 16 − 4 = 12 > 0 y por tanto x2 − 4x + 1 puede ser factorizado mediante factores reales. Completando cuadrados: (

4 x − 4x + 1 =x − 4x + − 2 2

2

)2

(

4 +1− − 2

)2

=x2 − 4x + (−2)2 + 1 − (−2)2 = (x − 2)2 + 1 − 4 =(x − 2)2 − 3 (√ )2 = (x − 2)2 − 3 ( ) √ ( √ ) = x−2− 3 x−2+ 3 . √ )( √ ) ( Esto es: x2 − 4x + 1 = x − 2 − 3 x − 2 + 3 . 2. Identificando x2 + 3x + 5 con ax2 + bx + c se tiene que a = 1, b = 3 y c = 5. El discriminante es b2 − 4ac = (3)2 − 4 (1) (5) = 9 − 20 = −11 < 0 y por tanto x2 + 3x + 5 no puede ser factorizado mediante factores reales.

2.5. Factorizaci´ on

121

Completando cuadrados:

( )2 ( )2 3 3 +5− x + 3x + 5 =x + 3x + 2 2 ( )2 3 9 = x+ +5− 2 4 ( )2 3 20 − 9 = x+ + 2 4 ( )2 3 11 = x+ + . 2 4 2

2

Lo que indica que x2 + 3x + 5 es una suma de cuadrados ya que

( x2 + 3x + 5 =

x+

3 2

)2 +

11 = 4

( x+

3 2

(√

)2 +

11 2

)2

luego, no se tienen factores reales.

3. El discriminante es b2 −4ac = (−17)2 −4 (6) (12) = 289−288 = 1 > 0 y por tanto 6x2 − 17x + 12 puede ser factorizado mediante factores reales.

122

Cap´ıtulo 2.

´ Algebra

Completando cuadrados: ( ) 17 12 2 2 6x − 17x + 12 =6 x − x + 6 6 ( [ ( [ ( )]2 )]2 ) 1 1 17 17 17 +2− =6 x2 − x + − − 6 2 6 2 6 [ ( )2 ( )2 ] 17 17 17 +2− − =6 x2 − x + − 6 12 12 [( ] )2 17 289 =6 x − +2− 12 144 [( ] )2 17 288 − 289 =6 x − + 12 144 ] [( [( )2 )2 ( )2 ] 1 1 17 17 − =6 x− − =6 x − 12 144 12 12 [( ) ] [( ) ] 17 17 1 1 =6 x − x− − + 12 12 12 12 ( ( )( ) )( ) 18 3 16 4 =6 x − x− = 3 (2) x − x− 12 12 2 3 ) ( ) ( 4 3 3 x− = (2x − 3) (3x − 4) . =2 x − 2 3 Esto es: 6x2 − 17x + 12 = (2x − 3) (3x − 4) . 4. Comparando 9x2 + 42x + 49 con ax2 + bx + c se tiene que a = 9, b = 42 y c = 49. El discriminante es b2 − 4ac = (42)2 − 4 (9) (49) = 1764 − 1764 = 0 y por tanto 9x2 + 42x + 49 puede ser factorizado mediante factores reales y adem´as es un trinomio cuadrado perfecto, a saber 9x2 + 42x + 49 = (3x + 7)2 . Por supuesto, se obtiene el mismo resultado aplicando el m´etodo expuesto. 5. Comparando 3x2 − 4x + 5 con ax2 + bx + c se tiene que a = 3, b = −4 y c = 5. El discriminante es b2 − 4ac = (−4)2 − 4 (3) (5) = 16 − 60 = −44 < 0,

2.5. Factorizaci´ on

123

y por tanto 3x2 − 4x + 5 no puede ser factorizado mediante factores reales. Completando cuadrados: [ ] 4 5 2 3x − 4x + 5 =3 x − + 3 3 [ ( [ ]]2 ( [ ])2 ] 1 5 1 4 4 4 =3 x2 − x + + − − − 3 2 3 3 2 3 [ ] ( )2 ( )2 5 4 2 2 2 + − − =3 x − x + − 3 3 3 3 [( ] [ ] )2 )2 ( 2 5 4 2 15 − 4 =3 x − + − =3 x− + 3 3 9 3 9 [( ] )2 2 11 =3 x − + . 3 9 2

Lo que nos indica que 3x2 − 4x + 5 es una suma de cuadrados, ya que ( ( )2 )2 (√ )2 2 11 2 11 3x2 − 4x + 5 = 3 x − + =3 x− + . 3 3 3 3 Luego, no se tienen factores reales. 6. Comparando 4x2 − 4x − 11 con ax2 + bx + c se tiene que a = 4, b = −4 y c = −11. El discriminante es b2 −4ac = (−4)2 −4 (4) (−11) = 16+176 = 192 > 0 y por tanto 4x2 −4x−11 puede ser factorizado mediante factores reales. Completando cuadrados: [ ] 11 2 4x − 4x − 11 =4 x − x − 4 [ ( )2 ( )2 ] 1 11 1 =4 x2 − x + − − − − 2 4 2 [( ] [ ] )2 )2 ( 1 11 1 12 1 =4 x − − − =4 x− − 2 4 4 2 4 2

124

Cap´ıtulo 2.

[(

)2 ( ) ] √ 2 1 4x2 − 4x − 11 =4 −3 =4 x− − 3 2 [( ] [( ] ) ) √ √ 1 1 =4 x − x− − 3 + 3 2 2 ( )( ) 1 √ 1 √ =2 (2) x − − 3 x− + 3 2 2 ( ) ( ) 1 √ 1 √ =2 x − − 3 2 x − + 3 2 2 ( √ )( √ ) = 2x − 1 − 2 3 2x − 1 + 2 3 . 1 x− 2

]

´ Algebra

)2

[(

√ )( √ ) ( Esto es: 4x2 − 4x − 11 = 2x − 1 − 2 3 2x − 1 + 2 3 .



Ejercicio 2.5.5 Factorizar los siguientes trinomios. 1. 12x2 − 7x − 10

2. x2 − 6x + 4

3. 25x2 − 40x + 16

4. 16x2 − 16x + 1

5. −2x2 + 3x − 4

6. 15x2 − 8x − 16

7. 4x2 + 12x − 141

8. 9x2 + 48x + 64

9. 8x2 − 26x + 15

10. 9x2 − 30x − 7 Soluciones. √

1. (3x + 2)(4x − 5)

2. (x − 3 +

3. (5x − 4)2

4. (4x − 2 −

5.

6. (5x + 4)(3x − 4)

No se puede factorizar

√ √ 7. (2x + 3 − 5 6)(2x + 3 + 5 6) 9. (2x − 5)(4x − 3)

5)(x − 3 −





3)(4x − 2 +

5) √

3)

8. (3x + 8)2 √ √ 10. (3x − 5 − 4 2)(3x − 5 + 4 2)

2.5. Factorizaci´ on

2.5.6.

125

Suma y diferencia de cubos

As´ı como se han estudiado algunos productos notables y su uso en la factorizaci´on, tambi´en se puede hablar de algunos cocientes notables y utilizarlos para factorizar expresiones algebraicas muy particulares. Se estudian los cocientes a3 + b3 a3 − b 3 y . a+b a−b Efectuando la primera divisi´on se obtiene para el primer cociente

a+b

a2 a3 − a3

− ab + b3 − a2 b − a2 b + a2 b

+

b2

+ b3 + ab2 + ab2 − ab2

+ b3 − b3 0

Es decir, a3 + b3 = a2 − ab + b2 a+b y de forma an´aloga a3 − b3 = a2 + ab + b2 . a−b Por lo tanto se tienen las siguientes igualdades

a3 + b3 = (a + b)(a2 − ab + b2 ) y

a3 − b3 = (a − b)(a2 + ab + b2 ) . La primera dice como factorizar a una suma de cubos y la otra una diferencia de cubos.

126

Cap´ıtulo 2.

Ejemplo 1. x3 + 27

´ Algebra

2.5.9 Factorizar los binomios 2. x3 − 27

5. (x + 2)3 + (x − 1)3

3. 8x3 − 1

4. 64x9 − 8y 6

6. (a + b)3 − (a − b)3

Soluciones 1. x3 + 27 = x3 + 33 = (x + 3)[x2 − (x)(3) + 32 ] = (x + 3)(x2 − 3x + 9) . 2. x3 − 27 = x3 − 33 = (x − 3)[x2 + (x)(3) + 32 ] = (x − 3)(x2 + 3x + 9) . 3. 8x3 − 1 = (2x)3 − 13 = (2x − 1)[(2x)2 + (2x)(1) + 12 ] = (2x − 1)(4x2 + 2x + 1) . 4. 64x9 − 8y 6 = (4x3 )3 − (2y 2 )3 = (4x3 − 2y 2 )[(4x3 )2 + (4x3 )(2y 2 ) + (2y 2 )2 ] = (4x3 − 2y 2 )(16x6 + 8x3 y 2 + 4y 4 ) . 5. (x + 2)3 + (x − 1)3 = [(x + 2) + (x − 1)][(x + 2)2 − (x + 2)(x − 1) + (x − 1)2 ] = (x + 2 + x − 1)[(x2 + 4x + 4) − (x2 + x − 2) + (x2 − 2x + 1)] = (2x + 1)[x2 + 4x + 4 − x2 − x + 2 + x2 − 2x + 1] = (2x + 1)(x2 + x + 7) . 6. (a + b)3 − (a − b)3 = [(a + b) − (a − b)][(a + b)2 + (a + b)(a − b) + (a − b)2 ] = (a + b − a + b)[(a2 + 2ab + b2 ) + (a2 − b2 ) + (a2 − 2ab + b2 )] = (2b)[a2 + 2ab + b2 + a2 − b2 + a2 − 2ab + b2 ] = 2b(3a2 + b2 ) . 

2.5. Factorizaci´ on

127

Se observa que un factor de (a3 + b3 ) es (a + b) y que el otro factor se a3 + b3 . De manera an´aloga se procede para obtiene efectuando la divisi´on a+b factorizar (a5 + b5 ), (a7 + b7 ), . . . , (an + bn ) para n entero positivo impar. En estos casos, un factor de (an + bn ) es (a + b) y el otro factor se obtiene an + bn . Por ejemplo efectuando la divisi´on a+b

a5 + b5 = (a + b)(a4 − a3 b + a2 b2 − ab3 + b4 ) .

El mismo procedimiento se aplica para factorizar (an − bn ), para n entero positivo, donde uno de los factores es (a − b) y el otro se obtiene al efectuar an − b n la divisi´on . Por ejemplo a−b

a5 − b5 = (a − b)(a4 + a3 b + a2 b2 + ab3 + b4 ) .

Ejercicio 2.5.6 Factorizar los binomios

1. x3 + 8

2. 1 − x3

3. 27x3 − 8

4. 8x3 + 125

5. 64a3 − 729y 3

6.

8. (x − 2)3 + (x − 3)3

9. (x − 4)3 − (1 − x)3

7.

x3 125 − 3 125 x

m6 n3 + 27 8

10. x3 + 1

11. a5 − b5

12. x5 + 1

13. x5 − 32

14. a5 + t5

15. x6 − 1

128

Cap´ıtulo 2.

´ Algebra

Soluciones. 1. (x + 2)(x2 − 2x + 4)

2. (1 − x)(1 + x + x2 )

3. (3x − 2)(9x2 + 6x + 4)

4. (2x + 5)(4x2 − 10x + 25) (

5. (4a − 9y)(16a + 36ay + 81y ) 2

( 7.

x 5 − 5 x

)(

2

25 x2 +1+ 2 25 x

6.

m2 n + 3 2

)(

m4 m2 n n2 − + 9 6 4

)

)

9. (2x − 5)(x2 − 5x + 13)

8. (2x − 5)(x2 − 5x + 7) 10. (x + 1)(x2 − x + 1)

11. (a − b)(a4 + a3 b + a2 b2 + ab3 + b4 ) 12. (x + 1)(x4 − x3 + x2 − x + 1) 13. (x − 2)(x4 + 2x3 + 4x2 + 8x + 16) 14. (a + t)(a4 − a3 t + a2 t2 − at3 + t4 ) 15. (x − 1)(x5 + x4 + x3 + x2 + x + 1)

2.5.7.

Miscel´ anea de ejercicios

Para factorizar una expresi´on algebraica es usual aplicar m´as de un tipo de factorizaci´on. Un procedimiento recomendable para factorizar es el siguiente: primero ver si existe o no un factor com´ un, en caso de existir determinarlo y factorizar, para luego proceder a factorizar el factor restante, mediante alguno o algunos de los m´etodos mencionados, y as´ı sucesivamente. Ejemplo

2.5.10 Factorizar las expresiones algebraicas siguientes.

1. x3 − 4x

2. 3x4 − 6x3 − 24x2

3. t5 − 2t3 + t

4. u4 − 81

5. a3 bx + ab3 y − a3 by − ab3 x

6. x4 − 5x2 + 4

7. −16y − 2y 4

8. −10x3 + 25x2 + 15x

2.5. Factorizaci´ on

129

Soluciones. 1. Ya que x3 − 4x tiene el factor com´ un x, entonces x3 − 4x = x(x2 − 4) . Luego como (x2 − 4) es una diferencia de cuadrados x2 − 4 = x2 − 22 = (x − 2)(x + 2) y por lo tanto, x3 − 4x = x(x2 − 4) = x(x − 2)(x + 2). 2. En 3x4 − 6x3 − 24x2 se tiene el factor com´ un 3x2 , entonces 3x4 − 6x3 − 24x2 = 3x2 (x2 − 2x − 8) . Luego, x2 − 2x − 8 es un trinomio cuadr´atico que se puede factorizar como x2 − 2x − 8 = (x + m)(x + n) = (x − 4)(x + 2) y por lo tanto, 3x4 − 6x3 − 24x2 = 3x2 (x2 − 2x − 8) = 3x2 (x − 4)(x + 2) . 3. ( ) t5 − 2t3 + t = t(t4 − 2t2 + 1) = t (t2 )2 − 2(t2 ) + 1 ( )2 = t(t2 − 1)2 = t(t2 − 12 )2 = t (t + 1)(t − 1) = t(t + 1)2 (t − 1)2 . 4. El binomio u4 − 81 es una diferencia de cuadrados, as´ı u4 − 81 = (u2 )2 − (9)2 = [(u2 ) + (9)][(u2 ) − (9)] = (u2 + 9)(u2 − 9) . Como el binomio u2 + 9 es irreducible y el binomio u2 − 9 = u2 − 32 es de nuevo una diferencia de cuadrados, se tiene u4 − 81 = (u2 + 9)(u2 − 32 ) = (u2 + 9)(u + 3)(u − 3) .

130

Cap´ıtulo 2.

´ Algebra

5. En a3 bx + ab3 y − a3 by − ab3 x se tiene el factor com´ un ab, entonces a3 bx + ab3 y − a3 by − ab3 x = (ab)(a2 x + b2 y − a2 y − b2 x) Luego, al otro factor se le puede aplicar la factorizaci´on por agrupaci´on a2 x + b2 y − a2 y − b2 x = (a2 x − a2 y) + (−b2 x + b2 y) = a2 (x − y) − b2 (x − y) = (x − y)(a2 − b2 ) . Hasta aqu´ı se tiene que a3 bx + ab3 y − a3 by − ab3 x = ab(a2 x + b2 y − a2 y − b2 x) = ab(x − y)(a2 − b2 ) y como a2 − b2 es una diferencia de cuadrados se tiene finalmente a3 bx + ab3 y − a3 by − ab3 = ab(x − y)(a2 − b2 ) = ab(x − y)(a + b)(a − b) . 6. El trinomio x4 − 5x2 + 4 no tiene un factor com´ un y no es un trinomio 4 2 cuadr´atico. Se puede reescribir como x −5x +4 = (x2 )2 −5(x2 )+4 y con el cambio de variable a = x2 se transforma en un trinomio cuadr´atico: x4 − 5x2 + 4 = (x2 )2 − 5(x2 ) + 4 = a2 − 5a + 4 . Debido a que: a2 − 5a + 4 = (a − 4)(a − 1) se tiene x4 − 5x2 + 4 = [(x2 ) − 4][(x2 ) − 1] = (x2 − 4)(x2 − 1) y como (x2 − 4) y (x2 − 1) son diferencias de cuadrados x4 − 5x2 + 4 = [(x + 2)(x − 2)][(x + 1)(x − 1)] = (x + 2)(x − 2)(x + 1)(x − 1) . 7. El binomio −16y − 2y 4 tiene como factor com´ un −2y, entonces −16y − 2y 4 = (−2y)(8 + y 3 ) = −2y(y 3 + 8) . El binomio y 3 + 8 = y 3 + 23 es una suma de cubos y se factoriza como y 3 + 8 = y 3 + 23 = (y + 2)(y 2 − 2y + 4) Por lo tanto, −16y − 2y 4 = −2y(8 + y 3 ) = −2y(y 3 + 8) = −2y(y + 2)(y 2 − 2y + 4) .

2.5. Factorizaci´ on

131

8. Ya que −10x3 + 25x2 + 15x tiene el factor com´ un −5x, entonces, −10x3 + 25x2 + 15x = −5x(2x2 − 5x − 3) . Luego, el trinomio cuadr´atico 2x2 − 5x − 3 se puede factorizar como 2x2 − 5x − 3 = 2x2 + (−6 + 1)x − 3 = 2x2 − 6x + x − 3 = 2x(x − 3) + 1(x − 3) = (x − 3)(2x + 1) . Por lo tanto, −10x3 + 25x2 + 15x = −5x(2x2 − 5x − 3) = −5x(x − 3)(2x + 1) .

Ejercicio 2.5.7 Factorizar las expresiones algebraicas siguientes.

1. x3 − x

2. 2x3 + 4x2 − 6x

3. −3x4 + 9x3 + 30x2

4. a2 m − a2 n + 2abm − 2abn

5. 6x3 − 7x2 + 2x

6. x4 − x

7. a2 x3 + b2 xy 2 − b2 x3 − a2 xy 2

8. 10x − 5x3

9. t4 − 10t2 + 9

10. x4 + 27x

11. −t5 + 3t3 + 4t

12. 4x4 − 10x3 − 6x2

13. a4 x2 − a4 − ab3 x2 + ab3

14. x6 − 1

15. 2x6 − 16x4 + 32x2

132

Cap´ıtulo 2.

Soluciones. 1. x(x + 1)(x − 1)

2. 2x(x + 3)(x − 1)

3. −3x2 (x − 5)(x + 2)

4. a(a + 2b)(m − n)

5. x(3x − 2)(2x − 1)

6. x(x − 1)(x2 + x + 1)

7. x(x + y)(x − y)(a + b)(a − b)

8. −5x(x −

9. (t + 1)(t − 1)(t + 3)(t − 3) 11. −t(t2 + 1)(t + 2)(t − 2)

2)(x +



2)

10. x(x + 3)(x2 − 3x + 9) 12. 2x2 (2x + 1)(x − 3)

13. a(a − b)(a2 + ab + b2 )(x + 1)(x − 1) 14. (x + 1)(x − 1)(x2 − x + 1)(x2 + x + 1) 15. 2x2 (x − 2)2 (x + 2)2



´ Algebra

2.6. Operaciones con fracciones algebraicas

2.6.

133

Operaciones con fracciones algebraicas

En el conjunto de los polinomios con las operaciones de suma y producto se puede verificar que se cumplen las propiedades enunciadas para los n´ umeros reales, con excepci´on de la existencia de inversos multiplicativos. Por ello imitando la definici´on de los n´ umeros racionales, a partir de los cocientes de n´ umeros enteros, se introducen ahora las fracciones algebraicas racionales como el conjunto de todos los cocientes entre polinomios, a saber { } p(x) : con p(x), q(x) polinomios y q(x) ̸= 0 . q(x) En este conjunto se definir´an las operaciones de suma y producto de manera que ahora se verifiquen las propiedades dichas para los n´ umeros reales, con lo cual se puede hacer ´algebra de la misma forma que se hace en los n´ umeros reales, en particular con los n´ umeros racionales. Se muestra primero como simplificar fracciones algebraicas racionales, para a continuaci´on definir sus operaciones y presentar algunos ejemplos. Despu´es se presentan en general las fracciones algebraicas, en donde tanto el numerador como el denominador permiten expresiones algebraicas que no son necesariamente polinomios; su presentaci´on y operatividad se realiza s´olo mediante ejemplos, se aprovechan estas expresiones para introducir una t´ecnica fundamental en la simplificaci´on de las mismas: la racionalizaci´on.

2.6.1.

Simplificaci´ on de fracciones algebraicas

Al igual que con los n´ umeros racionales, una fracci´on algebraica racional tiene un n´ umero infinito de representaciones, las cuales se pueden reducir a una que no tenga factores comunes en el numerador y denominador, en este caso se dice que la fracci´on algebraica racional est´a escrita en su forma irreducible. Ejemplo

1.

2.6.1 Determinar formas irreducibles para

3x2 + 11x − 20 6x2 + 7x − 20

Soluciones.

x2 + x − 6 2. (x + 1)(x2 − 4)

8 x2 + x − 1 3 3. 1 2 (x + )(x + x − 6) 2

134

Cap´ıtulo 2.

´ Algebra

1. Se factoriza cada polinomio y se simplifica 3x2 + 11x − 20 (x + 5)(3x − 4) x+5 = = . 2 6x + 7x − 20 (2x + 5)(3x − 4) 2x + 5 2. Se factoriza tanto el numerador como el denominador y se simplifica (x − 2)(x + 3) x+3 x2 + x − 6 = = . 2 (x + 1)(x − 4) (x + 1)(x − 2)(x + 2) (x + 1)(x + 2) 3. Al factorizar primero el denominador, se observa si alguno de sus factores es factor del numerador. As´ı se puede factorizar al numerador, luego 8 1 1 x2 + x − 1 (x − )(x + 3) x− 3 3 3 = = 1 2 1 1 (x + )(x + x − 6) (x + )(x + 3)(x − 2) (x + )(x − 2) 2 2 2 3x − 1 2(3x − 1) 3 = = (2x + 1)(x − 2) 3(2x + 1)(x − 2) 2 donde se observa que las tres u ´ltimas igualdades son formas equivalentes de escribir la forma irreducible de la fracci´on propuesta.  Un polinomio con coeficientes reales se dice irreducible si no puede ser factorizado como producto de polinomios de grado menor y con coeficientes reales. Un resultado fundamental para los polinomios con coeficientes reales, de gran utilidad en el manejo y simplificaci´on de las fracciones racionales, es el siguiente teorema. Teorema 2.6.1 Todo polinomio con coeficientes reales se puede factorizar en polinomios irreducibles de grado uno o dos, con coeficientes reales. Ejemplo

2.6.2 Las siguientes son factorizaciones irreducibles de los poli-

2.6. Operaciones con fracciones algebraicas

135

nomios dados x5 − 8x4 + 22x3 − 26x2 + 21x − 18 = (x − 3)2 (x − 2)(x2 + 1) x4 + 8x3 + 22x2 − 40x − 375 = (x + 5)(x − 3)(x2 + 6x + 25) x5 − x4 + 8x3 − 8x2 + 16x − 16 = (x2 + 4)2 (x − 1) x6 − 2x5 + 3x4 − 4x3 + 3x2 − 2x + 1 = (x − 1)2 (x2 + 1)2 x3 − 4x2 + 4x = x(x − 2)2 x4 − 2x3 + 2x2 − x = x(x − 1)(x2 − x + 1) 

2.6.2.

Multiplicaci´ on y divisi´ on de fracciones algebraicas

Como en los n´ umeros racionales se presenta primero la multiplicaci´on y la divisi´on de fracciones algebraicas racionales. p(x) r(x) El producto de y se define como q(x) s(x)

p(x) r(x) p(x)r(x) · = q(x) s(x) q(x)s(x) Observaci´ on 2.6.1 Si

p(x) ̸= 0, como p(x) ̸= 0, se tiene q(x) p(x)q(x) p(x) q(x) · = =1. q(x) p(x) q(x)p(x)

q(x) p(x) es el inverso multiplicativo de , en particular si p(x) p(x) q(x) es un polinomio diferente de cero entonces en el conjunto de las fracciones 1 algebraicas racionales, es su inverso multiplicativo, es decir p(x)

Esto dice que

(p(x))−1 =

1 . p(x)

136

Cap´ıtulo 2.

´ Algebra

El inverso multiplicativo de p(x) no es en general un polinomio. De id´entica manera si p(x), q(x) ̸= 0 ( )−1 p(x) q(x) = . q(x) p(x) Ejemplo

2.6.3 Calcular el producto de las siguientes fracciones racionales

algebraicas. 1.

3.

x − 5 x2 − 16 · x + 4 x2 − 25

2.

6x2 − 5x − 6 x2 − 4 · x2 + x − 6 4x2 − 9

x4 − 8x2 + 16 x2 + 10x + 25 3x2 + 11x − 20 x2 − 9 · 4. · x2 + 3x − 10 (x2 − 4)(x + 1) x2 − 5x + 6 6x2 + 7x − 20

Soluciones. 1. Por la definici´on de multiplicaci´on, se tiene x − 5 x2 − 16 x3 − 5x2 − 16x + 80 (x − 5)(x2 − 16) · 2 = . = x + 4 x − 25 (x + 4)(x2 − 25) x3 + 4x2 − 25x − 100 Si se factorizan los polinomios involucrados en el producto y se simplifica se obtiene x − 5 x2 − 16 (x − 5)(x2 − 16) (x − 5)(x + 4)(x − 4) = · 2 = 2 x + 4 x − 25 (x + 4)(x − 25) (x + 4)(x + 5)(x − 5) x−4 = . x+5 2. Primero se aplica la definici´on, luego se factorizan los polinomios y se simplifica 6x2 − 5x − 6 x2 − 4 (6x2 − 5x − 6)(x2 − 4) · = x2 + x − 6 4x2 − 9 (x2 + x − 6)(4x2 − 9) =

(3x + 2)(2x − 3)(x − 2)(x + 2) (x − 2)(x + 3)(2x − 3)(2x + 3)

=

(3x + 2)(x + 2) 3x2 + 8x + 4 = 2 (x + 3)(2x + 3) 2x + 9x + 9

2.6. Operaciones con fracciones algebraicas

137

2. Se observa que x4 − 8x2 + 16 = (x2 − 4)2 . Se procede a factorizar los polinomios faltantes de ambas fracciones y despu´es se simplifica x4 − 8x2 + 16 x2 + 10x + 25 (x2 − 4)2 (x + 5)2 · = · x2 + 3x − 10 (x2 − 4)(x + 1) (x − 2)(x + 5) (x2 − 4)(x + 1) =

(x2 − 4)2 (x + 5)2 (x − 2)(x + 5)(x2 − 4)(x + 1)

=

(x2 − 4)(x + 5) (x − 2)(x + 1)

=

(x − 2)(x + 2)(x + 5) (x − 2)(x + 1)

=

x2 + 7x + 10 x+1

3. Se factorizan los polinomios de ambas fracciones y se simplifica 3x2 + 11x − 20 x2 − 9 (x + 5)(3x − 4) (x − 3)(x + 3) · = · 2 2 x − 5x + 6 6x + 7x − 20 (x − 2)(x − 3) (2x + 5)(3x − 4) =

(x + 5)(3x − 4)(x − 3)(x + 3) · (x − 2)(x − 3)(2x + 5)(3x − 4)

=

(x + 5)(x + 3) x2 + 8x + 15 = 2 (x − 2)(2x + 5) 2x + x − 10

p(x) r(x) La divisi´ on de entre , cuando r(x) ̸= 0, se define como q(x) s(x)

p(x) r(x) p(x)s(x) ÷ = q(x) s(x) q(x)r(x) El cociente o la divisi´on tambi´en se escribe como p(x) p(x)s(x) q(x) = r(x) q(x)r(x) s(x)



138

Cap´ıtulo 2.

´ Algebra

y por tanto sigue v´alida la regla del emparedado. Ejemplo

2.6.4 Efectuar la divisi´on de las siguientes fracciones racionales

algebraicas x2 − 1 x3 − x ÷ x2 − 4 x+2 x4 − 8x2 + 16 x3 − 4x 2. ÷ x2 + 3x − 10 x2 − 4x + 4

1.

6x2 − 5x − 6 4x2 − 12x + 9 ÷ 2 x2 + x − 6 x + 5x + 6 3x2 − 27 x2 − 9 4. ÷ 2x2 + 3x − 5 6x2 + 7x − 20 3.

Soluciones. 1. Se aplica la definici´on de divisi´on x2 − 1 x3 − x (x2 − 1)(x + 2) x3 + 2x2 − x − 2 = . ÷ = x2 − 4 x+2 (x2 − 4)(x3 − x) x5 − 5x3 + 4x Se factorizan los polinomios involucrados en el producto del denominador y se simplifica x2 − 1 x 3 − x (x2 − 1)(x + 2) (x2 − 1)(x + 2) ÷ = = x2 − 4 x+2 (x2 − 4)(x3 − x) (x + 2)(x − 2)x(x2 − 1) =

1 . x−2

2. Se aplica la definici´on de la divisi´on, se factoriza y simplifica 6x2 − 5x − 6 4x2 − 12x + 9 (6x2 − 5x − 6)(x2 + 5x + 6) ÷ = x2 + x − 6 x2 + 5x + 6 (x2 + x − 6)(4x2 − 12x + 9) =

(3x + 2)(2x − 3)(x + 3)(x + 2) (x − 2)(x + 3)(2x − 3)2

=

(3x + 2)(x + 2) 3x2 + 8x + 4 = 2 (x − 2)(2x − 3) 2x − 7x + 6

3. Se usa la regla del cociente y la factorizaci´on x4 − 8x2 + 16 = (x2 − 4)2 .

2.6. Operaciones con fracciones algebraicas

139

Despu´es se factoriza y simplifica x4 − 8x2 + 16 x3 − 4x (x4 − 8x2 + 16)(x2 − 4x + 4) ÷ = x2 + 3x − 10 x2 − 4x + 4 (x2 + 3x − 10)(x3 − 4x) =

(x2 − 4)2 (x − 2)2 (x − 2)(x + 5)(x2 − 4)x

=

(x2 − 4)(x − 2) (x + 5)x

=

x3 − 2x2 − 4x + 8 x2 + 5x

4. Dividiendo, factorizando y simplificando se obtiene 3x2 − 27 x2 − 9 (3x2 − 27)(6x2 + 7x − 20) ÷ = 2x2 + 3x − 5 6x2 + 7x − 20 (2x2 + 3x − 5)(x2 − 9) =

3(x2 − 9)(2x + 5)(3x − 4) (2x + 5)(x − 1)(x2 − 9)

=

3(3x − 4) 9x − 12 = x−1 x−1

Ejercicio 2.6.1 Efectuar las operaciones siguientes y simplificar. 1.

x2 + x − 6 x2 − 4x + 3 · x2 − 2x − 3 x2 + 2x − 3

2.

x2 − 3x + 2 x2 + x − 6 ÷ x2 − 1 x2 − 2x − 3

3.

x2 + x − 20 x2 − x − 6 · x−3 x−4

2x2 + 3x − 2 8x3 − 12x2 + 6x − 1 4. ÷ x−2 x2 − 9x + 14 5.

5x2 + 3x − 2 3x2 − 5x + 2 · 2 3x − 2 5x + 8x − 4



140

Cap´ıtulo 2.

6.

3x2 + 5x − 2 3x2 + 8x − 3 ÷ 3x2 − 5x − 2 2x2 − 3x − 2

7.

16x4 − 81 x2 + 1 · x4 + 2x2 + 1 4x2 − 9

8.

4x2 + 9 16x4 − 81 ÷ x2 + 1 x4 + 2x2 + 1

9.

x4 − 5x2 + 4 x4 − 18x2 + 81 · x3 − 3x2 − 9x + 27 x3 − 2x2 − x + 2

10.

x3 + 2x2 − 9x − 18 x4 − 18x2 + 81 ÷ 3 x−3 x − 2x2 − x + 2

´ Algebra

Soluciones. 1.

x−2 x+1

2.

x2 − 5x − 14 4. 4x2 − 4x + 1 7.

10.

4x2 + 9 x2 + 1

x−3 x+3

x2 − 1 5. x+2 8.

3. x2 + 7x + 10 2x2 + 5x + 2 6. 3x2 + 10x + 3

x2 + 1 9. x2 + 5x + 6 4x2 − 9

x4 − 5x2 + 4 x3 − 3x2 − 9x + 27

2.6.3.

Suma de fracciones algebraicas

Como en la multiplicaci´on y la divisi´on de n´ umeros racionales, la suma de p(x) r(x) fracciones algebraicas racionales sigue la misma regla. Sean y . Se q(x) s(x) define la suma de estas dos fracciones algebraicas racionales como:

p(x) r(x) p(s)s(x) + r(x)q(x) + = q(x) s(x) q(x)s(x)

2.6. Operaciones con fracciones algebraicas

141

La definici´on de la suma muestra que los c´alculos se reducen a operaciones con polinomios, de la misma manera como la suma de racionales se reduce a operaciones entre enteros. Se destaca la importancia de la factorizaci´on de los polinomios para obtener el m´ınimo com´ un m´ ultiplo de los denominadores. Ejemplo

2.6.5 Calcular las siguientes sumas y diferencias. Simplificar a su m´ınima expresi´on. 1. 2. 3.

4.

5.

6x2

x−3 x+3 + 2 − 5x − 6 6x − 13x + 6

6x2

x−3 x+3 − 2 − 5x − 6 6x − 13x + 6

x+5 x−5 + 2x2 + 5x − 25 2x2 − 15x + 25

2x2

2x − 7 2x + 7 + 2 − 15x + 7 2x + 13x − 7

x2 − x − 12 x2 + x − 12 − (x2 − 4)(x − 3) (x − 2)(x2 + 5x + 6)

Soluciones. 1. Aplicando directamente la definici´on, se tiene

6x2

x−3 x+3 + 2 − 5x − 6 6x − 13x + 6 =

(x − 3)(6x2 − 13x + 6) + (x + 3)(6x2 − 5x − 6) (6x2 − 5x − 6)(6x2 − 13x + 6)

=

12x3 − 18x2 + 24x − 36 . 36x4 − 108x3 + 65x2 + 48x − 36

142

Cap´ıtulo 2.

´ Algebra

Otra manera de proceder es factorizar los denominadores y obtener el m´ınimo com´ un m´ ultiplo de ellos, as´ı 6x2

x+3 x−3 + 2 − 5x − 6 6x − 13x + 6 =

x−3 x+3 + (2x − 3)(3x + 2) (2x − 3)(3x − 2)

=

(x − 3)(3x − 2) + (x + 3)(3x + 2) (2x − 3)(3x + 2)(3x − 2)

=

6x2 + 12 (2x − 3)(3x + 2)(3x − 2)

=

6x2 + 12 18x3 − 27x2 − 8x + 12

Este segundo procedimiento permite escribir el resultado de una manera m´as simple. Se observa que la siguiente identidad justifica la igualdad de los dos c´alculos presentados ( ) 2x − 3 12x3 − 18x2 + 24x − 36 6x2 + 12 = · . 36x4 − 108x3 + 65x2 + 48x − 36 2x − 3 18x3 − 27x2 − 8x + 12 Se destaca que la expresi´on obtenida en la primera suma, no es en general, f´acil de simplificar. 2. Calculando directamente la diferencia de las fracciones se tiene 6x2

x−3 x+3 − 2 − 5x − 6 6x − 13x + 6 =

(x − 3)(6x2 − 13x + 6) − (x + 3)(6x2 − 5x − 6) (6x2 − 5x − 6)(6x2 − 13x + 6)

=

−44x2 + 66x 36x4 − 108x3 + 65x2 + 48x − 36

2.6. Operaciones con fracciones algebraicas

143

Otra manera, es obtener el m´ınimo com´ un m´ ultiplo de los denominadores y realizar la diferencia x−3 x+3 − (2x − 3)(3x + 2) (2x − 3)(3x − 2)

=

(x − 3)(3x − 2) − (x + 3)(3x + 2) (2x − 3)(3x + 2)(3x − 2)

=

−22x (2x − 3)(3x + 2)(3x − 2)

=

−22x 18x3 − 27x2 − 8x + 12

3. La suma est´a dada por x−5 x+5 4x3 − 10x2 + 100x − 250 + = 2x2 + 5x − 25 2x2 − 15x + 25 4x4 − 20x3 − 75x2 + 500x − 625 Sin embargo, al considerar el m´ınimo com´ un m´ ultiplo de los denominadores se tiene

2x2

x−5 x+5 x−5 x+5 + 2 = + + 5x − 25 2x − 15x + 25 (x + 5)(2x − 5) (x − 5)(2x − 5) =

(x − 5)2 + (x + 5)2 (x + 5)(x − 5)(2x − 5)

=

2x2 + 50 2x3 − 5x2 − 50x + 125

4. Se calcula la suma usando el m´ınimo com´ un m´ ultiplo de los denomina-

144

Cap´ıtulo 2.

´ Algebra

dores 2x + 7 2x − 7 + 2x2 − 15x + 7 2x2 + 13x − 7 =

2x − 7 2x + 7 + (2x − 1)(x − 7) (2x − 1)(x + 7)

=

(2x − 7)(x + 7) + (2x + 7)(x − 7) (2x − 1)(x − 7)(x + 7)

=

4x2 − 98 2x3 − x2 − 98x + 49

5. Factorizando se obtiene

x2 − x − 12 x2 + x − 12 − (x2 − 4)(x − 3) (x − 2)(x2 + 5x + 6)

=

(x + 3)(x − 4) (x − 3)(x + 4) − (x − 2)(x + 2)(x − 3) (x − 2)(x + 2)(x + 3)

=

(x + 3)2 (x − 4) − (x − 3)2 (x + 4) (x − 2)(x + 2)(x − 3)(x + 3)

=

4x2 − 72 x4 − 13x2 + 36 

Ejercicio 2.6.2 Efectuar las siguientes operaciones entre fracciones y sim-

2.6. Operaciones con fracciones algebraicas

145

plificar a su m´ınima expresi´on. 1.

x2 + 1 x2 − 1 + 2x2 + 3x − 2 2x2 + 5x + 2

2.

4x3 − 2 x2 + 1 − 4x3 + 8x2 − x − 2 2x2 + 5x + 2

3.

2x + 3 2x − 3 + 3x − 1 3x + 1

4.

12x2 + 6 2x + 3 − 9x2 − 1 3x − 1

5.

3x2 + 13x − 10 −3x2 − 13x + 10 + x−2 x+2

12x2 + 52x − 40 −3x2 − 13x + 10 6. − x2 − 4 x+2 7.

2x2 + 3x − 9 2x2 + 5x − 3 + 3x + 4 3x + 4

8.

2x2 + 3x − 9 2x2 + 5x − 3 − 3x + 4 3x + 4

9.

x2 − 21x + 62 x2 − x − 6 + x2 − 7x + 12 x−4

10.

x2 + x − 20 x2 − x − 6 − x−3 x−4

Soluciones. 4x3 − 2 1. 4x3 + 8x2 − x − 2 4.

2x − 3 3x + 1

x2 − 1 2. 2x2 + 3x − 2 5.

12x2 + 52x − 40 x2 − 4

12x2 + 6 3. 9x2 − 1 6.

3x2 + 13x − 10 x−2

146

Cap´ıtulo 2.

7.

4x2 + 8x − 12 3x + 4

8.

−2x − 6 3x + 4

9.

´ Algebra

x2 + x − 20 x−3

x2 − 21x + 62 10. x2 − 7x + 12

2.6.4.

Otras fracciones algebraicas.

En los ejemplos de la secci´on anterior se incluyeron u ´nicamente fracciones algebraicas racionales. En esta secci´on se consideran diversas fracciones algebraicas y no s´olo fracciones de polinomios de una variable. Por ejemplo, expresiones de la forma xy 2 − y 2 z , x2 z − yz 2 √

t+h− h



u3/2 w t

,

uv − vw , + uw − u1/2 v 1/2 w − v 1/2 w3/2

(t +

h)2/3

1 . + (t + h)1/3 t1/3 + t2/3

La manipulaci´on algebraica permite escribir una fracci´on algebraica en otra m´as simple. Ejemplo

2.6.6 Simplificar las siguientes fracciones algebraicas

1.

(100x − x2 ) − (100y − y 2 ) x−y

2.

z 2 − 3z + 2 2z 3 + 3z 2 − 8z − 12

3.

y 6 − 9y 3 + 8 y 3 − 6y 2 + 11y − 6

4.

x2 y + 2xy 2 + y 3 − x2 z − 2xyz − y 2 z xy 2 + y 3 − 2xyz − 2y 2 z + xz 2 + yz 2

5.

u−2 − v −2 u−1 − v −1

6.

x2 y 2 − y 2 z 2 x3 z + x2 z 2 − xyz 2 − yz 3

Soluciones. 1. Se eliminan par´entesis en el numerador, se agrupa, se factoriza y se

2.6. Operaciones con fracciones algebraicas

147

simplifica. 100x − x2 − 100y + y 2 (100x − x2 ) − (100y − y 2 ) = x−y x−y =

(100x − 100y) − (x2 − y 2 ) x−y

=

(x − y)(100 − (x + y)) x−y

= 100 − x − y 2. Se factoriza z 2 − 3z + 2 = (z − 1)(z − 2). Tambi´en 2z 3 + 3z 2 − 8z − 12 = 2z(z 2 − 4) + 3(z 2 − 4) = (2z + 3)(z − 2)(z + 2). Por lo tanto se tiene z 2 − 3z + 2 (z − 1)(z − 2) z−1 = = 2 . 2 3 2z + 3z − 8z − 12 (2z + 3)(z − 2)(z + 2) 2z + 7z + 6 3. La expresi´on y 6 − 9y 3 + 8 = (y 3 )2 − 9(y 3 ) + 8 con el cambio de variable t = y 3 se escribe como el polinomio cuadr´atico t2 −9t+8 = (t−1)(t−8), por lo cual se tiene la factorizaci´on y 6 − 9y 3 + 8 = (y 3 − 1)(y 3 − 8). De esta expresi´on se observa que 1 y 2 son ceros del numerador y se verifica que tambi´en son ceros del denominador. La factorizaci´on del denominador se obtiene al hacer las dos divisiones sint´eticas entre 1 y 2, y as´ı se tiene y 6 − 9y 3 + 8 (y 3 )2 − 9y 3 + 8 = y 3 − 6y 2 + 11y − 6 (y − 1)(y 2 − 5y + 6) =

(y 3 − 1)(y 3 − 8) (y − 1)(y − 2)(y − 3)

=

(y − 1)(y 2 + y + 1)(y − 2)(y 2 + 2y + 4) (y − 1)(y − 2)(y − 3)

=

(y 2 + y + 1)(y 2 + 2y + 4) . (y − 3)

148

Cap´ıtulo 2.

´ Algebra

4. Se agrupan t´erminos, se factoriza y se simplifica. x2 y + 2xy 2 + y 3 − x2 z − 2xyz − y 2 z xy 2 + y 3 − 2xyz − 2y 2 z + xz 2 + yz 2 =

x2 y − x2 z + 2xy 2 − 2xyz + y 3 − y 2 z xz 2 + yz 2 − 2xyz − 2y 2 z + xy 2 + y 3

=

(y − z)x2 + (2y 2 − 2yz)x + (y 3 − y 2 z) (x + y)z 2 − (2xy + 2y 2 )z + (xy 2 + y 3 )

=

(y − z)x2 + 2y(y − z)x + y 2 (y − z) (x + y)z 2 − 2y(x + y)z + y 2 (x + y)

=

(y − z)(x2 + 2xy + y 2 ) (x + y)(z 2 − 2zy + y 2 )

=

(y − z)(x + y)2 (x + y)(z − y)2

=

(y − z)(x + y)2 (x + y)(y − z)2

=

x+y y−z

5. Pasando a exponentes positivos se tiene

−2

−2

u −v u−1 − v −1

1 1 v 2 − u2 − 2 2 2 v 2 = u2 v 2 = uv(v − u ) = u v−u 1 1 u2 v 2 (v − u) − uv u v (v + u)(v − u) v+u = = . uv(v − u) uv

6. Se agrupan convenientemente los t´erminos para poder obtener las fac-

2.6. Operaciones con fracciones algebraicas

149

torizaciones, se factoriza y simplifica x2 y 2 − y 2 z 2 y 2 (x2 − z 2 ) = x3 z + x2 z 2 − xyz 2 − yz 3 z[x3 + x2 z − xyz − yz 2 ] =

y 2 (x − z)(x + z) z[x2 (x + z) − yz(x + z)]

=

y 2 (x − z)(x + z) z(x + z)(x2 − yz)

y 2 (x − z) = z(x2 − yz) =

xy 2 − y 2 z x2 z − yz 2

Los siguientes ejemplos abundan sobre la forma de realizar operaciones entre fracciones algebraicas. Ejemplo tado

(

1. ( 3. ( 5.

2.6.7 Realizar las siguientes operaciones y simplificar el resul-

2xz 2 x2 − 4z 2

)(

xy − xz yx2 − yz 2 2

2

36u2 − 1 12u + 6v

xz + 2z 2 3xz 3

)(

)

2.

xy − zy x2 y + x2 z

( ÷

)

2

2

)

(2 − 12u)(1 + 6u) 24u2 − 6v 2

2

7.

a + 3a a+1 a + − 2 a+2 4−a 3a − 6

9.

u2 − 4v 2 u2 + 2uv − 8v 2 + u2 − 2uv 4v 2 − u2

4. )

x+2 x+2 − x−4 x+4 1 1 − 2 2 (x + h) x h

x2 y xy 2 2xy 3 6. + − x + y x − y x2 − y 2 z 1−z 8. z 2+ 1+z 2−

10.

(a + y)2 (a + x)2 − 2 xy − y 2 x − xy

150

Cap´ıtulo 2.

´ Algebra

Soluciones. 1. (

2xz 2 x2 − 4z 2

)(

xz + 2z 2 3xz 3

)

2xz 2 (x + 2z)z 2 = 3 3xz (x − 2z)(x + 2z) 3(x − 2z)

=

2. x+2 x+2 (x + 2)(x + 4) − (x + 2)(x − 4) − = x−4 x+4 (x − 4)(x + 4) =

(x2 + 6x + 8) − (x2 − 2x − 8) x2 − 16

=

8x + 16 x2 − 16

3. (

xy 2 − xz 2 yx2 − yz 2

)(

xy 2 − zy 2 x2 y + x2 z

) =

x(y 2 − z 2 )(x − z)y 2 y(x2 − z 2 )x2 (y + z)

=

(y − z)(y + z)(x − z)y (x − z)(x + z)x(y + z)

=

(y − z)y (x + z)x

4. Se realiza primero la diferencia del numerador 1 1 − 2 2 (x + h) x h

=

x2 − (x + h)2 x2 − (x2 + 2xh + h2 ) (x + h)2 x2 (x + h)2 x2 = h h −

=

2xh + h2 −h(2x + h) 2x + h (x + h)2 x2 = =− 2 2 h h(x + h) x (x + h)2 x2

2.6. Operaciones con fracciones algebraicas

151

5. Se aplica la definici´on del cociente ) ( ) ( (36u2 − 1)(24u2 − 6v 2 ) 36u2 − 1 (2 − 12u)(1 + 6u) = ÷ 12u + 6v 24u2 − 6v 2 (12u + 6v)(2 − 12u)(1 + 6u) =

(6u − 1)(6u + 1)6(4u2 − v 2 ) 6(2u + v)2(1 − 6u)(1 + 6u)

= − =

(2u − v)(2u + v) 2(2u + v)

1 v−u 2

6. x2 y xy 2 2xy 3 x2 y(x − y) + xy 2 (x + y) − 2xy 3 + − 2 = x + y x − y x − y2 (x − y)(x + y) =

x3 y − x2 y 2 + x2 y 2 + xy 3 − 2xy 3 x2 − y 2

=

x3 y − xy 3 xy(x2 − y 2 ) = = xy x2 − y 2 x2 − y 2

7. a2 + 3a a + 1 a + − a+2 4 − a2 3a − 6 =

a a2 + 3a a+1 − 2 − a+2 a −4 3(a − 2)

=

3a(a − 2) − 3(a2 + 3a) − (a + 1)(a + 2) 3(a + 2)(a − 2)

=

a2 + 18a + 2 −a2 − 18a − 2 =− 3(a + 2)(a − 2) 3(a + 2)(a − 2)

=−

a2 + 18a + 2 3(a2 − 4)

152

Cap´ıtulo 2.

´ Algebra

8. Primero se realizan las operaciones en el numerador y en el denominador

z 1−z z 2+ 1+z

=

2(1 − z) − z −3z + 2 (1 + z)(−3z + 2) 1−z = 1−z = 3z + 2 2(1 + z) + z (1 − z)(3z + 2) 1+z 1+z

=

2 − z − 3z 2 3z 2 + z − 2 = 2 + z − 3z 2 3z 2 − z − 2

2−

9. Se observa que el numerador de la segunda fracci´on se puede reescribir como u2 + 2uv − 8v 2 = u2 + 2uv + v 2 − 9v 2 = (u + v)2 − 9v 2 , luego se tiene u2 − 4v 2 u2 + 2uv − 8v 2 + u2 − 2uv 4v 2 − u2 =

(u − 2v)(u + 2v) (u + v − 3v)(u + v + 3v) + u(u − 2v) (2v − u)(2v + u)

=

(u − 2v)(u + 2v) (u − 2v)(u + 4v) + u(u − 2v) (2v − u)(2v + u)

=

u + 2v u + 4v − u 2v + u

=

(u + 2v)(2v + u) − (u + 4v)u u(2v + u)

=

4v 2 u(u + 2v)

2.6. Operaciones con fracciones algebraicas

153

10. (a + y)2 (a + x)2 (a + y)2 (a + x)2 − = − xy − y 2 x2 − xy y(x − y) x(x − y) =

x(a + y)2 − y(a + x)2 xy(x − y)

=

x(a2 + 2ay + y 2 ) − y(a2 + 2ax + x2 ) xy(x − y)

=

xa2 + xy 2 − ya2 − yx2 xy(x − y)

=

a2 (x − y) − xy(x − y) xy(x − y)

=

a2 − xy a2 = −1 xy xy

Ejercicio 2.6.3 Realizar las operaciones indicadas y simplificar

x−2 − y −2 1. −4 x − y −4

2.

2 −a a+1 1 1 + 2 a−1 a −1

3.

x2 − 1 (x4 )4 − x4 x8

4.

x9 + 2x6 − 3x3 x9 − x 3

5.

6u2 − 13u + 6 3u3 − 5u2 + 2u

6.

y 2 − 3y + 2 y 4 − 5y 2 + 4

7.

y 2 − 8y + 15 3y 2 − 18y + 27

8.

(x2 + 2xy + y 2 )(x − z) (x2 − 2xz + z 2 )(x + y)

9. 1 +

1 1 1+ u

1

10. 1 + 1+

1 1+

1 u

154

Cap´ıtulo 2.

´ Algebra

11.

22y+z − 2y+2z 2y−x − 2z−x

12.

2y+2z − 22y+z 22x+y − 22x+z

13.

2x − 3 x + 3 − x−1 x+1 x x+1

14.

1 1 + a−b 1−u 1 − ub−a

x−y y−z 15. ÷ 2 yz + yx − xz − x −yz − yx + xz + z 2

17.

5 5 − 3(x + h) 3x h

a2 + ab + b2 16. a b − 2 2 b a 4ab + 4b2 a2 − b2 2b 1− a+b

2+ 18.

7 5u − 3 20. 7u − 7 5u + u−2

√ √ √ x+h+ x 19. h √ √ − 2 x2 + hx x+h− x

u−

Soluciones. 1.

x2 y 2 y 2 + x2

2. −(a − 1)2

5.

2u − 3 u2 − u

6.

9.

2u + 1 u+1

10.

13.

x−3 x−1

14. 1

5 17. − 3(x + h)x

y2

1 + 3y + 2

3u + 2 2u + 1

( 18. 2

a+b a−b

3.

1 14 x + x12

4.

x3 + 3 x3 + 1

7.

y−5 3y − 9

8.

x+y x−z 2y+z 22x

11. 2x+y+z

12. −

15. 1

16.

a2 b2 a−b

19. 2x + h

20.

u−2 5u − 3

)2

2.7. Racionalizaci´ on

2.7.

155

Racionalizaci´ on

La multiplicaci´on por el binomio conjugado es u ´til para racionalizar algunas expresiones algebraicas, esto significa, eliminar una expresi´on radical del numerador y/o el denominador. En la simplificaci´on se aplican los procesos de factorizaci´on. Ejemplo √ 1.

2.7.1 Racionalizar y simplificar las expresiones

x2 + 7 − 4 3−x

2+x 2. √ 8x2 + 4 + 3x

√ 3.

8x2 + 2 − 2x − 1 2x2 + 5x − 3

√ √ x+3− 5 4. √ √ 5 − 2x + 1 Soluciones. √ √ 1. El numerador x2 + 7−4 tiene como binomio conjugado a x2 + 7+4. Se multiplica tanto al numerador como al denominador de la fracci´on por este binomio conjugado; se factoriza y se simplifica. √

√ x2 + 7 − 4 x2 + 7 + 4 ·√ 3−x x2 + 7 + 4 √ ( x2 + 7)2 − 16 x2 + 7 − 16 √ √ = = (3 − x)( x2 + 7 + 4) (3 − x)( x2 + 7 + 4) x2 − 9 (x − 3)(x + 3) √ √ = = (3 − x)( x2 + 7 + 4) (3 − x)( x2 + 7 + 4) x+3 . = −√ x2 + 7 + 4

x2 + 7 − 4 = 3−x



√ 2. El denominador 8x2 + 4 + 3x tiene como binomio conjugado a la √ 2 expresi´on 8x + 4 − 3x. Se multiplica tanto al numerador como al denominador de la fracci´on por este binomio conjugado; se factoriza y

156

Cap´ıtulo 2.

´ Algebra

se simplifica. √ 2+x 2+x 8x2 + 4 − 3x √ = √ ·√ 8x2 + 4 + 3x 8x2 + 4 + 3x 8x2 + 4 − 3x √ √ (2 + x)( 8x2 + 4 − 3x) (2 + x)( 8x2 + 4 − 3x) √ = = 8x2 + 4 − 9x2 ( 8x2 + 4)2 − (3x)2 √ √ (2 + x)( 8x2 + 4 − 3x) (2 + x)( 8x2 + 4 − 3x) = = 4 − x2 (2 − x)(2 + x) √ 2 8x + 4 − 3x . = 2−x 3. El numerador se puede escribir como √

8x2 + 2 − 2x − 1 =



8x2 + 2 − (2x + 1)

y su binomio conjugado es entonces √

8x2 + 2 + (2x + 1) .

Se multiplica tanto al numerador como al denominador de la fracci´on por este binomio conjugado; se factoriza y se simplifica. √

8x2 + 2 − 2x − 1 = 2x2 + 5x − 3 = = = = =



√ 8x2 + 2 − (2x + 1) 8x2 + 2 + (2x + 1) √ · 2x2 + 5x − 3 8x2 + 2 + (2x + 1) √ ( 8x2 + 2)2 − (2x + 1)2 (√ ) (2x2 + 5x − 3) 8x2 + 2 + (2x + 1) 8x2 + 2 − (4x2 + 4x + 1) (√ ) (2x2 + 5x − 3) 8x2 + 2 + (2x + 1) 4x2 − 4x + 1 (√ ) (2x2 + 5x − 3) 8x2 + 2 + (2x + 1) (2x − 1)2 (√ ) (2x − 1)(x + 3) 8x2 + 2 + (2x + 1) 2x − 1 (√ ) . (x + 3) 8x2 + 2 + (2x + 1)

2.7. Racionalizaci´ on

157

√ √ 4. Para racionalizar el numerador √ x + 3 − 5 se multiplica la fracci´on √ x+3+ 5 √ y para racionalizar el denominador dada por la fracci´on √ x + 3 + 5 √ √ √ √ 5 − 2x + 1 se multiplica por la fracci´on 5 + 2x + 1. As´ı √ √ √ √ √ √ √ √ x+3− 5 x+3− 5 x+3+ 5 5 + 2x + 1 √ √ ·√ = √ ·√ √ √ √ 5 − 2x + 1 5 − 2x + 1 x+3+ 5 5 + 2x + 1 √ √ √ √ √ √ ( x + 3 − 5)( x + 3 + 5) 5 + 2x + 1 √ √ = √ · √ √ √ ( 5 − 2x + 1)( 5 + 2x + 1) x+3+ 5 √ √ √ √ ( x + 3)2 − ( 5)2 5 + 2x + 1 √ · √ = √ √ ( 5)2 − ( 2x + 1)2 x+3+ 5 √ √ 5 + 2x + 1 x+3−5 √ · √ = 5 − (2x + 1) x+3+ 5 √ √ x−2 5 + 2x + 1 √ = · √ −2(x − 2) x+3+ 5 √ √ 5 + 2x + 1 √ = − √ 2( x + 3 + 5)  Ejercicio 2.7.1 Racionalizar y reducir las siguientes fracciones √ 1.

√ 2x + 3 − x x2 + 4x + 3



3x2 − 3x − 2 − x − 1 6x2 − 7x − 5 √ √ 9x2 + 4 − 36x2 + 4 4. x3 + 2x2 2.

x2 + 5x − 6 3. √ 8x + 1 − 3 √ √ √ 2x + 3 − 3 5x2 − 2x + 3 − 4x2 + 7x + 3 5. 3 6. x − 4x2 + 4x − 3 x √ √ x2 + 9 − 5 3x2 − 2x + 2x 7. √ 8. √ . −9x − 2 − 4 x2 − 7 − 3

158

Cap´ıtulo 2.

´ Algebra

Soluciones. 1.

1 √ √ (x + 1)( 2x + 3 + x)

√ (x + 6)( 8x + 1 + 3) 3. 8 5.

2 √ (x2 − x + 1)( 2x + 3 + 3)

√ x2 − 7 + 3 7. √ x2 + 9 + 5

2.

x−3 √ (3x − 5)( 3x2 − 3x − 2 + x + 1)

4. −

27 √ √ 2 (x + 2)( 9x + 4 + 36x2 + 4)

6. √

x−9 √ 5x2 − 2x + 3 + 4x2 + 7x + 3

√ x( −9x − 2 + 4) √ 8. 9( 3x2 − 2x − 2x)

2.8. Ecuaciones de primer grado

2.8.

159

Ecuaciones de primer grado

La soluci´on de ecuaciones es, quiz´a, el objetivo primordial del ´algebra elemental y es el fundamento sobre el cual se desarroll´o el ´algebra. Esta secci´on estudia los m´etodos de soluci´on para ecuaciones de primer grado con una y dos inc´ognitas.

2.8.1.

Ecuaciones de primer grado con una inc´ ognita

Un viejo planteamiento algebraico dice: Un gavil´an al ver volar unas palomas dijo -Adi´ os mis cien palomas- a lo que una de las palomas contest´o -No somos cien palomas, pues nosotras, m´as otras tantas, m´as la mitad de nosotras, m´as la cuarta parte de nosotras y usted seremos cien-. ¿Cu´antas eran las palomas? Se puede resolver este problema por tanteo, sin embargo se puede determinar su soluci´on al plantear y resolver una ecuaci´on. M´as precisamente, sea x el n´ umero de palomas que hay, entonces: x son la mitad de las palomas y 2 x son la cuarta parte de las palomas. 4 As´ı, el enunciado se puede expresar algebraicamente de la manera siguiente: x x x + x + + + 1 = 100 . (2.8.5) 2 4 Reduciendo t´erminos semejantes se tiene 11 x + 1 = 100 , 4 de donde:

11 x = 99 4

´o sea

11x = 99 (4) .

Es decir, 396 = 36 . 11 As´ı, eran 36 palomas. A la expresi´on (2.8.5) se le llama ecuaci´on y al n´ umero 36 se le dice soluci´on de la ecuaci´on, conceptos que a continuaci´on se precisan. 11x = 396

y despejando x =

160

Cap´ıtulo 2.

´ Algebra

Se llama ecuaci´ on a una igualdad en la que hay una o m´as cantidades desconocidas, denominadas inc´ ognitas. Se suele denotar a las inc´ognitas con literales. Ejemplos de ecuaciones son: 2x − 3 = x + 5 , x2 + y 2 = 25 ,

x2 + 4x − 7 = 0 , x2 = 4y ,

x − 3y = 61 , √

x2 + 1 − 3 = y ,

donde se observa que son igualdades con una o m´as inc´ognitas. Un n´ umero a se dice soluci´ on de una ecuaci´on con una inc´ognita, si al sustituir a la inc´ognita por el n´ umero a en la ecuaci´on, ´esta se reduce a una identidad. Ejemplo

2.8.1 Comprobar que x = 1 es soluci´on de la ecuaci´on 2x − 1 = 6x − 5 .

Soluci´on. Sustituyendo x = 1 en la ecuaci´on propuesta se tiene 2(1) − 1 = 6(1) − 5 2−1=6−5 1=1 

que es una identidad. Ejemplo

2.8.2 De los n´ umeros −2, 1 y 3, determinar aqu´ellos que son soluciones de la ecuaci´on x2 − x − 6 = 0 . Soluci´on. Sustituyendo x = −2 en la ecuaci´on se tiene: (−2)2 − (−2) − 6 = 0 4+2−6=0 0=0,

2.8. Ecuaciones de primer grado

161

luego x = −2 es soluci´on de la ecuaci´on. Se sustituye ahora x = 1 en la ecuaci´on y se obtiene (1)2 − (1) − 6 = 0 1−1−6=0 −6 = 0 , que no es una identidad, por lo tanto x = 1 no es soluci´on de la ecuaci´on. Sustituyendo x = 3 en la ecuaci´on se tiene (3)2 − 3 − 6 = 0 9−3−6=0 9−9=0 0=0, luego x = 3 es soluci´on de la ecuaci´on.  Para obtener la soluci´on de una ecuaci´on se usan las siguientes propiedades de los n´ umeros reales.

Si a = b entonces a + c = b + c . Es decir, si a cantidades iguales se les suma el mismo n´ umero los resultados son iguales.

Si a = b entonces ac = bc . Esto es, si a cantidades iguales se les multiplica por un mismo n´ umero los productos son iguales. Las ecuaciones en las que el exponente de la inc´ognita es uno se llaman ecuaciones lineales o de primer grado. Los siguientes ejemplos muestran como resolver este tipo de ecuaciones. Ejemplo

2.8.3 Resolver la ecuaci´on 4x = 7 − 3x .

162

Cap´ıtulo 2.

´ Algebra

Soluci´ on. Por la primera propiedad se puede sumar 3x en ambos lados de la ecuaci´on conservando la igualdad. As´ı 4x + 3x = 7 − 3x + 3x 7x = 7 .

reduciendo t´ erminos semejantes

1 Por la segunda propiedad, se multiplica por ambos lados de la ecuaci´on 7 conservando la igualdad. ( ) 1 1 (7x) = 7 7 7 x=1. Se comprueba el resultado sustituyendo x = 1 en la ecuaci´on 4x = 7 − 3x, as´ı 4(1) = 7 − 3(1) 4=7−3 4=4. Por lo tanto, x = 1 es la soluci´on de la ecuaci´on. Ejemplo



2.8.4 Encontrar la soluci´on de la ecuaci´on 7 − x = 23 − 5x .

Soluci´on. Se suma 5x en ambos lados de la ecuaci´on (primera propiedad) y se obtiene 7 − x + 5x = 23 − 5x + 5x 7 + 4x = 23 . Ahora se suma −7 en ambos lados de la ecuaci´on (primera propiedad). −7 + 7 + 4x = −7 + 23 4x = 16 .

2.8. Ecuaciones de primer grado

163

1 Al multiplicar por en ambos lados de la ecuaci´on (segunda propiedad), se 4 obtiene la soluci´on 1 1 (4x) = (16) 4 4 x=4. De aqu´ı en adelante se deja la comprobaci´on de la soluci´on al lector.  Las propiedades anteriores com´ unmente se usan de la siguiente forma

a + c = b si y s´olo si a = b − c .

ac = b si y s´olo si a =

b , con c ̸= 0 . c

Es decir se aplican las conocidas reglas “lo que est´a sumando pasa restando”y “lo que est´a multiplicando pasa dividiendo”. Ejemplo

2.8.5 Hallar la soluci´on de la ecuaci´on 2x − (x + 5) = 6x .

Soluci´ on. 2x − (x + 5) 2x − x − 5 x−5 x

= 6x = 6x = 6x = 6x + 5 .

quitando par´ entesis reduciendo t´ erminos semejantes sumando 5 en ambos lados

Se observa que ya no se escribi´o x − 5 + 5 = 6x + 5. Ahora sumando −6x en ambos lados de la ecuaci´on se obtiene: −6x + x = 5 −5x = 5 x = −1 .

reduciendo t´ erminos semejantes ( ) 1 multiplicando por −5

Por lo tanto x = −1 es la soluci´on de la ecuaci´on.



164

Ejemplo

Cap´ıtulo 2.

´ Algebra

2.8.6 Resolver la ecuaci´on 4(3x + 1) − 6(x + 1) = −2(x − 3) + 6(x − 2) .

Soluci´on. Quitando par´entesis 12x + 4 − 6x − 6 = −2x + 6 + 6x − 12 . Reduciendo t´erminos 6x − 2 = 4x − 6 6x − 4x = −6 + 2 2x = −4 x = −2 .  Hay ecuaciones que se pueden reducir a una lineal. En este caso la soluci´on de la ecuaci´on lineal, no siempre es soluci´on de la ecuaci´on original, por ello se debe verificar directamente que el valor obtenido satisfaga la ecuaci´on original. Ejemplo

2.8.7 Resolver la ecuaci´on 1 2 − 2 =0 x−1 x −1

Soluci´on. Se observa que en la ecuaci´on se debe tener x ̸= ±1, pues en estos valores x2 − 1 = 0. Al multipicar la ecuaci´on por x2 − 1 se tiene ( ) 1 2 2 (x − 1) − =0 x − 1 x2 − 1 la cual reduce a la ecuaci´on lineal x+1−2=0 y al despejar se tiene x = 1, que no es soluci´on de la ecuaci´on propuesta.  Ejemplo

2.8.8 Resolver la ecuaci´on (x − 3)2 − (2 − x)2 = x − 1

2.8. Ecuaciones de primer grado

165

Soluci´ on. Desarrollando los binomios al cuadrado x2 − 6x + 9 − (4 − 4x + x2 ) = x − 1 . Quitando par´entesis x2 − 6x + 9 − 4 + 4x − x2 = x − 1 . Reduciendo t´erminos semejantes −2x + 5 = x − 1 5 + 1 = x + 2x 6 = 3x 6 =x 3 x=2.

 Una manera de resolver ecuaciones con fracciones es cancelando los denominadores y esto se logra multiplicando la ecuaci´on por el m´ınimo com´ un denominador (m.c.d.). Los siguientes ejemplos ilustran esta situaci´on. Ejemplo

2.8.9 Hallar la soluci´on de la ecuaci´on 2x +

1 2 =4+ x . 3 3

Soluci´ on. En esta ecuaci´on el m.c.d es 3, as´ı que multiplicando la ecuaci´on por 3 se tiene: ( ( ) ) 2 1 3 2x + =3 4+ x 3 3 6x + 2 = 12 + x 6x − x = 12 − 2 5x = 10 10 x= 5  x=2.

166

Cap´ıtulo 2.

Ejemplo

´ Algebra

2.8.10 Resolver la ecuaci´on 1 1 1 1 + + = . 2x x 3 6x

Soluci´ on. Multiplicando la ecuaci´on por el m.c.d. que es 6x ( ) ( ) 1 1 1 1 + + 6x = 6x 2x x 3 6x 6x 6x 6x 6x + + = 2x x 3 6x 3 + 6 + 2x = 1 9 + 2x = 1 2x = −8 x = −4 . Ejemplo



2.8.11 Resolver la ecuaci´on 3 4 5 − 2 = . x−3 x −9 x+3

Soluci´on. Multiplicando por el m.c.d. x2 − 9 = (x + 3)(x − 3) ( ) ( ) 5 3 4 2 2 (x − 9) − = (x − 9) x − 3 x2 − 9 x+3 ( 2 ) 2 x −9 (x − 9) 5(x + 3)(x − 3) 3 −4 2 = x−3 x −9 x+3 3(x + 3) − 4 = 5(x − 3) 3x + 9 − 4 = 5x − 15 3x + 5 = 5x − 15 3x − 5x = −15 − 5 −2x = −20 x = 10 . Ejercicio 2.8.1 Resolver las siguientes ecuaciones. 1. 7x − 11 = −5x − 17



2.8. Ecuaciones de primer grado

2. 4 − 6x = 4x − 16 3. 3x − (2x + 5) = 8x + (−2x + 5) 4. 6x − (3x + 8) = 5x − (4 − 6x) 5. (2 − 5x) − (7 − 3x) = (8x + 5) − (−4 + 3x) 6. 2x − (5 + 4x − (3 − 2x)) = 8x − (6x + 2) 7. 9x − (5x + 2) − (7x + 4 − (x − 2)) = 0 8. 10(3 − x) + 4(−x + 5) = 3(8x − 20) − 4 9. 5x + 3(x − 2) = 4 − 2(3x + 7) 10. 3(3x − 2) − 2(5 − 2x) = 4(1 − 3x) − 5(−2 − 2x) 11.

x 1 x −4= + 3 6 2

12. 3 −

x x 5 = − 12 4 6

13.

x+1 x+2 x+3 + = 2 3 4

14.

1 1 1 (x − 3) − (x − 2) = (x − 4) 2 3 4

15.

3 1 22x − 4 x − (x − 5) − (1 − 2x) = 4 3 24

16. x(2x − 7) − 2(x + x2 ) = 20 − x 17. (5x − 2)(1 − 7x) = 7(x − 2)(3 − 5x) 18. (x − 3)2 − (4 − x)2 = 3 − 8x 19. (x − 2)3 − (x + 2)3 = (3x − 1)(2 − 4x)

167

168

Cap´ıtulo 2.

´ Algebra

Soluciones. 1 2

2. x = 2

3. x = −2

4. x = −

5. x = −2

6. x = 0

7. x = −4

8. x = 3

9. x = −

2 7

10. x = 2

11. x = 27

13. x = −

5 7

14. x = 2

15. x = −

5 9

17. x = −

5 9

18. x = 1

19. x = −

7 5

1. x = −

12. x =

1 2

23 2

16. x = −

5 2

Ejercicio 2.8.2 1. Despejar t de la ecuaci´on v = v0 + at 1 2. Despejar a de la ecuaci´on x = x0 + vt + at2 2 3. Despejar R de la ecuaci´on 4. Despejar x de y =

1 1 1 = + R R1 R2

2x + 1 3x − 4

Soluciones. v − v0 2(x − x0 − vt) 2. a = a t2 1 + 4y 4. x = 3y − 2

1. t =

2.8.2.

3. R =

R1 R2 R1 + R2

Sistemas de dos ecuaciones de primer grado con dos inc´ ognitas

Una ecuaci´ on de primer grado con dos inc´ ognitas es una expresi´on de la forma: ax + by = c ,

2.8. Ecuaciones de primer grado

169

con a, b, c, n´ umeros reales a ̸= 0, b ̸= 0 . Son ejemplos de ecuaciones de primer grado con dos inc´ognitas: 1 −3x + y = −9 . 2 Se dice que los n´ umeros x = p, y = q son una soluci´ on de la ecuaci´ on ax + by = c si al sustituirlos en la ecuaci´on, ´esta se reduce a una identidad, es decir, se cumple que ap + bq = c . 2x + 3y = 5 ,

Ejemplo

x − 4y = 8 ,

2.8.12 Comprobar que x = 1, y = 1 son una soluci´on de la

ecuaci´on 2x + 3y = 5 . 5 Adem´as, verificar que x = 0, y = son tambi´en soluci´on de la misma 3 ecuaci´on. Soluci´ on. Se sustituyen x = 1, y = 1 en la ecuaci´on dada 2(1) + 3(1) = 5 2+3=5 5=5 y la ecuaci´on se ha reducido a una identidad, luego los valores dados son una soluci´on de la ecuaci´on. 5 Se sustituyen ahora x = 0, y = en la ecuaci´on: 3 2x + 3y = 5 ( ) 5 y se tiene que: 2(0) + 3 =5 3 0+5=5 5=5 que es una identidad. As´ı tambi´en son una soluci´on de la ecuaci´on.  Se llama sistema de dos ecuaciones de primer grado con dos inc´ ognitas al arreglo ax + by = c dx + ey = f . Se dice que x = p, y = q son una soluci´ on del sistema si p, q son una soluci´on de ambas ecuaciones.

170

Cap´ıtulo 2.

´ Algebra

Ejemplo

2.8.13 Comprobar que x = 1, y = 2, son una soluci´on del sistema de ecuaciones: 5x − 2y = 1 x + 3y = 7 . Soluci´on. Se sustituye x = 1, y = 2 en ambas ecuaciones. Sustituyendo en la primera ecuaci´on: 5x − 2y = 1 5(1) − 2(2) = 1 5−4=1 1=1. Ahora, sustituyendo en la segunda ecuaci´on x + 3y = 7 1 + 3(2) = 7 1+6=7 7=7. Como x = 1, y = 2 son una soluci´on de ambas ecuaciones, entonces son una soluci´on del sistema de ecuaciones.  Ejemplo

2.8.14 Decidir si x = 3, y = −1 son una soluci´on del sistema de ecuaciones: 4x + 3y = 9 2x − y = 5 . Soluci´on. Se sustituye x = 3, y = −1 en ambas ecuaciones. Al sustituir en la primera se tiene 4x + 3y = 9 4(3) + 3(−1) = 9 12 − 3 = 9 9=9.

2.8. Ecuaciones de primer grado

171

Ahora, sustituyendo en la segunda ecuaci´on 2x − y = 5 2(3) − (−1) = 5 6+1=5 7=5, lo cual es falso, por lo tanto, x = 3, y = −1 no es una soluci´on del sistema de ecuaciones, ya que s´olo es soluci´on de una de las ecuaciones del sistema y no de ambas. 

2.8.3.

M´ etodo de sustituci´ on

En esta secci´on se presenta un m´etodo para resolver sistemas de dos ecuaciones de primer grado con dos inc´ognitas.

M´etodo de sustituci´on Se despeja una inc´ognita de una de las ecuaciones y se sustituye en la otra ecuaci´on. Ejemplo

2.8.15 Resolver el sistema de ecuaciones: 2x + 3y = 7 5x − 2y = 8 .

Soluci´ on. Despejando x de la primera ecuaci´on se tiene: 2x + 3y = 7 2x = 7 − 3y 7 − 3y x= ; 2 sustituyendo la expresi´on obtenida para x en la segunda ecuaci´on 5x − 2y = 8 ) 7 − 3y 5 − 2y = 8 2 35 − 15y − 2y = 8 2 (

172

Cap´ıtulo 2.

´ Algebra

y resolviendo esta ecuaci´on: ( ) 35 − 15y 2 − 2y = 2(8) 2 35 − 15y − 4y = 16 35 − 19y = 16 −19y = −19 y=1. Ahora, tomando y = 1 en la expresi´on de la inc´ognita despejada se tiene 7 − 3y 7 − 3(1) 7−3 4 = = = =2. 2 2 2 2 As´ı, x = 2, y = 1 es la soluci´on del sistema de ecuaciones. El m´etodo garantiza que solamente hay una soluci´on para este sistema de ecuaciones, es decir, la soluci´on es u ´nica.  x=

Ejemplo

2.8.16 Resolver el siguiente sistema de ecuaciones: − 4x + 3y = − 1 5x + y = 6.

Soluci´on. Se despeja y de la segunda ecuaci´on: y = 6 − 5x . sustituyendo la expresi´on obtenida para y en la primera ecuaci´on −4x + 3(6 − 5x) = −1 −4x + 18 − 15x = −1 −19x = −19 x=1 sustituyendo x = 1 en y = 6 − 5x y = 6 − 5(1) = 6 − 5 = 1 . Luego x = 1, y = 1 es la soluci´on del sistema dado. El m´etodo garantiza que solamente hay una soluci´on para este sistema de ecuaciones, es decir, la soluci´on es u ´nica.  Los ejemplos anteriores muestran que se puede despejar x ´o y, es decir cualquiera de las dos inc´ognitas.

2.8. Ecuaciones de primer grado

Ejemplo

173

2.8.17 Resolver el siguiente sistema de ecuaciones: 7x − 9y = 6 14x − 18y = 5 .

Soluci´ on. Despejando x de la primer ecuaci´on: 7x = 6 + 9y 6 + 9y x= . 7 Sustituyendo la expresi´on obtenida para x en la segunda ecuaci´on se tiene: ( ) 6 + 9y 14 − 18y = 5 7 2(6 + 9y) − 18y = 5 12 + 18y − 18y = 5 12 = 5 , lo cual es falso. Se dice entonces que el sistema no tiene soluci´on.  El ejemplo anterior muestra que hay sistemas que no tienen soluci´on. Ejemplo

2.8.18 Resolver el siguiente sistema de ecuaciones: 2x − 3y = 6 −4x + 6y = − 12 .

Soluci´ on. Despejando x de la primer ecuaci´on: 2x = 6 + 3y 6 + 3y x= . 2 Sustituyendo la expresi´on obtenida para x en la segunda ecuaci´on se tiene: ( ) 6 + 3y −4 + 6y = −12 2 −2(6 + 3y) + 6y = −12 −12 − 6y + 6y = −12 0=0,

174

Cap´ıtulo 2.

´ Algebra

es decir, se obtiene una identidad para cualquier valor de y. Este sistema tiene una infinidad de soluciones, pues dando cualquier valor real a la variable y se 9 calcula el correspondiente valor de x. Por ejemplo x = 3, y = 0; x = , y = 1; 2 3 x = , y = −1 son algunas soluciones.  2

Ejercicio 2.8.3 Mediante el m´etodo de sustituci´on, resolver los siguientes sistemas de ecuaciones.

1.

6x + y = − 2 5x + 3y = 7

2.

9x + 4y = − 22 7x − 2y = − 12

3.

3x − 8y = − 2 4x + 3y = 11

4.

7x + 4y = 2 1 x + 3y = − 8 2

5.

4x + 5y = 65 8x − 11y = − 59

6.

5x + 3y = 9 2x − 4y = 14

7.

13x − 4y = 38 7x + 9y = − 13

8.

−2x + 5y = − 7 4x − 3y = 7

9.

11x + 2y = 10 3x + 8y = − 42

10.

9x − 4y = − 26 12x + 5y = − 14

11.

6x − 2y = − 3 8x + 3y = 13

12.

3x − 5y = − 3 4 2x + y = − 15

13.

1 5 x − y = 6 4 3 3x + 2y = 6

14.

13 4 3x − 2y = − 1 3x + 7y = −

2.8. Ecuaciones de primer grado

175

Soluciones. 1. x = −1, y = 4

2. x = −2, y = −1

3. x = 2, y = 1

4. x = 2, y = −3

5. x = 5, y = 9

6. x = 3, y = −2

7. x = 2, y = −3

8. x = 1, y = −1

9. x = 2, y = −6

10. x = −2, y = 2

1 11. x = , y = 3 2

13. x = 4, y = −3

1 1 14. x = − , y = − 2 4

2.8.4.

1 2 12. x = − , y = 3 5

M´ etodo de suma o resta

Como se puede apreciar en la secci´on anterior, resolver un sistema de dos ecuaciones de primer grado con dos inc´ognitas es sencillo, aunque en algunos casos resulta laborioso. Hay otros m´etodos de soluci´on como es el denominado de suma o resta. Una ecuaci´on ax + by = c se dice equivalente a αx + βy = γ si existe t ̸= 0 tal que ta = α, tb = β, tc = γ. Ecuaciones equivalentes tienen las mismas soluciones.

M´etodo de suma o resta Se obtiene un sistema de ecuaciones equivalentes tal que al sumar las ecuaciones se elimina una inc´ognita. Los siguientes ejemplos muestran el procedimiento. Ejemplo

2.8.19 Resolver el sistema de ecuaciones x + 5y = − 3 2x + 3y = 8.

Soluci´ on. Se desea eliminar la inc´ognita x, para ello los coeficientes de x deben tener signo contrario y el mismo valor absoluto. En la segunda ecuaci´on el coeficiente de x es 2, por ello se necesita que el coeficiente de x en la primera

176

Cap´ıtulo 2.

´ Algebra

ecuaci´on sea −2. Esto se logra multiplicando la primera ecuaci´on por −2. El sistema equivalente es −2(x + 5y = − 3) 2x + 3y = 8

;

−2x − 10y = 2x + 3y =

6 8

Se suman ahora las dos ecuaciones del sistema y se despeja y − 2x − 10y = 2x + 3y = 0 − 7y =

6 8 14 14 y = − 7 y = − 2

sustituyendo y = −2 en la primera ecuaci´on del sistema x + 5(−2) = −3 x − 10 = −3 x = −3 + 10 x=7. Por lo tanto, la soluci´on del sistema es x = 7; y = −2 . Ejemplo



2.8.20 Resolver el sistema de ecuaciones 15x + 4y = 3 5x + 3y = − 4 .

Soluci´on. El coeficiente de x en la primera ecuaci´on es 15 y en la segunda ecuaci´on deber´a ser −15 para poder eliminar la inc´ognita x. As´ı se multiplica por −3 la segunda ecuaci´on para obtener 15x + 4y = 3 −3(5x + 3y = − 4)

;

15x + 4y = 3 −15x − 9y = 12

Se suman ahora las dos ecuaciones del sistema para obtener el valor de y

2.8. Ecuaciones de primer grado

177

15x + 4y = 15x − 9y = 0 − 5y =

3 12 15

y =

15 −5

y = −

3.

Sustituyendo ahora y = −3 en la primera ecuaci´on se tiene 15x + 4(−3) = 3 15x − 12 = 3 15x = 3 + 12 15 x= 15 x=1. As´ı, la soluci´on del sistema es: x = 1; y = −3.  Al igual que en el m´etodo de sustituci´on, tambi´en se puede eliminar cualquiera de las dos inc´ognitas, as´ı no s´olo x se puede cancelar, tambi´en se puede eliminar y. El siguiente ejemplo muestra esta situaci´on. Ejemplo

2.8.21 Resolver el sistema de ecuaciones 3x − y = 7 6x + 5y = 28

Soluci´ on. Los coeficientes de y tienen signos opuestos, luego basta multiplicar la primera ecuaci´on por 5. As´ı 15x − 5y = 6x + 5y = 21x + 0 =

35 28 63

x =

63 21

x =

3

178

Cap´ıtulo 2.

´ Algebra

y sustituyendo x = 3, en la segunda ecuaci´on se obtiene el valor de y 6(3) + 5y = 28 18 + 5y = 28 5y = 28 − 18 = 10 10 y= 5 y=2. 

Luego, la soluci´on del sistema es: x = 3; y = 2.

Ejercicio 2.8.4 Resolver por el m´etodo de suma o resta los siguientes sistemas de ecuaciones: 1.

x − 4y = 8 3x + 2y = 10

2.

2x − y = 12 3x + 4y = 7

3.

4x + 5y = 13 8x − 7y = 9

4.

3x + 10y = 24 6x − 5y = − 27

5.

− 4x + 5y = 16 3x + 5y = 58

6.

5x + 6y = − 7 3x + 5y = − 7

7.

4x + 7y = 5 11x − 8y = 41

8.

5x + 4y = − 11 − 11x + 8y = − 43

9.

13x + 9y = 10 2x + 5y = − 31

10.

8x + 3y = 7 2x + 11y = 53

11.

− 2x + 7y = 18 4x + 9y = 10

12.

− 3x + 4y = − 10 5x − 2y = 12

13.

7x + 2y = − 5 2x + 3y = 1

14.

9x + 8y = 13 − 2x + 5y = − 30

15.

11x − 4y = 7 8x + 5y = 13

16.

13x − 2y = 25 4x + 5y = 47

2.8. Ecuaciones de primer grado

17.

1 y = −3 5 1 19x + 2y = 2

19.

5x − 3y = − 1 10x + 9y = 18

179

18.

2x + 17y = −10 51 5x − 8y = 2

20.

5 1 x + y = 2 8 6 3 x − 2y = − 9 4

8x +

Soluciones 1. x = 4, y = −1

2. x = 5, y = −2

3. x = 2, y = 1

4. x = −2, y = 3

5. x = 6, y = 8

6. x = 1, y = −2

7. x = 3, y = −1

8. x = 1, y = −4

9. x = 7, y = −9

10. x = −1, y = 5

11. x = −2, y = 2

12. x = 2, y = −1

13. x = −1, y = 1

14. x = 5, y = −4

15. x = 1, y = 1

16. x = 3, y = 7

1 17. x = − , y = 5 2

7 18. x = , y = −1 2

4 3 19. x = , y = 5 3

20. x = −4, y = 3

2.8.5.

Aplicaciones de las ecuaciones de primer grado

Para resolver problemas que se modelan mediante ecuaciones de primer grado, se pueden aplicar los siguientes pasos. Leer cuidadosamente el problema. Responder a las preguntas: ¿cu´ales son los datos? ¿qu´e se quiere encontrar o determinar? En el caso de problemas geom´etricos una figura puede ayudar a una mejor comprensi´on del problema.

180

Cap´ıtulo 2.

´ Algebra

Asignar literales a las cantidades desconocidas. Establecer relaciones algebraicas entre las literales y responder a la pregunta: ¿se han usado los datos relevantes del problema? Resolver la(s) ecuaci´on(es) resultante(s). Comprobar la respuesta. Ejemplo

2.8.22 Hallar dos n´ umeros naturales consecutivos que sumen

235. Soluci´on. ¿Qu´e se busca? Dos n´ umeros naturales consecutivos. Se asignan literales x, y a los n´ umeros buscados, si x es el primer n´ umero entonces y = x + 1. La condici´on de que estos dos n´ umeros sumen 235 se expresa algebraicamente como x + y = 235, o sea x + (x + 1) = 235 . Se procede a resolver la ecuaci´on anterior. x + x + 1 = 235 2x + 1 = 235 2x = 234 x = 117 . Por lo tanto los n´ umeros buscados son 117 y 118. Se observa que los n´ umeros son naturales, consecutivos y en efecto suman 235.  Ejemplo

2.8.23 Las edades de Luis y Roc´ıo suman 35 a˜ nos, si Luis es 3 a˜ nos m´as grande que Roc´ıo, ¿qu´e edad tiene cada uno? Soluci´on. ¿Cu´al es el problema? Determinar las edades de Luis y Roc´ıo. Se asignan literales. Sean R la edad de Roc´ıo, L la edad de Luis. Se establece la relaci´on algebraica L=R+3 ya que, la edad de Luis es tres a˜ nos m´as que la de Roc´ıo. Como la suma de las edades es 35 se tiene R + L = 35 .

2.8. Ecuaciones de primer grado

181

Substituyendo L: R + (R + 3) = 35 R + R + 3 = 35 2R + 3 = 35 2R = 32 R = 16 . As´ı, Roc´ıo tiene 16 a˜ nos y Luis tiene 19 a˜ nos. Se observa que Luis tiene 3 a˜ nos m´as que Roc´ıo y la suma de sus edades es 35.  Ejemplo

2.8.24 Yolanda entrena seis d´ıas a la semana. Si cada d´ıa corre 1 kil´ometro m´as que el d´ıa anterior, ¿cu´antos kil´ometros corre cada d´ıa si en una semana recorre 45 kil´ometros? Soluci´ on. ¿Cu´al es el problema? Encontrar el n´ umero de kil´ometros que corre cada d´ıa Yolanda. Se asigna x al n´ umero de kil´ometros que corre Yolanda el primer d´ıa. Se establecen las relaciones algebraicas. Si el primer d´ıa corre x kil´ometros entonces los d´ıas siguientes corre respectivamente x + 1, x + 2, x + 3, x + 4, x + 5 . Adem´as, la suma de kil´ometros recorridos es 45, por lo tanto: x + (x + 1) + (x + 2) + (x + 3) + (x + 4) + (x + 5) = 45 . Se resuelve la ecuaci´on x + x + 1 + x + 2 + x + 3 + x + 4 + x + 5 = 45 6x + 15 = 45 6x = 30 x=5. As´ı, ha recorrido el primer d´ıa 5 km, el segundo 6 km y sucesivamente 7 km, 8km, 9 km y 10 km. Comprobando la respuesta. Cada d´ıa recorre un kil´ometro m´as y la suma es 45 kil´ometros.  Ejemplo

2.8.25 Un libro cuesta 8 veces lo que vale un cuaderno. Si el costo del cuaderno y del libro es de $144 pesos, ¿cu´anto vale el cuaderno?

182

Cap´ıtulo 2.

´ Algebra

Soluci´ on. Se busca el precio del cuaderno. Sea C el precio del cuaderno, entonces el libro cost´o 8C . Por lo tanto C + 8C = 144 9C = 144 144 C= 9 C = 16 . Es decir, el cuaderno vale 16 pesos. Se observa que el libro vale 128 pesos, 8 veces el precio del cuaderno y la suma de ambas cantidades es 144 pesos.  Ejemplo

2.8.26 Dos ´angulos son complementarios y el doble del ´angulo menor es igual al ´angulo mayor, ¿cu´anto mide cada ´angulo? Soluci´on. Sea α el ´angulo menor y β el ´angulo mayor. α + β = 90◦ β = 2α .

por ser complementarios por ser el mayor el doble del menor

Por lo tanto, α + 2α = 90◦ 3α = 90◦ α = 30◦ . Luego los ´angulos son 30◦ y 60◦ .



Ejemplo

2.8.27 La suma de dos n´ umeros es 120, si el triple del menor es igual al doble del mayor, ¿cu´ales son los n´ umeros? Soluci´on. Sea x el n´ umero menor, y el n´ umero mayor. x +

y = 120 3x = 2y

2y Despejando x de la segunda ecuaci´on se tiene x = . Se sustituye este valor 3 en la primera ecuaci´on y se obtiene el valor de y:

2.8. Ecuaciones de primer grado

183

2 y + y = 120 3 5 y = 120 3 120 · 3 y= 5 y = 72 . Ahora al sustituir y = 72 en la primera ecuaci´on se obtiene el valor de x x + 72 = 120 x = 120 − 72 x = 48 . Los n´ umeros buscados son 48 y 72.



Ejemplo

2.8.28 En una posada hay 57 habitaciones. Si las habitaciones de la planta alta son el doble de las que hay en la planta baja, ¿cu´antas habitaciones hay en cada planta? Soluci´ on. Sea x el n´ umero de habitaciones que hay en la planta baja, y el n´ umero de habitaciones que hay en la planta alta. x + y = 57 y = 2x Sustituyendo la segunda ecuaci´on en la primera x + 2x = 57 3x = 57 x = 19 Entonces el n´ umero de habitaciones de la planta baja es 19 y en la planta alta hay 38 habitaciones.  Ejemplo

2.8.29 La suma de dos n´ umeros es 132 y su diferencia es 64, hallar los n´ umeros.

184

Cap´ıtulo 2.

´ Algebra

Soluci´ on. Sean x, y los n´ umeros buscados, entonces x + y = 132 x − y = 64 , sumando ambas ecuaciones 2x = 196 x = 98 y sustituyendo ahora en la primera ecuaci´on 98 + y = 132 y = 132 − 98 y = 34 . 

Los n´ umeros son 34 y 98. Ejemplo

2.8.30 Un comerciante gast´o $22, 400 en la compra de 60 camisas. Los precios de las camisas son $350 y $400, ¿cu´antas camisas de cada precio compr´o? Soluci´on. Sea x el n´ umero de camisas de $350 que compr´o, y el n´ umero de camisas de $400. x + y = 60 350x + 400y = 22400 Pasando a un sistema equivalente y sumando − 350x − 350y = − 21000 350x + 400y = 22400 50y = 1400 y =

1400 50

y =

28

.

Por lo tanto, compr´o 32 camisas de $350 y 28 camisas de $400.



2.8. Ecuaciones de primer grado

185

Ejemplo

2.8.31 Un ahorrador tiene $100, 000 y puede invertir en bonos del gobierno al 8 % y en una empresa vidriera (con un riesgo mayor) al 12 %. Si quiere obtener una ganancia del 10 %, ¿cu´anto tiene que invertir a cada porcentaje? Soluci´ on. Sea x lo que invierte al 8 % e y al 12 %, entonces: x + y = 100000 0. 08x + 0. 12y = 10000 Resolviendo el sistema − 0. 08x − 0. 08y = − 8000 0. 08x + 0. 12y = 10000 2000 0. 04y = y =

2000 0. 04

y =

50000

. 

Debe invertir $50, 000 al 8 % y $50, 000 al 12 % . Ejemplo

2.8.32 Un joyero tiene dos aleaciones. La primera aleaci´on contiene 35 % de oro y la segunda aleaci´on 60 % de oro. ¿Cu´anto debe fundir de cada una para que al combinarlas obtenga 100 gr de una aleaci´on con 50 % de oro (oro de 12 kilates)? Soluci´ on. Sean x la cantidad de la primera aleaci´on, y la cantidad de la segunda aleaci´on. x + y = 100 0. 35x + 0. 6y = 50 Pasando al sistema equivalente y sumando − 0. 35x − 0. 35y = − 35 0. 35x + 0. 60y = 50 0. 25y = 15 y =

15 0. 25

y =

60 .

186

Cap´ıtulo 2.

´ Algebra

As´ı 40 gramos de la primera aleaci´on y 60 gramos de la segunda aleaci´on producir´an una aleaci´on con el 50 % de oro.  Ejercicio 2.8.5 Resolver los siguientes problemas. 1. La edad de Leticia es la mitad que la de Luis y ambas edades suman 48 a˜ nos ¿Qu´e edad tiene cada uno? 2. Dividir al n´ umero 846 en dos partes tal que una parte exceda a la otra en 54. 3. Hallar 3 n´ umeros enteros consecutivos cuya suma sea 942. 4. Una persona deja una herencia de $3, 460, 000 para repartir entre sus 3 hijos y 2 hijas, indicando que cada hija reciba $200, 000 m´as que cada hijo. ¿Cu´anto recibi´o cada hija y cada hijo? 5. Un almacen rebaja el precio de un art´ıculo en 30 %, si un cliente paga $840 por el art´ıculo. ¿cu´al era el precio original del art´ıculo? 6. El triple de un n´ umero equivale al n´ umero aumentado en 172, ¿cu´al es el n´ umero? 7. En su viaje Felipe ha recorrido un total de 62400 km. En avi´on recorri´o 30 veces m´as que en autob´ us y en autob´ us 5 veces m´as que en tren, ¿cu´antos kil´ometros ha recorrido en avi´on, en tren y en autob´ us? 8. Itzel y Mar´ıa inician el juego de la oca con 10 pesos cada una, ¿cu´anto ha perdido Mar´ıa, si ahora Itzel tiene el triple de lo que tiene Mar´ıa? 9. V´ıctor ten´ıa $3, 600 y realiz´o algunas compras, lo que le sobra es un quinto de lo que gast´o, ¿cu´anto gast´o? 10. La diferencia de los cuadrados de dos n´ umeros naturales consecutivos es 27. ¿Cu´ales son los n´ umeros? 4 11. Dentro de 6 a˜ nos la edad de Diana ser´a de la edad de Irving y en 5 5 10 a˜ nos la edad de Diana ser´a de la edad de Irving. ¿cu´ales son las 6 edades actuales?

2.8. Ecuaciones de primer grado

187

12. Una cuerda de 3 m se ha pintado de azul y blanco, la parte pintada de azul es la mitad menos doce cent´ımetros de lo pintado de blanco. ¿Qu´e parte de la cuerda est´a pintada de azul? 13. La suma de dos n´ umeros es 106, si al n´ umero mayor lo dividimos entre el n´ umero menor el resultado es 1 con un residuo de 32. ¿Cu´ales son los n´ umeros? 14. A una clase de matem´aticas asisten 42 personas, si al doble del n´ umero de mujeres que asisten se le agrega 6, se obtiene el n´ umero de varones. ¿Cu´antas mujeres hay en la clase? 15. La edad de Leticia es la mitad de la que tiene Luis y la edad de Rogelio es el triple de la de Leticia. Si todas las edades suman 126 a˜ nos, ¿qu´e edad tiene cada uno? 16. En una sala rectangular el largo es el doble del ancho. Si cada dimensi´on se aumenta en 3 metros el ´area aumentar´ıa 90 m2 . ¿Cu´ales son las dimensiones de la sala? 3 de lo que le quedaba es lo que 4 pag´o por una camisa. Si ahora tiene $125, ¿cu´anto ten´ıa inicialmente?

17. Un estudiante gast´o $875 en libros y

18. En la entrada del acuario de Veracruz hay tortugas y tres tipos de aves, 1 guacamayas verdes, guacamayas rojas y tucanes, si de las aves son 4 1 guacamayas rojas, de las aves son guacamayas verdes y los tucanes 3 son 5. ¿Cu´antas aves hay en total? 19. Dos atletas se encuentran corriendo en una competencia y los separa una distancia de 24 m. El corredor rezagado avanza a 6 m s y el otro a m 4 s . ¿En cu´anto tiempo el corredor rezagado dar´a alcance al que lleva la delantera? 20. Una liebre est´a a 81 de sus saltos de un can lebrejero. La liebre da 5 saltos mientras el can da 2 pero, un salto del can equivale a 4 saltos de la liebre. ¿Cu´antos saltos debe dar el can para alcanzar a la liebre? Soluciones.

188

Cap´ıtulo 2.

1. Leticia tiene 16 a˜ nos, Luis tiene 32 a˜ nos. 2. Los n´ umeros son 396 y 450. 3. 313, 314 y 315. 4. Cada hijo recibi´o $612, 000 y cada hija recibi´o $812, 000. 5. $1, 200 6. 86 7. En avi´on 60 000km, en autobus 2 000km y en tren 400km. 8. Itzel tiene $15 y Mar´ıa tiene $5. 9. Gast´o $3, 000. 10. 13 y 14. 11. Diana tiene actualmente 10 a˜ nos, Irving tiene 14 a˜ nos. 12. 92 cm. 13. 37 y 69. 14. 12 mujeres. 15. Rogelio 63, Luis 42, Leticia 21. 16. Largo 18, ancho 9. 17. Inicialmente ten´ıa $1, 375. 18. 12 aves. 19. 12 segundos. 20. 54 saltos.

´ Algebra

2.9. Ecuaciones de segundo grado

2.9.

189

Ecuaciones de segundo grado

2.9 Ecuaciones de segundo grado Una ecuaci´on de la forma

ax2 + bx + c = 0

con

a ̸= 0

se llama ecuaci´ on de segundo grado o ecuaci´ on cuadr´ atica, en la literal x. Recibe este nombre porque el exponente mayor de x es 2. A los valores de x que satisfacen esta ecuaci´on se les llama ceros o ra´ıces de la ecuaci´on. Las expresiones siguientes son ejemplos de ecuaciones de segundo grado −3x2 + 5x + 3 = 0 ,

3 z2 − z = 8 , 5

u2 = 6 .

Los m´etodos que se estudian aqu´ı para resolver una ecuaci´on cuadr´atica son: Por factorizaci´on. Por completaci´on del trinomio cuadrado perfecto. Por la f´ormula general. La forma de operar en los dos primeros m´etodos ya se ha estudiado en la secci´on 2.5.5. La f´ormula general es consecuencia de la completaci´on del trinomio cuadrado perfecto y adem´as dice que una ecuaci´on cuadr´atica puede tener a los m´as dos soluciones.

2.9.1.

El m´ etodo de factorizaci´ on

El m´etodo de factorizaci´on se basa en la propiedad de que un producto de n´ umeros reales es cero si y s´olo si alguno de los factores es igual a cero. Este procedimiento consiste en factorizar ax2 + bx + c en dos factores lineales o sea de primer grado. El Teorema 2.5.1 garantiza que esto es posible si el discriminante b2 − 4ac es mayor o igual que cero. Cuando se tienen los dos factores lineales se iguala a cero cada uno de ellos y se resuelven las ecuaciones lineales.

190

Ejemplo

Cap´ıtulo 2.

´ Algebra

2.9.1 Resolver la ecuaci´on de segundo grado x2 + 3x = 10 .

Soluci´on. Se reescribe la ecuaci´on en la forma x2 + 3x − 10 = 0 . Se factoriza el trinomio cuadr´atico y se obtiene (x + 5)(x − 2) = 0 . Esta u ´ltima igualdad se cumple si alguno de los factores es cero, es decir, si x+5=0

´o

x−2=0 .

Al resolver las ecuaciones lineales se tiene x = −5, x = 2. Por lo anterior, las soluciones de la ecuaci´on x2 + 3x − 10 = 0 son x1 = −5 y x2 = 2.  Com´ unmente se denota por x1 y x2 las soluciones de la ecuaci´on cuadr´atica. Ejemplo

2.9.2 Resolver la ecuaci´on cuadr´atica −17x = 5 − 12x2 empleando el m´etodo de factorizaci´on. Soluci´on. Se reescribe la ecuaci´on en la forma 12x2 − 17x − 5 = 0 . Se factoriza el trinomio cuadr´atico y se obtiene (3x − 5)(4x + 1) = 0 . Esta igualdad se cumple si 3x − 5 = 0

´o

4x + 1 = 0 .

1 5 Al despejar x se obtienen las soluciones x1 = − , x2 = . 4 3 Ejemplo

2.9.3 Resolver la ecuaci´on cuadr´atica x2 − 4x = 0.



2.9. Ecuaciones de segundo grado

191

Soluci´ on. Se factoriza el trinomio cuadr´atico y se obtiene x(x − 4) = 0 . Esta u ´ltima igualdad se cumple si alguno de los factores es cero, es decir, cuando x=0 ´o x−4=0 . Al despejar x se obtienen las soluciones x1 = 0, x2 = 4. Ejemplo



2.9.4 Resolver la ecuaci´on x2 − 12x + 36 = 0.

Soluci´ on. Se factoriza el trinomio cuadr´atico y se obtiene (x − 6)2 = 0 . Se observa que los factores son iguales. Despejando se tiene x−6=0 ,

x=6.

De lo anterior se concluye que x1 = x2 = 6 y se dice que la ecuaci´on tiene una ra´ız de multiplicidad dos.  Un caso particular es cuando la ecuaci´on cuadr´atica no tiene t´ermino lineal, es decir, b = 0 y por tanto ax2 + c = 0 . Si ac < 0 ( a y c tienen signos opuestos) se tiene una diferencia de cuadrados, la cual se factoriza como producto de binomios conjugados. Otra alternativa consiste en despejar directamente la inc´ognita x. Ambos procedimientos se muestran en el siguiente ejemplo. Ejemplo

2.9.5 Resolver la ecuaci´on 4x2 − 81 = 0.

Soluci´ on. Se factoriza la diferencia de cuadrados 4x2 − 81 = 0 ,

(2x + 9)(2x − 9) = 0 .

Esta u ´ltima igualdad se cumple si alguno de los factores es cero, es decir, si 2x + 9 = 0 ,

2x − 9 = 0 .

192

Cap´ıtulo 2.

´ Algebra

9 9 Al despejar se obtiene x1 = − , x2 = que son las soluciones de la ecuaci´on. 2 2 Como la ecuaci´on no tiene t´ermino lineal, se puede despejar directamente a x y obtener las mismas soluciones: 2

4x = 81 ,

81 , x = 4 2

√ x=±

9 81 =± . 4 2 

Ejercicio 2.9.1 Resolver las siguientes ecuaciones cuadr´aticas 1. x2 + x − 12 = 0

2. x2 − 12x + 35 = 0

3. x2 − 5x − 24 = 0

4. −x2 − 10x − 21 = 0

5. −x2 + 8x + 9 = 0

6. x2 + 3x = 0

7. −x2 + 7x = 0

8. 8x − 2x2 = 0

9. 8x2 − 2x − 21 = 0

10. 30x2 + 13x − 3 = 0

11. 6x2 − 25x + 4 = 0

12. −10x2 + 11x + 6 = 0

13. 30x2 + 15x = 0

14. −12x2 + x + 1 = 0

15. −10x2 + 15x = 0

16. x2 = 9 16 = x2 25

4 x

17. x2 − 2 = 0

18.

20. x2 − 36a4 = 0

21. z 2 − 5 = 0

22. −7x2 + 8 = 0

23. 3x2 − 1 = 0

24. −6x2 = −8

25. 2(x2 − 10) = 4

26.

25 2 x −9=0 16

27. (x + 3)2 − 81 = 0

19. x =

2.9. Ecuaciones de segundo grado

193

Soluciones. 1. x1 = −4, x2 = 3

2. x1 = 5, x2 = 7

3. x1 = −3, x2 = 8

4. x1 = −7, x2 = −3

5. x1 = −1, x2 = 9

6. x1 = −3, x2 = 0

7. x1 = 0, x2 = 7

8. x1 = 0, x2 = 4

9. x1 = −

3 1 10. x1 = − , x2 = 5 6

1 11. x1 = , x2 = 4 6

2 3 12. x1 = − , x2 = 5 2

1 13. x1 = − , x2 = 0 2

1 1 14. x1 = − , x2 = 4 3

15. x1 = 0, x2 =

16. x1 = −3, x2 = 3

√ √ 17. x1 = − 2, x2 = 2

4 4 18. x1 = − , x2 = 5 5

19. x1 = −2, x2 = 2

20. x1 = −6a2 , x2 = 6a2

√ √ 21. z1 = − 5, z2 = 5

√ 22. x1 = −

8 , x2 = 7



8 7

2 2 24. x1 = − √ , x2 = √ 3 3 26. x1 = −

2.9.2.

12 12 , x2 = 5 5

3 7 , x2 = 2 4

3 . 2

1 1 23. x1 = − √ , x2 = √ 3 3 √ √ 25. x1 = −2 3, x2 = 2 3 27. x1 = −12, x2 = 6

M´ etodo de completaci´ on de cuadrados

Este m´etodo se basa en la completaci´on de cuadrados, visto en la subsecci´on 2.5.5 y lo ilustra el siguiente ejemplo. Ejemplo

2.9.6 Resolver la ecuaci´on x2 + 4x − 7 = 0 completando el trinomio cuadrado perfecto.

194

Cap´ıtulo 2.

´ Algebra

Soluci´ on. Se tiene x2 + 4x − 7 = 0 ( )2 ( )2 4 4 2 −7= x + 4x + 2 2 2 x + 4x + 4 = 7 + 4 (x + 2)2 = 11 √ x + 2 = ± 11 √ x = −2 ± 11

se completa el trinomio se reduce se expresa como binomio al cuadrado se extrae ra´ız cuadrada se despeja x

Luego, los ceros de la ecuaci´on son x1 = −2 −



11, x2 = −2 +





11.

Otra forma de obtener la soluci´on, consiste en realizar una variante de la completaci´on del trinomio, seg´ un se muestra.

ax2 + bx + c = 0 a2 x2 + a(bx) + ac = 0 (ax)2 + b(ax) + ac = 0 ( )2 ( )2 b b 2 (ax) + b(ax) + + ac = 2 2 ( )2 b b2 ax + = − ac 2 4 √ b b2 − 4ac ax + = ± 2 √4 −b ± b2 − 4ac x= . 2

Ejemplo cuadrado.

se multiplica por a se agrupa se completa el trinomio se factoriza

se toma ra´ız cuadrada

se despeja x

(2.9.6)

2.9.7 Resolver la ecuaci´on 2x2 + 3x − 4 = 0 completando el

2.9. Ecuaciones de segundo grado

195

Soluci´ on. Por el procedimiento mencionado antes 2x2 + 3x − 4 = 0 2(2x2 ) + 2(3x) + 2(−4) = 0 (2x)2 + 3(2x) − 8 = 0 9 9 (2x)2 + 3(2x) + − 8 = 4 4 ( )2 41 3 = 2x + 2 4 √ 3 41 2x + = ± 2 4 √ 3 41 x=− ± 4 4

se multiplica por 2 se agrupa se completa el cuadrado se factoriza y reduce

se toma ra´ız cuadrada

se despeja x

Luego los ceros de la ecuaci´on 2x2 + 3x − 4 = 0 son √ √ 3 41 −3 − 41 x1 = − − = , 4 4 4

√ √ 3 41 −3 + 41 x2 = − + = . 4 4 4

Tambi´en se puede hacer por la completaci´on usual de cuadrados. 2x2 + 3x − 4 = 0 3 x2 + x − 2 = 0 2 ( )2 3 9 x+ −2− =0 4 16 ( )2 3 41 x+ = 4 16 √ 41 3 x+ =± 4 16 √ 3 41 x=− ± 4 4

se divide entre 2 se completa el cuadrado y se factoriza

y se reduce

se toma ra´ız cuadrada

se despeja

 Ejercicio 2.9.2 Resolver las siguientes ecuaciones de segundo grado com-

196

Cap´ıtulo 2.

´ Algebra

pletando el trinomio cuadrado perfecto. 1. x2 + 3x − 1 = 0

2. x2 − 7x = 2

3. x2 = −9x + 4

4. 2x + x2 = 5

5. 4x2 + 8x + 4 = 0

6. 5x2 − 2x − 7 = 0

7. 3x2 − x + 1 = 0

8. 6x2 = 5 − 2x

9. 9x2 − 4 − x = −1

10.

x2 + 3x − 5 = 0 3

11. x2 +



2x − 3 = 0

12. x2 + x + 1 = 0

Soluciones. √ √ −3 − 13 −3 + 13 1. x1 = , x2 = 2 2 √ √ −9 − 97 −9 + 97 3. x1 = , x2 = 2 2



57

2

4. x1 = −1 −



, x2 =

1−

√ 6

13

, x2 =

1+

√ 6

13

,

√ √ −1 − 31 −1 + 31 8. x1 = , x2 = 6 6 √ √ −1 − 109 −1 + 109 9. x1 = , x2 = 18 18 √ √ −9 − 141 −9 + 141 10. x1 = , x2 = 2 2 √ √ 1 √ 1 √ 11. x1 = (− 2 − 14), x2 = (− 2 − 14) 2 2 12. No tiene soluci´on real

7+



7 5

57

2

6, x2 = −1 +

6. x1 = −1, x2 =

5. x = 1

7. x1 =

2. x1 =

7−



6

2.9. Ecuaciones de segundo grado

2.9.3.

197

La f´ ormula general

Otro m´etodo para resolver una ecuaci´on cuadr´atica es una consecuencia directa del m´etodo anterior, concretamente de la f´ormula 2.9.6 y del Teorema 2.5.1. Teorema 2.9.1 La ecuaci´on cuadr´atica general

ax2 + bx + c = 0 tiene por soluciones

x=

−b ±



b2 − 4ac . 2a

Estas soluciones: a) Son dos n´ umeros reales diferentes si b2 − 4ac > 0. b) Son dos n´ umeros reales e iguales y se dice que la ra´ız es de multiplicidad dos, si b2 − 4ac = 0. c) No son n´ umeros reales si b2 − 4ac < 0. Para resolver la ecuaci´on de segundo grado ax2 + bx + c = 0 utilizando la f´ormula cuadr´atica se identifican los valores de a, b y c, se sustituyen en la f´ormula general y se reduce para obtener la soluci´on de la ecuaci´on. Ejemplo

2.9.8 Resolver la siguiente ecuaci´on de segundo grado aplicando la f´ormula general x2 = 17 .

198

Cap´ıtulo 2.

´ Algebra

Soluci´ on. Se escribe la ecuaci´on en la forma general x2 − 17 = 0 . En este caso se identifican: a = 1, b = 0 y c = −17. Se sustituyen estos valores en la f´ormula general y se tiene √ −(0) ± (0)2 − 4(1)(−17) x= 2(1) √ ± 4(17) x= 2 √ 2 ± 17 x= . 2 De aqu´ı

√ x1 = − 17

y

x2 =



17 

son las soluciones de la ecuaci´on x2 = 17. Ejemplo

2.9.9 Resolver la siguiente ecuaci´on de segundo grado aplicando la f´ormula general −8x2 + 7x = 0 . Soluci´on. De la ecuaci´on se identifica En este caso se identifican: a = −8, b = 7 y c = 0. Se sustituyen estos valores en la f´ormula general y se tiene √ −(7) ± (7)2 − 4(−8)(0) x= 2(−8) √ −7 ± 49 x= −16 −7 ± 7 x= . −16 De aqu´ı x1 = As´ı x1 = 0 y x2 =

−7 + 7 =0 −16

y

x2 =

−7 − 7 7 = . −16 8

7 son las soluciones de la ecuaci´on −8x2 + 7x = 0. 8



2.9. Ecuaciones de segundo grado

199

Ejemplo

2.9.10 Resolver la siguiente ecuaci´on de segundo grado aplicando la f´ormula general x2 − 5x = 14 . Soluci´ on. Se escribe la ecuaci´on en la forma general x2 − 5x − 14 = 0 . En este caso se identifican: a = 1, b = −5 y c = −14. Se sustituyen estos valores en la f´ormula general y se tiene √ −(−5) ± (−5)2 − 4(1)(−14) x= 2(1) √ 5 ± 81 x= 2 5±9 x= . 2 De aqu´ı 5−9 5+9 x1 = = −2 y x2 = =7. 2 2 As´ı x1 = −2 y x2 = 7 son las soluciones de la ecuaci´on x2 − 5x = 14.  Ejemplo

2.9.11 Resolver la siguiente ecuaci´on de segundo grado aplicando la f´ormula general x2 − 14x + 49 = 0 Soluci´ on. En ecuaci´on anterior se identifican a = 1, b = −14 y c = 49. Sustituyendo estos valores en la f´ormula general se obtiene √ −(−14) ± (−14)2 − 4(1)(49) x= 2(1) √ 14 ± 0 x= 2 Luego 14 x1 = = 7 = x2 . 2 La soluci´on de la ecuaci´on x2 − 14x + 49 = 0 es x = 7, que es una ra´ız de multiplicidad dos. 

200

Cap´ıtulo 2.

´ Algebra

Ejemplo

2.9.12 Resolver la siguiente ecuaci´on de segundo grado aplicando la f´ormula general 5x2 − 7x − 1 = 0 . Soluci´on. En este caso se tiene que a = 5, b = −7 y c = −1. Se sustituyen estos valores en la f´ormula general y se tiene √ −(−7) ± (−7)2 − 4(5)(−1) x= 2(5) √ 7 ± 63 + 20 x= √10 7 ± 83 x= . 10 De aqu´ı

√ √ 7 − 83 7 + 83 x1 = y x2 = 10 10 2 son las soluciones de la ecuaci´on 5x − 7x − 1 = 0. Ejemplo



2.9.13 Resolver la ecuaci´on x2 − 6x + 10 = 0

aplicando la f´ormula general. Soluci´on. Usando la f´ormula general se tiene √ √ −(−6) ± (−6)2 − 4(1)(10) 6 ± −4 x= = . 2(1) 2 Se observa que b2 − 4ac = −4 es menor que cero. Luego, no existe ning´ un 2 n´ umero real x que permita que el polinomio x − 6x + 10 sea igual a cero. En otras palabras, la ecuaci´on de segundo grado x2 − 6x + 10 = 0 no tiene soluciones reales.  Ejercicio 2.9.3 Resolver las siguientes ecuaciones de segundo grado aplicando la f´ormula general.

2.9. Ecuaciones de segundo grado

201

1. 15x2 + 32x − 7 = 0

2. 14x2 = 15 − 29x

3. 18x2 − 21x − 4 = 0

4. −2x2 + 15x − 28 = 0

5. 15x2 − 37x − 8 = 0

6. −15x2 + 61x − 22 = 0

7. −63x2 + 2x + 1 = 0 9. −x2 +



2x + 8 = 0

√ √ √ 11. − 5x2 + 10x + 3x − 3 2 = 0 13.



x 1 8. x2 − √ − = 0 8 4 √ 2 10. 7x + 2x − 1 = 0 12.



2x2 − 6x − (

7x − 3x + 2 = 0 2

15. x2 − zd + zx = dx 17. x − 1 =



11 − 5x

19. 7x4 + 4x2 − 3 = 0

3 14. − x + 2



)2 =

8=0



3

√ 16. − 77 − 17x + x = 7 18. x =



3x + 10

20. x4 + 100 = 29x2

Soluciones. 7 1 1. x1 = − , x2 = 3 5

5 3 2. x1 = − , x2 = 2 7

1 4 3. x1 = − , x2 = 6 3

7 4. x1 = , x2 = 4 2

1 8 5. x1 = − , x2 = 5 3

2 11 6. x1 = , x2 = 5 3

1 1 7. x1 = − , x2 = 9 7

1 1 8. x1 = − √ , x2 = √ 2 2 2

202

Cap´ıtulo 2.

−1 − √ 9. x1 =

10. x1 =

−1 −



17

2 √

√ −1 + 17 √ , x2 = 2

1+ √ 7



7

, x2 =

−1 +



1+ √ 7



3 2, x2 = √ 5 √ √ −3 − 13 −3 + 13 √ √ 12. x1 = , x2 = 2 2

11. x1 =

13. No tiene soluciones reales 14.

No tiene soluciones reales

15. x1 = −z, x2 = d 16. x1 = −7, x2 = 4 17. x1 = −5, x2 = 2 18. x1 = −2, x2 = 5 3 3 19. x1 = −1, x2 = − , x3 = , x4 = 1 7 7 20. x1 = −2, x2 = 2, x3 = 5, x4 = −5



7

´ Algebra

Cap´ıtulo 3 Geometr´ıa Este cap´ıtulo se apoya en la noci´on intuitiva que tiene el lector sobre algunos conceptos geom´etricos, es por ello que algunas definiciones y resultados se argumentar´an desde su propia representaci´on gr´afica.

3.1.

´ Angulos y rectas paralelas

Dados dos puntos A, B en un plano, la l´ınea recta que pasa por ´estos se ←→ −→ denota por AB, el rayo con extremo A que pasa por B se escribe AB. La figura 3.1 ilustra su representaci´on gr´afica. El segmento de recta determinado por A y B se denota como AB. La distancia entre los puntos A, B A

B

A

B

Figura 3.1: La recta y el rayo que pasan a trav´es de los puntos A y B. se denota por AB y es precisamente la longitud del segmento AB. En la pr´actica se usa una regla para medir distancias entre puntos como ilustra la figura 3.2. Un ´ angulo consta de dos rayos que tienen el mismo punto extremo. Si −→ −−→ los rayos son OA y OB, el punto O es el v´ ertice y se denota al ´angulo por ∠AOB o ∠BOA, la figura 3.3 muestra su representaci´on gr´afica. El grado es una unidad de medida para ´angulos. Un grado se obtiene al dividir la circunferencia en 360 partes iguales. A cada ´angulo se le asocia 203

204

Cap´ıtulo 3.

A 0

Geometr´ıa

B 1

2

3

4

5

6

7

8

9

10

Figura 3.2: La regla sobrepuesta al segmento AB.

B

A O

Figura 3.3: El ´angulo ∠AOB. su medida en grados y se usa un transportador para determinarla, seg´ un aparece en la figura 3.4. Se cumple el principio de adici´ on para la medida

120 145 150 180

90

60 45 30 0

Figura 3.4: El transportador. de ´angulos, es decir, si B es punto en el interior del ´angulo ∠AOC entonces su medida, tambi´en denotada por ∠AOC, cumple ∠AOC = ∠AOB + ∠BOC , seg´ un muestra la figura 3.5. Dos ´angulos se dicen congruentes si tienen igual medida. Un ´angulo se llama agudo si su medida es menor que 90◦ , es recto si su medida es 90◦ y es obtuso si su medida es mayor que 90◦ y menor que 180◦ seg´ un muestra la figura 3.6.

3.1.

´ Angulos y rectas paralelas

205

C

B

A O

Figura 3.5: La adici´on de la medida de ´angulos. Dos ´angulos son complementarios si suman 90◦ y son suplementarios si suman 180◦ . La medida de los ´angulos se expresa en grados (◦ ), minutos (′ ) y segundos (′′ ), donde 1◦ = 60′ y 1′ = 60′′ .

HaL

HbL

HcL

´ ´ ´ Figura 3.6: (a) Angulo agudo, (b) Angulo recto, (c) Angulo obtuso.

Sean L, M y T tres rectas distintas entre si. La l´ınea recta T es una transversal a L y M si intersecta a L y M , ver la figura 3.7. De esta misma figura se llaman: Alternos internos los ´angulos ∠α y ∠η, ∠β y ∠θ. Alternos externos los ´angulos ∠γ y ∠ϵ, ∠δ y ∠ζ. Correspondientes los ´angulos ∠δ y ∠θ, ∠γ y ∠η, ∠β y ∠ζ, ∠α y ∠ϵ. Opuestos por el v´ ertice los ´angulos ∠α y ∠γ, ∠β y ∠δ, ∠ζ y ∠θ, ∠η y ∠ϵ.

206

Cap´ıtulo 3.

Geometr´ıa

Los ´angulos opuestos por el v´ertice siempre son congruentes. Dadas dos l´ıneas rectas L y M en un plano se dice que son paralelas si

T Ε

Ζ

L

Η

Θ

M

Β Α Γ ∆

Figura 3.7: La recta transversal T a las rectas L y M no se intersectan. El postulado que caracteriza a la geometr´ıa Euclidiana se denomina Quinto Postulado de Euclides, ver figura 3.8, y dice En un plano, dada una l´ınea recta y un punto fuera de ella, existe s´olo una l´ınea recta que pasa por dicho punto y que es paralela a la l´ınea dada.

M P L

Figura 3.8: La u´nica recta M paralela a la recta L por el punto P . Una equivalencia de este quinto postulado de la geometr´ıa Euclidiana es Teorema 3.1.1 Sean L y M dos rectas diferentes y T una transversal a las dos rectas. Entonces L y M son paralelas si y s´olo si es cierta alguna de las afirmaciones siguientes, ver figura 3.9:

3.1.

´ Angulos y rectas paralelas

207

Los pares de ´angulos alternos internos son congruentes, es decir ∠α1 = ∠α2 y ∠β1 = ∠β2 . Los pares de ´angulos alternos externos son congruentes ∠γ1 = ∠γ2 y ∠δ1 = ∠δ2 . Los pares de ´angulos correspondientes son congruentes, por ejemplo ∠δ1 = ∠β2 y ∠γ1 = ∠α2 .

M

∆2 Α2

Γ2

Β2

T Β1 Γ1

Α1

L

∆1

Figura 3.9: La recta transversal T a las rectas paralelas L y M Ejemplo

3.1.1 Determinar el valor del ´angulo α, que aparece en la figura

3.10. Soluci´ on. Como los ´angulos opuestos por el v´ertice son iguales, se tiene que 2α + 78◦ 27′ + α + 32◦ 45′ = 180◦ . Como 1◦ = 60′ , al sumar se obtiene; 78◦ 27′ + 32◦ 45′ = 110◦ + 72′ = 110◦ + 1◦ + 12′ = 111◦ 12′ . As´ı 3α + 111◦ 12′ = 180◦ 3α = 179◦ + 1◦ − 111◦ − 12′ = 88◦ 48′ . Entonces 3α = 87◦ + 108′ y α = 29◦ 36′ .



208

Cap´ıtulo 3.

Geometr´ıa

78°27' 2Α

32°45' Α

Figura 3.10: El ´angulo α. Ejercicio 3.1.1 Hallar el complemento de los ´angulos siguientes: 1. 72◦

2. 65◦ 32′

3. 25◦ 45′ 18′′

y el suplemento de los ´angulos siguientes 4. 81◦

5. 115◦ 43′

6. 148◦ 33′ 23′′

Soluciones. 1. 18◦ 2. 24◦ 28′ 6. 31◦ 26′ 37′′

3. 64◦ 14′ 42′′

4. 99◦

5. 64◦ 17′

Ejercicio 3.1.2 Determinar los valores de los ´angulos sujetos a una de las condiciones siguientes: 1. El complemento del ´angulo α es el triple del propio ´angulo α. 2. El suplemento β del ´angulo α es 35◦ m´as grande que el ´angulo α. 3. Los ´angulos α y β son complementarios y su diferencia es 25◦ . 4. Los ´angulos α y β son suplementarios y la diferencia entre α y el doble de β es de 120◦ .

3.1.

´ Angulos y rectas paralelas

209

5. El ´angulo α y su cuadrado son complementarios. Soluciones. 1. α = 22◦ 30′ , β = 67◦ 30′ 3. α = 57◦ 30′ , β = 32◦ 30′ 5. α = 9◦

2. α = 72◦ 30′ , β = 107◦ 30′ 4. α = 160◦ , β = 20◦

Ejercicio 3.1.3 1. En la figura 3.11 calcular el valor del ´angulo α.

30° 50°

Α



Figura 3.11: El ´angulo α 2. En la figura 3.12 las rectas L y M son paralelas. Calcular el valor del ´angulo α.

L Α

150°

M 38°

Figura 3.12: Las rectas L, M y el ´angulo α. 3. En la figura 3.13 las rectas L y M son paralelas. Calcular el valor de los ´angulos α, β y γ. Soluciones. 1. α = ( 100 )◦ 2. α = 68◦ 3. α = 51◦ , β = 92◦ , γ = 37◦ . 3

210

Cap´ıtulo 3.

L

Geometr´ıa

Γ

Α Β

M

129 °

37

°

Figura 3.13: Las rectas L, M y los ´angulos α, β y γ.

3.2.

Pol´ıgonos

Una poligonal simple es una sucesi´on de segmentos rectos en la que cada uno est´a unido por sus extremos con otros dos segmentos, siendo ´estos los dos u ´nicos puntos comunes entre segmentos. Se dice que la poligonal es cerrada si el u ´ltimo segmento est´a unido con el primero. Se usar´a s´olo el t´ermino poligonal. Un pol´ıgono es una porci´on del plano, limitada por una poligonal cerrada y tiene los siguientes elementos: Lados son los segmentos de la poligonal. ertices son las intersecciones de dos lados consecutivos. V´ ´ Angulos interiores son los ´angulos formados por dos lados consecutivos. ´ exteriores son los ´angulos formados en un v´ertice por un Angulos lado y la prolongaci´on del lado consecutivo. Diagonales son l´ıneas rectas que unen dos v´ertices no consecutivos. El per´ımetro de un pol´ıgono es la suma de las longitudes de sus lados.

3.2.1.

Tri´ angulos

Los pol´ıgonos m´as simples son los tri´ angulos, que son aqu´ellos con tres lados. El tri´angulo con v´ertices A, B, C se denota por △ABC. Los tri´angulos se clasifican de acuerdo con la longitud de sus lados en:

3.2.

Pol´ıgonos

211

L

Α

D

V

P

Β

Figura 3.14: Se muestra un v´ertice V , un lado L, una diagonal D, un ´angulo interno α y un ´angulo externo β del pol´ıgono P . ateros si los tres lados tienen igual longitud. Equil´ Is´ osceles si dos lados tienen igual longitud. Escalenos si los tres lados tienen diferente longitud.

HaL

HbL

HcL

Figura 3.15: (a) Equil´atero, (b) Is´osceles, (c) Escaleno. Tambi´en se pueden clasificar de acuerdo con sus ´angulos como: Acut´ angulo si cada uno de los ´angulos interiores es agudo. Rect´ angulo si uno de sus ´angulos es recto. Obtus´ angulo si tiene un ´angulo obtuso.

212

Cap´ıtulo 3.

Geometr´ıa

En la figura 3.15 los tri´angulos (a) y (b) son acut´angulos y (c) es obtus´angulo. En la figura 3.16 se muestra un tri´angulo rect´angulo, los lados que forman el ´angulo recto se llaman catetos y el opuesto a dicho ´angulo se llama hipotenusa. C

hipotenusa

cateto

A

cateto

B

Figura 3.16: El tri´angulo △ABC con ´angulo recto en el v´ertice A. Una equivalencia del Quinto Postulado de Euclides concierne a los tri´angulos. Teorema 3.2.1 La suma de los ´angulos interiores de cualquier tri´angulo es 180◦ . Demostraci´on. Dado el tri´angulo △ABC, de la figura 3.17, se traza la recta M ←→ paralela a la recta AC, que pasa por B. De la figura se tiene α′ +β +γ ′ = 180◦ y del Teorema 3.1.1 se observa que α = α′ y γ = γ ′ . Por lo tanto, la suma de los ´angulos interiores es α + β + γ = 180◦ .  Dos tri´angulos son semejantes si sus v´ertices se pueden etiquetar como A, B, C y A′ , B ′ , C ′ , de manera que sus respectivos ´angulos sean congruentes. Esta asociaci´on de lados y por tanto tambi´en de ´angulos se llama correspondencia. La figura 3.18 ilustra el concepto de semejanza y lo caracteriza el siguiente teorema. Teorema 3.2.2 Sea una correspondencia entre dos tri´angulos semejantes △ABC y △A′ B ′ C ′ . Entonces se cumple BC CA AB = ′ ′ = ′ ′ . ′ ′ AB BC CA

3.2.

Pol´ıgonos

213

B Α'

M Γ'

Β

Α

Γ

A

C

L

Figura 3.17: Si L y M son paralelas, α + β + γ = 180◦ . C'

C'

C

A

B

C

A'

B'

A A'

B B'

Figura 3.18: Los tri´angulos semejantes △ABC y △A′ B ′ C ′ Demostraci´on. Primero se supone que los tri´angulos semejantes, △ABC y △A′ B ′ C ′ , son rect´angulos y se dibujan en el arreglo mostrado en la figura 3.19. Por la igualdad de ´areas por debajo y por encima de la diagonal del AB BC rect´angulo, se tiene (A′ B ′ )(BC) = (AB)(B ′ C ′ ) o sea ′ ′ = ′ ′ . Como AB BC (AB)2 (AB)2 + (BC)2 (AC)2 = = (A′ B ′ )2 (A′ B ′ )2 + (B ′ C ′ )2 (A′ C ′ )2 se obtiene la igualdad faltante al tomar ra´ız cuadrada. Si ahora los tri´angulos semejantes no son rect´angulos se considera el siguiente arreglo, ver figura 3.20. Al observar los tri´angulos rect´angulos, se tiene por lo anterior: DB BC AC DC AD = = ′ ′ = ′ ′ = ′ ′ ′ AD AC DC DB BC

214

Cap´ıtulo 3.

Geometr´ıa

C

C'

B A

A'

B'

Figura 3.19: Los tri´angulos rect´angulos semejantes △ABC y △A′ B ′ C ′ C

C'

A

A'

D

B'

B

Figura 3.20: Los tri´angulos semejantes △ABC y △A′ B ′ C ′ y por tanto, tambi´en AC AD AD + DB AB = = = A′ C ′ A′ D A′ D + DB ′ A′ B ′ lo cual concluye la prueba.  El siguiente resultado es quiz´a el m´as c´elebre de la geometr´ıa y se atribuye a Pit´agoras. Teorema 3.2.3 En un tri´angulo rect´ angulo, la suma de los cuadrados de las longitudes de los catetos es igual al cuadrado de la longitud de la hipotenusa. La figura 3.21 ilustra el significado del teorema de Pit´agoras. En el tri´angulo △ABC el ´area de los cuadrados levantados sobre sus catetos es igual al ´area del cuadrado levantado en la hipotenusa. Demostraci´on. La argumentaci´on se sigue de la figura 3.22.

3.2.

Pol´ıgonos

215

A

b2

c2 c

b a

B

C

a2

Figura 3.21: Teorema de Pit´agoras a2 + b2 = c2 .

b2 A

b c

c2 C

a

a2

a

c b

B

Figura 3.22: Los cuatro tri´angulos inscritos dentro del primer cuadrado se distribuyen en el per´ımetro del segundo cuadrado.

216

Cap´ıtulo 3.

Geometr´ıa

Dado un tri´angulo, una altura del tri´angulo es el segmento trazado perpendicularmente desde un v´ertice del tri´angulo a la l´ınea que contiene el lado opuesto al v´ertice. Un tri´angulo tiene tres diferentes alturas, como muestra la figura 3.23

h1 h2

h1 h1

h2

h2

h3 h3 h3

Figura 3.23: Las tres alturas de un tri´angulo.

Ejercicio 3.2.1 En la figura 3.24 los segmentos AC y DF son paralelos y el segmento BG es perpendicular al segmento AC. Si AC = 20, BE = 6, EG = 8 y AB = 8. Calcular las longitudes de los segmentos CG, EF , F G, F C, IC y AG, DE, DG, DA, AH . Soluci´on.√ √ √ 8 85 6 85 CG = 2 85, EF = 48 , F G = , F C = , IC = 7 √ 7 √7 32 8 6 24 DE = 7 , DG = 7 65, DA = 7 65, AH = 7 .

36 . 7

√ AG = 2 65,

Ejercicio 3.2.2 Los tri´angulos mencionados en este ejercicio son rect´angulos. 1. Calcular la hipotenusa si los catetos miden 9 y 13 unidades de longitud. 2. La hipotenusa mide 28 y un cateto 7. Calcular el otro cateto. 3. Calcular los catetos si la hipotenusa mide 6 y un cateto es el triple del otro cateto.

3.2.

Pol´ıgonos

217 G

D

A

F

E

H

B

I

C

Figura 3.24: Divisiones del tri´angulo 4. Un cateto mide 8 y la hipotenusa es 3 unidades m´as grande que el otro cateto. Calcular la hipotenusa y el cateto faltante. 5. Un cateto es el triple del otro cateto y la hipotenusa es 5 unidades m´as grande que el cateto m´as peque˜ no. Calcular las dimensiones del tri´angulo. Soluciones. √ 1. 5 10 √ 5. 5+59 10 ,



5+5 10 , 3

√ 50+5 10 9

√ 2. 7 15

3.

√ √ 3 10 9 10 , 5 5

4.

55 73 , 6 6

Ejercicio 3.2.3 Sea un tri´angulo rect´angulo con catetos de longitud 10 y 20. Sea h la altura que baja a la hipotenusa. La altura divide a la hipotenusa en dos segmentos. Sea a el segmento menor y b el segmento mayor. Hacer un dibujo que muestre la situaci´on y calcular las longitudes de h, a y b. Soluci´ on. √ √ √ h = 4 5, a = 2 5, b = 8 5. Ejercicio 3.2.4 Se da un tri´angulo rect´angulo con un cateto de longitud 15 y el otro de longitud b. Sea h la altura bajada a la hipotenusa, con h = 7. La altura divide a la hipotenusa en dos segmentos. Sea a el segmento que

218

Cap´ıtulo 3.

Geometr´ıa

hace v´ertice con el cateto de longitud 15 y c el otro segmento que completa la hipotenusa. Hacer un dibujo que muestre la situaci´on y calcular las longitudes de a, b y c. Soluci´ on. √ a = 4 11, b =

105 √ , 4 11

c=

49 √ . 4 11

Ejercicio 3.2.5 Un tri´angulo rect´angulo tiene un cateto de longitud 5 y el otro de longitud b. Sea h la altura bajada a la hipotenusa. La altura divide a la hipotenusa en dos segmentos. Sea a el segmento que hace v´ertice con el cateto de longitud 5 y c = 9 el otro segmento que completa la hipotenusa. Hacer un dibujo que muestre la situaci´on y calcular las longitudes de h, a y b. Soluci´ on.√ √ √ √ √ 3 h = 2 2 181 − 18, a = 12 ( 181 − 9), b = 32 18 + 2 181.

3.2.2.

Cuadril´ ateros

Despu´es de los tri´angulos, los pol´ıgonos m´as simples son los que tienen cuatro lados y se les llama cuadril´ ateros. A continuaci´on se definen diversos tipos de cuadril´ateros. Un trapecio es un cuadril´atero con solamente un par de lados paralelos. Si los dos pares de lados opuestos de un cuadril´atero son paralelos se llama paralelogramo, ver figura 3.25. Un rect´ angulo es un paralelogramo cuyos D

A

C

B E

H

G

F

Figura 3.25: El trapecio ABCD y el paralelogramo EF GH. ´angulos interiores son rectos, ver figura 3.26. Uno de sus lados recibe el nombre de base y un adyacente altura. En la figura 3.26, b = AB es la base y h = BC es la altura.

3.2.

Pol´ıgonos

219

D

C

H

G

A

B

E

F

Figura 3.26: El rect´angulo ABCD y el cuadrado EF GH. El ´ area de un rect´ angulo se define como el producto de las longitudes de su base por su altura

A = bh . Un cuadrado es un rect´angulo cuyos lados miden lo mismo y si la longitud de un lado es l entonces su ´area es

A = l2 . El ´area un un paralelogramo de base b y altura h es

A = bh . y el ´area de un tri´angulo de base b y altura h es

A=

bh . 2

seg´ un lo muestra la figura 3.27 Ejercicio 3.2.6 Un tri´angulo equil´atero tiene arista de longitud l. Determinar el valor de cualesquiera de sus alturas y el valor de su ´area. Soluci´ √on. h = 23 l, A =



3 2 l . 4

220

Cap´ıtulo 3.

Geometr´ıa

h

b

b

Figura 3.27: El ´area de un tri´angulo es la mitad del ´area de un paralelogramo. Ejercicio 3.2.7 Calcular el ´area de los siguientes tri´angulos is´osceles: 1. Los lados iguales miden 7 y el lado restante 5. 2. La altura que baja al lado desigual mide 7 y el lado desigual mide una vez y media lo que mide uno de los lados iguales. 3. El lado desigual mide 20 y la altura que baja a dicho lado mide 8. 4. Los lados iguales miden 5 cada uno y la altura que baja al lado desigual mide 3. Soluciones. 1.

√ 15 19 4

√ 2. A = 21 7

3. 80

4. 12

Ejercicio 3.2.8 Un tri´angulo is´osceles tiene lados iguales de longitud 20. La altura bajada a uno de los lados iguales mide 6. Determinar la longitud del lado faltante, la altura que baja a este lado y el ´area del tri´angulo. Soluci´ on √ √ √ √ l = 4 5(10 − 91), h = 2 5(10 + 91), A = 60. Ejercicio 3.2.9 Un tri´angulo is´osceles tiene ´area de 48 unidades cuadradas. La altura bajada al lado desigual mide 6. Determinar el per´ımetro del tri´angulo. Soluci´on P = 36.

3.2.

Pol´ıgonos

221

Ejercicio 3.2.10 En la figura 3.28 escribir las longitudes a y b de manera que se pueda argumentar la f´ormula del binomio (a + b)2 = a2 + 2ab + b2 .

Figura 3.28: (a + b)2 = a2 + 2ab + b2 . Ejercicio 3.2.11 En la figura 3.29 escribir las longitudes a y b de manera que se pueda argumentar la f´ormula (a + b)(a − b) = a2 − b2 .

Figura 3.29: (a + b)(a − b) = a2 − b2 . Ejercicio 3.2.12 En un rect´angulo de base 30 y altura 10 se inscribe un cuadril´atero cuyos lados unen los puntos medios de cada lado del rect´angulo. Calcular su per´ımetro P y su ´area A.

222

Cap´ıtulo 3.

Geometr´ıa

√ Soluci´ on. P = 20 10, A = 150 unidades cuadradas. Ejercicio 3.2.13 Usando la figura 3.30, calcular el ´area del trapecio ABCD al realizar la diferencia del ´area entre los tri´angulos △AEB y △CED. E

h1 D

C h2

A

B

Figura 3.30: El trapecio ABCD. Soluci´on. A =

3.3.

AB+CD h2 . 2

El c´ırculo

Una circunferencia con centro O y radio r es el conjunto de puntos que est´an a distancia r del punto O. La regi´on limitada por una circunferencia se llama c´ırculo, as´ı el per´ımetro del c´ırculo es la longitud de su circunferencia.

3.3.1.

El per´ımetro de un c´ırculo

Dada una circunferencia de radio 1, y puntos A y B en ella, se desea medir la longitud del arco de circunferencia, desde el punto A hasta el punto d Una manera de hacerlo es deslizar B, la cual se indica con la notaci´on l(AB). la circunferencia de la figura 3.31 sobre la l´ınea recta L, hasta que el punto B haga contacto con L, como muestra la figura 3.32. d es igual a la longitud del segmento OP , Entonces la longitud del arco AB d es decir l(AB) = OP . Si se desliza la mitad de la circunferencia de radio 1, es una convenci´on denotar esta longitud por el n´ umero π y num´ericamente es π = 3. 1415926535897932384626433832795028841971693993751 . . .

3.3.

El c´ırculo

223

B r=1

L

A O

d desliza sobre la recta L. Figura 3.31: El arco de circunferencia AB

A r=1

B O

L

P

d = OP . Figura 3.32: La longitud del arco de la circunferencia l(AB)

por lo tanto, el per´ımetro de un circunferencia de radio 1 es 2π, ver la figura 3.33. Para fines pr´acticos π = 3. 1416 Otra manera de calcular la longitud de un arco es usar aproximaciones poligonales. d un arco de la circunferencia C y Sean AB A = A0 , A1 , . . . , An = B d Se considera la poligonal inscrita Pn , una sucesi´on de puntos en el arco AB. formada por la sucesi´on de los segmentos A0 A1 , A1 A2 , . . . , An−1 An , cuya suma de longitudes es l(Pn ) = A0 A1 + A1 A2 + · · · + An−1 An ,

224

Cap´ıtulo 3.

O

Π

r=1

Geometr´ıa



Figura 3.33: El c´ırculo con centro O, radio r = 1 y per´ımetro 2π. ver la figura 3.34. Al aumentar el n´ umero n de lados de la poligonal, con lados cada vez m´as peque˜ nos, la cantidad l(Pn ) se aproxima m´as a la longitud de d arco l(AB). Si se contin´ ua indefinidamente este proceso, se obtiene la longitud de arco. Esto se simboliza como d = l´ım l(Pn ) . l(AB) n→∞

Se consideran ahora dos arcos con igual medida angular y mismo v´ertice O, uno de radio r y otro de radio r′ , seg´ un muestra la figura 3.34. Como A'i+1 A'i '

B =A'n

Ai+1 Ai

B=An

O

A=A0 r

A'=A'0 r'

Figura 3.34: Longitudes de arco de radios r y r′ . los tri´angulos is´osceles △Ai OAi+1 y △A′i OA′i+1 son semejantes se tiene para cada lado de las poligonales Pn y Pn′ A′i A′i+1 r′ = Ai Ai+1 r

por tanto

A′i A′i+1 =

r′ Ai Ai+1 r

3.3.

El c´ırculo

225

as´ı al sumar la longitud de cada poligonal se tiene r′ ′ B ′ ) = l´ d = l´ım l(Pn ) , l(A [ l(Pn′ ) = l(Pn ) luego, si l(AB) ım l(Pn′ ) n→∞ n→∞ r entonces ′B′) d [ l(AB) r′ d l(A ′B′) = [ l(AB) = , l(A por tanto r r′ r es decir la raz´on de las longitudes de los arcos a sus radios, subtendidos por el ′ B ′ ) = r ′ l(AB). d [ mismo ´angulo es constante. En particular si r = 1, se tiene l(A En otras palabras la longitud del arco subtendido por un ´angulo en una circunferencia de radio r′ es igual a la longitud del arco subtendido por el mismo ´angulo en una circunferencia de radio 1, multiplicada por r′ . Como un c´ırculo de radio 1 tiene per´ımetro 2π entonces el per´ımetro P , de un c´ırculo de radio r es

P = 2πr . As´ı, el di´ametro de un c´ırculo cabe exactamente π veces en su circunferencia. Para cada ´angulo ∠AOB se define una nueva medida del ´angulo, llamada longitud de arco. Para esto se considera el arco de circunferencia con centro en el v´ertice O y radio 1 que subtiende el ´angulo. La medida del ´angulo ∠AOB es la longitud s de este arco de circunferencia y se dice que es la medida del ´angulo en radianes. Con esta nueva medida se mantiene la adici´on de las medidas de ´angulos. Ejercicio 3.3.1 Hallar el complemento en radianes de los ´angulos siguientes: π 3π 1. 2. 3. 0. 984 4 7 Soluciones. 1.

π 4

2.

π 14

3. 0. 5868

Ejercicio 3.3.2 Hallar el suplemento en radianes de los ´angulos siguientes 7π 2π 2. 3. 2. 374 11 3 Soluciones. 1.

1.

4π 11

2.

π 3

3. 0. 767593

226

3.3.2.

Cap´ıtulo 3.

Geometr´ıa

El ´ area de un c´ırculo

Sea n un entero positivo. En un circunferencia de radio r y per´ımetro 2πr se marcan n puntos separados igualmente entre s´ı por una longitud de arco 2πr . La poligonal de n lados inscrita en la circunferencia se denomina igual a n pol´ıgono regular de n lados. Se triangula el pol´ıgono usando como v´ertice com´ un el centro del c´ırculo, a la altura com´ un de cada uno de los tri´angulos se le llama apotema. Su ´area est´a dada por

A=

pa , 2

donde p es el per´ımetro del pol´ıgono y a es la longitud del apotema, ver figura 3.35

O

a

Figura 3.35: Un pol´ıgono y su apotema

Teorema 3.3.1 El ´area de un sector circular de radio r subtendido por un ´angulo de s radianes es

1 A = rs . 2 Demostraci´on. Se considera un sector circular limitado por el ´angulo ∠AOB y un arco de radio r. Sea una sucesi´on de puntos A = A0 , A1 . . . , An = B, uniformemente distribuidos en el arco de circunferencia, con ∠AOB =

3.3.

El c´ırculo

227

d l(AB) . Se construye el sector poligonal inscrito Kn . El ´area Kn subtendida n por el pol´ıgono con lados OA, AA1 , A1 A2 , . . . , An−1 B y OB es Kn = n

AA1 · hn 2

donde hn es la altura com´ un de los tri´angulos, ver figura 3.36. El valor del ´area Ai+1 Ai B=An

hn

A=A0 O

r

Figura 3.36: La aproximaci´on del ´area de un sector circular K. del sector circular se obtiene al aumentar el n´ umero de lados de la poligonal a infinito; esto es K = l´ım Kn . n→∞

d y l´ım hn = r entonces Como l´ım nAA1 = l(AB) n→∞

n→∞

1 d K = rl(AB) 2

(3.3.1) 

Corolario 3.3.1 El ´area de un c´ırculo de radio r es

A = πr2 . Ejercicio 3.3.3 Calcular el ´area y el per´ımetro total del conjunto de figuras formado por √ √ 1. Un c´ırculo√de radio r = √ 5, un cuadrado de arista 3 π y un rect´angulo de base 8 π y altura 5 π.a

228

Cap´ıtulo 3.

Geometr´ıa

2. Un semic´ırculo de radio √ r = π, un tri´angulo equil´atero de arista un cuadrado de arista 4 π 3 .



π3 y

Soluciones.

√ √ 1. P = 2 π(19 + 5π), A = 54π √ √ 3 2. P = π(2 + 19 π + π), A = π4 (66 + 3)

Ejercicio 3.3.4 En un rect´angulo de base 20 y altura 4 se inscriben todos los posibles c´ırculos tangentes a dos lados paralelos del rect´angulo. Determinar el per´ımetro P y ´area de la figura formada. Soluci´on. P = 4(π + 8), A = 4(π + 16) unidades cuadradas.

3.4.

S´ olidos

Se estudian los cilindros, conos y esferas. Se dan sus definiciones y f´ormulas de volumen y superficie. Para el lector interesado la deducci´on de las f´ormulas del volumen y ´area superficial del cono y de la esfera se argumentan en la pr´oxima secci´on

3.4.1.

Cilindros

La introducci´on del concepto cilindro generalizado o simplemente cilindro y algunas de sus propiedades permitir´a abordar el c´alculo de vol´ umenes de los propios cilindros, conos y esferas. Sean P y P ′ dos planos paralelos y B ⊂ P un subconjunto acotado con ´area finita. Sea L una l´ınea que intersecta a P en s´olo un punto. A trav´es de cada punto p ∈ B, se toma un segmento paralelo a L, con punto inicial en B y punto final en P ′ . La uni´on K de todos estos segmentos se llama cilindro de base B y de directriz L. La intersecci´on del cilindro K con el plano P ′ se llama base superior o tapa. La distancia perpendicular entre los planos P y P ′ se llama altura del cilindro. En la figura 3.37 aparecen dos cilindros, cuya directriz es una l´ınea inclinada, uno de ellos es de base circular y es entonces un cilindro circular (inclinado) y el otro tiene de base un pent´agono, es decir es un prisma pentagonal (inclinado). Si la base de un cilindro es un pol´ıgono se llama prisma y si el pol´ıgono es paralelogramo se llama paralelep´ıpedo. El volumen de un cilindro se define como

3.4.

S´ olidos

R'

229

L h

R

Figura 3.37: Un cilindro con base circular y un cilindro con base poligonal

´ de la base) (altura) . V = (Area Sean K y K ′ dos conjuntos en el espacio que limitan sendos vol´ umenes. Sea P0 un plano. Si P es otro plano paralelo a P0 entonces los conjuntos K ∩ P y K ′ ∩ P se llaman secciones transversales paralelas a P0 , correspondientes a K y K ′ respectivamente. Teorema 3.4.1 Las secciones transversales de un cilindro tienen igual ´area. En la figura 3.38 se muestran tres cilindros, cuyo volumen es igual en virtud del principio de Cavalieri, el cual se enuncia a continuaci´on. Teorema 3.4.2 [Principio de Cavalieri]. Sean K y K ′ dos conjuntos en el espacio que limitan sendos vol´ umenes y P0 un plano fijo. Si para cada plano P , paralelo a P0 , las secciones transversales K ∩ P y K ′ ∩ P tienen la misma ´area, entonces los vol´ umenes de K y K ′ son iguales. Teorema 3.4.3 Si dos cilindros tienen igual altura y el ´area de su base es igual entonces tienen igual volumen. Corolario 3.4.1 a) El volumen de un cilindro circular de radio r y altura h es

V = πr2 h .

230

Cap´ıtulo 3.

Geometr´ıa

Figura 3.38: Principio de Cavalieri. b) El volumen de un prisma con ´area de su base igual a B y altura h es

V =B·h . Demostraci´on a) Se considera un cilindro de radio r y un prisma cuadrangular, con lado √ de la base r π; as´ı el ´area de la base B es πr2 y es igual al ´area de la base del cilindro, ver figura 3.39. Por el Principio de Cavalieri los vol´ umenes √ 2 son los mismos y el volumen del prisma cuadrangular es V = B · h = (r π) h = πr2 h. b) Si el prisma es oblicuo, se le asocia un prisma recto con igual base B y altura h, como muestra la figura 3.40. Por el Principio de Cavalieri los vol´ umenes son iguales. Despu´es se completa el prisma recto a un paralelep´ıpedo P cuyo volumen es V = 2B · h; de donde el volumen del prisma es V = B · h.  Se observa que los resultados anteriores siguen siendo v´alidos aunque las figuras tengan directriz oblicua. El ´area superficial de un cilindro, de base circular, se obtiene a partir de su desarrollo, seg´ un se muestra en la figura 3.41 y por tanto su ´area superficial es

S = 2πr2 + 2πrh .

3.4.

S´ olidos

231

h

h

r

r

Π

Figura 3.39: El ´area de cada base es πr2

h B

B

B

Figura 3.40: Comparaci´on del volumen del prisma con el de un paralelep´ıpedo Ejercicio 3.4.1 Determinar las f´ormulas del volumen y la superficie de un cilindro si 1. La altura del cilindro C1 es el triple del radio de la base. 2. La altura del cilindro C2 es igual al per´ımetro de la base. Para cada uno de los cilindros anteriores: 3. Escribir el radio en t´erminos del volumen y de la superficie del cilindro. 4. Escribir las f´ormulas del volumen obtenidas anteriormente en t´erminos de la superficie del cilindro.

232

Cap´ıtulo 3.

h

Geometr´ıa

h r r

Figura 3.41: Cilindro de radio r, altura h y generatriz g. 5. Escribir las f´ormulas de la superficie obtenidas anteriormente en t´erminos del volumen del cilindro. Soluciones. 1. V1 = √ 3πr3 , S1 = √ 8πr2 2. V2 = √ 2π 2 r3 , S2 =√ 2πr2 (1 + 2π) V1 S1 V2 S 3. r1 = 3 3π , r1 = 8π r2 = 3 2π 2 , r2 = 2π+4π 2 √ √ √ √ π ( S ) 32 2 2 3 √S 3 , 4. V1 = 16 V = 5. S1 = 8 3 πV9 , S2 = 3 2Vπ (1 + 2π) 2 2 1+2π 2π

3.4.2.

Conos

Sean P un plano, B ⊂ P un subconjunto de ´area finita y V un punto exterior a P . El cono con v´ertice V es la uni´on de todos los segmentos con punto inicial en B y punto final V . En la figura 3.42 aparece un cono de base circular y otro cuya base es un tri´angulo. Si la base de un cono es una poligonal recibe el nombre de pir´ amide. La altura h del cono es la distancia perpendicular del v´ertice V al plano P. Corolario 3.4.2 El volumen de un cono general con ´area de su base igual a B y altura h es 1 V = B·h . 3

3.4.

S´ olidos

233

V

V'

h

P

h

B

B'

Figura 3.42: Un cono y una pir´amide Corolario 3.4.3 El volumen de un cono circular, de radio r y altura h es

1 V = πr2 h . 3 Para un cono de radio r y altura h, su desarrollo superficial est´a en la figura 3.43. Por la f´ormula 3.3.1 su ´area superficial est´a dada por

√ S = πr2 + πrg = πr2 + πr r2 + h2 . Ejercicio 3.4.2 Determinar las f´ormulas del volumen y la superficie de un cono si: 1. La altura del cono C1 es el doble del radio de la base. 2. La altura del cono C2 es igual al doble del per´ımetro de la base. Para cada uno de los conos anteriores:

234

Cap´ıtulo 3.

2Πr

Geometr´ıa

g

g

g h

r r

Figura 3.43: Cono de radio r, altura h y generatriz g. 3. Escriba el radio en t´erminos del volumen y de la superficie del cono. 4. Escriba las f´ormulas del volumen obtenidas anteriormente en t´erminos de la superficie del cono. 5. Escriba las f´ormulas de la superficie obtenidas anteriormente en t´erminos del volumen del cono. Soluciones. √ 1. V1 = 32 πr3 , S1 = πr2 (1 + √ 5) √ 4 2 3 2 2. V2 = √ π r , S2 = πr (1 + 1 + 1 + 16π 2 ) √ 3 √ √ √ √ 5π+S− 5π S√ 3 3V √ √S 3. r1 = 3 3V r = = ; r = r = , , 1 2 2 2π 4π 2 5 π(1+ 5) π(1+ 1+16π 2 ) √ √ 3 3 4. V1 = 23 π(1+S√5)3 ; V2 = 43 (1+√πS 1+16π 2 )3 √ √ √ √ 2 3 9V 2 π 2 5. S1 = (1 + 5) , S2 = (1 + 1 + 16π ) 3 9V 4 16π

3.4.3.

Esferas

Dado un punto O y un n´ umero r > 0, la esfera de centro O y radio r es el conjunto de puntos que dista r del centro O, ver figura 3.44 Teorema 3.4.4 La superficie de una esfera de radio r es

3.4.

S´ olidos

235

r

O

Figura 3.44: Esfera de centro 0 y radio r.

S = 4πr2 . Teorema 3.4.5 El volumen de la regi´ on limitada por una esfera de radio r es

4 V = πr3 . 3 Ejercicio 3.4.3 Resolver los siguientes problemas. 1. Una esfera tiene volumen igual a 125 π 7 unidades c´ ubicas. Determinar el valor de su radio y de su superficie. 2. Una esfera tiene superficie igual a 169 π 5 unidades cuadradas. Determinar el valor de su radio y de su volumen. 3. Determinar el radio, superficie y volumen de una esfera cuyo volumen es igual a su superficie. Soluciones. √ 1. S = 25π 5 3 36

2. V =

2197 7 π 6

3. V = S = 36π

236

Cap´ıtulo 3.

Geometr´ıa

Ap´ endice de s´ olidos Dada una regi´on D del plano con ´area A, una manera de obtener su ´area es inscribiendo rect´angulos en su interior. Sea Rn la suma de las ´areas de todos los rect´angulos completamente inscritos dentro de D, seg´ un muestra la figura 3.45

Figura 3.45: La aproximaci´on por rect´angulos del ´area D. Cuanto m´as fina es la red de rect´angulos se obtiene una mejor aproximaci´on del ´area. La repetici´on infinita de este proceso da como resultado el ´area de la regi´on, as´ı A = l´ım Rn n→∞

Teorema 3.4.6 Sea K una pir´amide de base rectangular R0 y con altura h. Sea Rk la secci´on transversal de la pir´amide a altura k. Entonces ( ´ Rk = Area ´ R0 · Area

h−k h

)2 .

Demostraci´on Se considera la figura 3.46 y se calcula primero la proporcionalidad entre las longitudes AB y A′ B ′ .

3.4.

S´ olidos

237

V h-k C' Rk B' A' C

k

A

R0 B

Figura 3.46: Pir´amide de base rectangular y su secci´on transversal Como AB es paralelo a A′ B ′ , se tienen las siguientes relaciones por tri´angulos semejantes AB AV h = ′ = , ′ ′ AB AV h−k h−k h−k y an´alogamente B ′ C ′ = BC . Al multiplicar por tanto A′ B ′ = AB h h se tiene ( )2 h − k h − k h − k ′ ′ ′ ′ ´ ´ Area Rk = (A B )(B C ) = AB ·BC = Area R0 · .  h h h Teorema 3.4.7 Todos los conos con id´entica altura e igual ´area de la base, tienen el mismo volumen. Demostraci´on. Se considera ahora un cono K general, como el que se muestra en la figura 3.47.

238

Cap´ıtulo 3.

Geometr´ıa

h

Bk k

B0

Figura 3.47: La correspondencia entre las aproximaciones de las ´areas de la base B0 y de la secci´on transversal Bk del cono K.

3.4.

S´ olidos

239

As´ı h es la altura del cono, con base B0 y k la altura de la secci´on transversal Bk . La figura ilustra que una aproximaci´on por rect´angulos del ´area de B0 , se corresponde con una aproximaci´on por rect´angulos para el ´area de Bk , y rec´ıprocamente. Por el Teorema 3.4.6 cada rect´angulo individual Sk asociado la secci´on tranversal Bk cumple ( )2 h − k ´ ´ Area Sk = Area S0 · h donde S0 es el correspondiente rect´angulo en la base B0 . Sea Rn la suma del ´area de todos los rect´angulos inscritos en Bk y Tn la suma del ´area de todos los rect´angulos correspondientes en B0 , se tiene ( )2 h−k Rn = Tn · . h De manera que al continuar el proceso al infinito se tiene la siguiente relaci´on entre las ´areas ( )2 ( )2 h−k h−k ´ ´ Area Bk = l´ım Rn = l´ım Tn · = Area B0 · . n→∞ n→∞ h h Esto dice que todos los conos con id´entica altura e igual ´area de la base tienen secciones transversales con igual ´area. Por el principio de Cavalieri tienen el mismo volumen.  Se calcula ahora el volumen de un cono en particular: el de una pir´amide. Dado un prisma recto, ver figuras 3.48, 3.49, se descompone en tres pir´amides. As´ı, este prisma se puede expresar como la uni´on de tres pir´amides. Como △BCF ≈ △BEF , entonces P1 , P2 son pir´amides con v´ertice en A e igual altura, precisamente la distancia de A al plano BCF = BEF , por tanto vol P1 = vol P2 . Ahora P2 y P3 son pir´amides con v´ertice en F y como △ABE ≈ △ADE tienen igual base e igual altura, a saber, la distancia de F al plano ABE = ADF . As´ı vol P2 = vol P3 . Por tanto vol P = vol (P1 + P2 + P3 ) = 3 vol P2 y as´ı 1 1 vol K = ´area B · h . 3 3 El caso general se sigue de la figura y del Principio de Cavalieri, puesto que la base de toda pir´amide es triangulable. vol P2 =

240

Cap´ıtulo 3.

Geometr´ıa

A B D

C E

F

Figura 3.48: Prisma dividido en tres pir´amides A

B A

B

A

C E D

P1 F

F

P2

E F

P3

Figura 3.49: Las pir´amides que conforman al prisma de la figura 3.48 Teorema 3.4.8 El volumen de una pir´amide es un tercio del producto del ´area de su base por su altura. Los corolarios 3.4.2, 3.4.3 que dan los vol´ umenes de conos se obtienen del teorema anterior. Se procede ahora a calcular ahora el ´area de una esfera. Para esto, se aproxima el ´area de una esfera, por medio del ´area de troncos de conos, seg´ un muestra la figura 3.50. As´ı, es necesario calcular primero el ´area del tronco de un cono. Teorema 3.4.9 Sea C el tronco de un cono con r el radio de la base menor y R el radio de la base mayor. Si g es la generatriz del tronco, entonces el ´area lateral del tronco c´onico es A = π(R + r)g .

3.4.

S´ olidos

241

Figura 3.50: Aproximaci´on por troncos de cono de la superficie de una esfera Demostraci´on. Se considera la figura 3.51. El ´area del tronco es la diferencia entre las ´areas de los conos de radio R y r. As´ı A = πR(a + g) − πra. De los R rg a+g tri´angulos rect´angulos se tiene = . De aqu´ı a = y luego r a R−r ( ) rg rg πR2 g πr2 g A = πR(a + g) − πra = πR + g − πr = − R−r R−r R−r R−r πg = (R2 − r2 ) = π(R + r)g . R−r  Sup´ongase una esfera engendrada por la rotaci´on de una semicircunferend de la semicircunferencia, cia alrededor del eje x y se considera un arco AD ver la figura 3.52. Al girar alrededor del eje este arco engendra un cascar´on esf´erico. Se d una l´ınea poligonal regular ABCD. Al girar esta inscribe dentro del arco AD poligonal entorno al eje x engendra una aproximaci´on del cascar´on esf´erico por troncos c´onicos. Si se aumenta el n´ umero de lados de esta poligonal, el ´area de la superficie engendrada se aproxima a la superficie del cascar´on esf´erico. Se calcula el ´area de la superficie engendrada por la rotaci´on de la poligonal ABCD. El tronco de cono engendrado por el segmento AB tiene radio menor Aa

242

Cap´ıtulo 3.

Geometr´ıa

a r

g

R

Figura 3.51: La superficie del tronco de un cono. y radio mayor Bb, por lo tanto tiene una superficie lateral de ´ Area del tronco AB = π(Aa + Bb)(AB) = 2π(Ee)(AB) . Sea OE el apotema de la l´ınea poligonal para el lado AB y sea AF paralela al eje x. Entonces los tri´angulos ∆ABF y ∆EOe son semejantes, por tener AB AF sus lados perpendiculares dos a dos. As´ı = y luego OE Ee (Ee)(AB) = (OE)(AF ) = (OE)(ab) . Por tanto ´ Area del tronco AB = 2π(OE)(ab) . Por ser regular la l´ınea poligonal, la longitud del apotema OE es constante, as´ı ´ Area del tronco BC = 2π(OE)(bc) ´ Area del tronco CD = 2π(OE)(cd) . Sumando miembro a miembro estas igualdades, se tiene ´ Area engendrada por ABCD = 2π(OE)(ab + bc + cd) = 2π(OE)(ad).

3.4.

S´ olidos

243 B E

A

a

C D

F

e

b

O

c

d

Figura 3.52: Arco y poligonal Si se aumenta indefinidamente el n´ umero de lados de la poligonal, su proyecci´on seguir´a siendo ad, que es la altura h del cascar´on esf´erico, mientras que la longitud del apotema OE se iguala con el radio R del arco de la circunferencia y de la esfera misma. Por tanto ´ Area de la zona esf´erica = 2πRh. Al considerar h = 2R se obtiene el siguiente: Teorema 3.4.10 La superficie de una esfera de radio r es

S = 4πr2 . Sea una esfera de radio r y B la bola s´olida limitada por la esfera. Se desea calcular el volumen de la bola B. Para ello se considera a B, inscrita en un cilindro C de radio r y altura 2r, ver figura 3.53. Sea L = C \ B, es decir el volumen que queda en el cilindro despu´es de retirar a la bola B. As´ı C = B ∪ L. Se tiene que vol C = πr2 · 2r = 2πr3 , por tanto el volumen de la bola es: vol B = 2πr3 − vol L.

244

Cap´ıtulo 3.

Geometr´ıa

s k

r

r r

Figura 3.53: Bola inscrita en un cilindro y corte transversal. Se calcula ahora el volumen de L por medio del Principio de Cavalieri. De la figura 3.53 se tiene r2 = k 2 + s2 . A cada altura 0 < k < r el ´area de L es ´area Lk = πr2 − πs2 = π(r2 − s2 ) = πk 2 seg´ un muestra la figura 3.54, que es la vista superior. Ahora se calcula el Lk s 1

Figura 3.54: El anillo Lk . volumen L. Para esto se considera la figura 3.55, donde la secci´on transversal a altura k del cono es πk 2 , es decir la misma ´area de Lk . Por el Principio de Cavalieri 1 vol L = 2 · πr3 . 3 Retomando el c´alculo del volumen de la bola se tiene 4 2 vol B = 2πr3 − πr3 = πr3 . 3 3

3.4.

S´ olidos

245

r

r k k

k r r

r

Figura 3.55: Secci´on tranversal con ´area igual a ´area Lk .

Cap´ıtulo 4 Geometr´ıa anal´ıtica En este cap´ıtulo se estudian aspectos b´asicos de las ecuaciones que representan a rectas, circunferencias y par´abolas verticales en el plano cartesiano.

4.1.

El plano cartesiano

En un plano P se trazan dos rectas perpendiculares entre s´ı. La primer recta se elige horizontal y se denomina eje x, la segunda se llama eje y, el punto de intersecci´on O se denomina origen de coordenadas. Se sobrepone una regla para medir distancias y se hace corresponder a todo punto, de cada l´ınea, el n´ umero real que corresponde a su distancia al origen O. En el eje x los n´ umeros positivos est´an a la derecha del origen y en el eje y arriba del origen. Ambos ejes se llaman ejes coordenados. El plano con estos ejes de coordenadas recibe el nombre de plano cartesiano. Dado un punto P ∈ P, se trazan segmentos perpendiculares a los ejes coordenados a partir de P . Donde uno de estos segmentos intersecta al eje x se denomina abscisa del punto P y donde el otro segmento cruza al eje y se llama ordenada de P . Se escribe P (x, y) para indicar las coordenadas del punto P . Rec´ıprocamente, para ubicar el punto P (x, y) en el plano P, se mide, a partir del origen, la abscisa x en el eje horizontal y la ordenada y en el eje vertical. Al trazar segmentos perpendiculares a los ejes por estos puntos, se obtiene la posici´on del punto P en la intersecci´on, seg´ un muestra la figura 4.1. Ejemplo

4.1.1 Localizar los puntos A(2, 3), B(−1, 2), C(−3, −2) y D(4, −1) 247

248

Cap´ıtulo 4.

Geometr´ıa anal´ıtica

y

2

y

P Hx,yL

1

-2

-1

0

1

x

2

x

-1 -2

Figura 4.1: El punto P (x, y) en el plano cartesiano xy. en el plano cartesiano. Soluci´on. Para situar al punto A se ubica la abscisa 2 sobre el eje x y se traza por este punto, un segmento perpendicular al eje x, con longitud de tres unidades contadas a partir del eje x. En el extremo final de este segmento se encuentra el punto A(2, 3). Si se mide en el eje y el punto 3 y se traza un segmento perpendicular a este punto, entonces el punto A(2, 3) se encuentra en la intersecci´on de estas dos perpendiculares. Para los otros puntos se procede en forma similar, seg´ un se aprecia en la figura 4.2.  La figura 4.2, tambi´en muestra que los ejes coordenados dividen al plano en cuatro partes y cada una se llama cuadrante. Todo punto situado en el primer cuadrante tiene sus dos coordenadas positivas; todo punto situado en el tercer cuadrante tiene sus dos coordenadas negativas. En el segundo cuadrante la abscisa es negativa y la ordenada positiva y en el cuarto cuadrante la abscisa es positiva y la ordenada es negativa. Ejercicio 4.1.1 1. Localizar los siguientes puntos A(2, 5), B(0, 4), C(−3, 2), D(−1, −3), E(−2, 0), F (1, −3) en el plano cartesiano.

4.1.

El plano cartesiano

249 y

II

3 B

A

I

2 1

-3

-2

-1 0

1

2

D

-1 C

x

3

-2

III

IV

Figura 4.2: Los puntos A, B, C, D en sus respectivos cuadrantes. 2. Tres v´ertices de un cuadrado son los puntos A(−2, −3), B(4, −3), C(4, 3). Localizar el cuarto v´ertice y determinar sus coordenadas. Calcular la longitud de una de sus diagonales usando el teorema de Pit´agoras, ver el Teorema 3.2.3. Dados dos puntos A y B en el plano cartesiano, se recuerda que la distancia entre ellos es la longitud del segmento recto que los une; esta distancia se denota ahora por d(A, B). Si sus coordendas son A(x1 , x2 ) y B(x2 , y2 ) entonces de la figura 4.3 y el teorema de Pit´agoras (ver el Teorema 3.2.3) se tiene d(A, B) =

√ (x2 − x1 )2 + (y2 − y1 )2 .

El punto medio entre los puntos A y B est´a dado por (x, y) donde x=

x1 + x2 2

y=

y1 + y2 . 2

Ejemplo

4.1.2 Hallar la distancia entre los puntos A(−3, 1) y B(5, −4) y su punto medio. Soluci´ on Se identifican (x1 , y1 ) = (−3, 1) y (x2 , y2 ) = (5, −4), as´ı √ √ √ d(A, B) = (5 − (−3))2 + (−4 − 1)2 = 82 + (−5)2 = 89

250

Cap´ıtulo 4.

Geometr´ıa anal´ıtica

BHx2 ,y2 L y2

y1 AHx1 ,y1 L 0

CHx2 ,y1 L

x1

x2

Figura 4.3: Los puntos A, B y C forman un tri´angulo rect´angulo y −3 + 5 =1 2

1 + (−4) 3 =− . 2 2 3 Luego el punto medio del segmento es (1, − ). 2 x=

4.2.

y=



La l´ınea recta

En esta secci´on se definen propiedades b´asicas de una recta y se obtienen las diversas ecuaciones de una recta.

4.2.1.

Inclinaci´ on y pendiente de una recta

En la figura 4.4 se ha dibujado el eje horizontal x en el plano cartesiano, una recta L no paralela a ´el y el ´angulo de inclinaci´on θ de la recta, definido como el ´ angulo que forma el rayo derecho del eje de las x y el rayo de la recta contenido en el semiplano superior . La medida del ´angulo de inclinaci´on es a partir del rayo derecho del eje de las x y en sentido contrario al movimiento de las manecillas del reloj. La pendiente de la recta L es la tangente de su ´ angulo de inclinaci´ on θ y se denota con la letra m, es decir m = tan θ .

4.2.

La l´ınea recta

251

L

Θ

x

Figura 4.4: El ´angulo de inclinaci´on de la recta L Se recuerda que por dos puntos distintos se puede trazar s´olo una l´ınea recta. Sean dos puntos A (x1 , y1 ) y B (x2 , y2 ) en el plano cartesiano, con x1 ̸= x2 ; se calcula ahora la pendiente de la recta que pasa por estos puntos. Para ello sea el punto C(x2 , y1 ), se completa el tri´angulo rect´angulo ABC con AC paralelo al eje x y CB paralelo al eje y, como muestra la figura 4.5. y

BHx2 ,y2 L

Θ AHx1 , y1 L

CHx2 ,y1 L Θ x

Figura 4.5: C´alculo de la pendiente de una recta As´ı, la pendiente de la recta que pasa por los puntos A(x1 , y1 ) y B(x2 , y2 ) est´a dada por: tan θ = Resumiendo

y2 − y1 CB = AC x2 − x1

es decir

m=

y2 − y1 . x2 − x1

252

Cap´ıtulo 4.

m=

Geometr´ıa anal´ıtica

y2 − y1 . x2 − x1

Si x1 < x2 y la pendiente es positiva se tiene necesariamente y1 < y2 . Luego la recta est´a inclinada a la derecha, respecto a la direcci´on vertical, como muestra la figura 4.5. Si la pendiente es negativa entonces la recta est´a inclinada a la izquierda. Ejemplo

4.2.1 Hallar la pendiente de la recta que pasa por los puntos (1, 2), (3, 4) y determinar el ´angulo de inclinaci´on de la misma. Soluci´on Se puede elegir libremente el primer o el segundo punto. Sean (1, 2) = (x1 , y1 ) y (3, 4) = (x2 , y2 ), es decir x1 = 1, y1 = 2, x2 = 3, y2 = 4. La pendiente est´a dada por 4−2 2 y2 − y1 = = =1. x 2 − x1 3−1 2

m=

As´ı, la pendiente de la recta es 1 y su ´angulo de inclinaci´on es 45◦ , ya que tan θ = 1,

y por lo tanto

θ = arctan 1 = 45◦ ,

el cual se obtiene con la ayuda de una calculadora, en el modo de grados (deg en ingl´es). La recta est´a inclinada a la derecha.  Ejercicio 4.2.1 Calcular la pendiente y el ´angulo de inclinaci´on de la recta que pasa por los puntos indicados: ( ) 1 1. (−1, −3), (2, 4) 2. (1, 2), −3, − 3. (5, 0), (−3, 2) 2 4. (1, 1), (4, −4) Soluciones. 7 , 66◦ 48′ 3 5 4. − , 120◦ 57′ 50′′ . 3 1.

2.

5 , 32◦ 19′′ 8

1 3. − , 165◦ 57′ 50′′ 4

4.2.

La l´ınea recta

4.2.2.

253

Ecuaci´ on de la recta que pasa por dos puntos

Sea L una recta que pasa por los puntos A(x1 , y1 ), B(x2 , y2 ) con pendiente y2 − y1 m= . Si P (x, y) es un punto arbitrario de la misma recta entonces x2 − x1 y − y1 se tiene tambi´en que m = . Como las pendientes de los segmentos AB x − x1 y AP son iguales se tiene

y2 − y1 y − y1 = . x − x1 x2 − x1

(4.2.1)

As´ı un punto P (x, y) est´a en la recta L si y s´olo si cumple la ecuaci´on (4.2.1) la cual se llama la ecuaci´ on de la recta que pasa por dos puntos dados. Ejemplo

4.2.2 Obtener la ecuaci´on de la recta que pasa por los puntos (3, 2) y (9, 8). Soluci´ on. Como los puntos se pueden elegir libremente, sean (x1 , y1 ) = (3, 2) y (x2 , y2 ) = (9, 8). Sustituyendo en la ecuaci´on (4.2.1) se tiene y−2 8−2 6 = = =1 x−3 9−3 6

o sea

y−2 =1. x−3

Despejando y−2=x−3

se tiene

y =x−1 .

As´ı, y = x − 1, es la ecuaci´on de la recta que pasa por los puntos (3, 2) y (9, 8). Se observa que si x = 3 se sustituye en la ecuaci´on de la recta se obtiene y = 2, comprobando que el punto (3, 2) est´a en la recta. Ahora, si x = 0 la ecuaci´on da y = −1, es decir el punto (0, −1) est´a tambi´en en la recta, como se aprecia en la figura 4.6, que muestra la gr´afica de la recta. Una forma equivalente de escribir la ecuaci´on de la recta y = x − 1 es x − y − 1 = 0.  En conclusi´on al concepto geom´etrico de l´ınea recta se le ha asociado una expresi´on algebraica, en este caso una ecuaci´on, que contiene toda la informaci´on concerniente a la recta, en particular da la relaci´on que satisfacen las coordenadas de todos los puntos que pertenecen a la recta.

254

Cap´ıtulo 4.

Geometr´ıa anal´ıtica

H9,8L

8

y=x-1

6

4

H3,2L

2

2

-2

4

6

8

10

H0,-1L -2

Figura 4.6: La recta y = x − 1 . Ejercicio 4.2.2 Dar la ecuaci´on de la recta que pasa por los puntos indicados: 1. (−4, −1) y (5, 3) ( 4.

1 1 , 2 4

2. (−6, 0) y (4, −4)

3. (2, 4) y (4, 1)

5. (3, 1) y (3, −3)

6. (3, 5) y (6, 5)

) y (2, −3)

7. (7, 0) y (0, −3)

8.

La ecuaci´on del eje x

Soluciones 1. −4x + 9y − 7 = 0

2. 2x + 5y + 12 = 0

3. 3x + 2y − 14 = 0

4. 13x + 6y − 8 = 0

5.

6. y = 5

No aplica la f´ormula

7. −3x + 7y + 21 = 0

4.2.3.

8. y = 0

Ecuaci´ on de la recta punto-pendiente

Sup´ongase que de una recta se conoce su pendiente m y un punto (x1 , y1 ) y2 − y1 por el cual pasa. Si en la ecuaci´on (4.2.1) se sustituye por el valor m x 2 − x1

4.2.

La l´ınea recta

se tiene:

255

y − y1 =m. x − x1

De donde

y − y1 = m(x − x1 ) .

(4.2.2)

A esta ecuaci´on de la recta se le denomina punto-pendiente. As´ı, para determinar la ecuaci´on de una recta es suficiente conocer su pendiente m y un punto (x1 , y1 ) de la misma. Ejemplo

4.2.3 Obtener la ecuaci´on de la recta con pendiente 3 y que pasa por el punto (2, 4). Soluci´ on. En este caso se conoce la pendiente m = 3 y un punto por donde pasa la recta, a saber (2, 4) = (x1 , y1 ). Sustituyendo en la ecuaci´on (4.2.2) se tiene y − 4 = 3(x − 2) o sea y − 4 = 3x − 6 , y reduciendo −3x + y + 2 = 0 . Una forma equivalente de la ecuaci´on es 3x − y − 2 = 0.



Ejercicio 4.2.3 Obtener la ecuaci´on de la recta con pendiente m y que pasa por el punto indicado. 1 y (0, 5) 3

1. m = −2 y (2, 3)

2. m = −5 y (−3, 5)

3. m = −

4. m = 4 y (−1, −2)

5. m = 4 y (0, 0)

6. m = 0 y (1, −1)

Soluciones 1. 2x + y − 7 = 0

2. 5x + y + 10 = 0

3. x + 3y − 15 = 0

4. −4x + y − 2 = 0

5. y = 4x

6. y = −1

256

4.2.4.

Cap´ıtulo 4.

Geometr´ıa anal´ıtica

Ecuaci´ on de la recta de pendiente-ordenada

Ahora, si la recta tiene pendiente m y pasa por el punto (0, b) la ecuaci´on y − y1 = m(x − x1 ) se reduce a: y − b = m(x − 0)

luego

y − b = mx

o sea

y = mx + b .

(4.2.3)

Esta ecuaci´on es conocida como la forma pendiente m y ordenada al origen b. Se observa que y est´a despejada; en este caso la pendiente de la recta es el coeficiente de x y cuando x = 0 se obtiene la ordenada al origen y = b. Ejemplo

4.2.4 Obtener la ecuaci´on de la recta con pendiente −6 y que pasa por el punto (0, 2). Soluci´on. La pendiente es m = −6 y la ordenada al origen es b = 2. Sustituyendo en la ecuaci´on (4.2.3) se tiene y = −6x + 2 .  Ejemplo

1 4.2.5 Dar la pendiente de la recta y = − x + 8. 3

Soluci´on. La ecuaci´on est´a escrita en la forma pendiente-ordenada, luego la 1 pendiente es el coeficiente de x, as´ı m = − .  3 Ejercicio 4.2.4 Dar la ecuaci´on de la recta de pendiente m y ordenada al origen indicados. 1. m = 3,

b = −1

4. m = 0, b = −2

1 2. m = − , b = 0 6

3. m = −4, b =

1 5

Soluciones. 1. y = 3x − 1

1 2. y = − x 6

3. y = −4x +

1 5

4. y = −2

4.2.

La l´ınea recta

4.2.5.

257

Rectas horizontales

Una recta horizontal es una recta paralela al eje x, luego todos sus puntos son de la forma (x, b). Al sustituir en (4.2.1) los puntos distintos (a, b) y (c, b) su ecuaci´on es: y−b b−b = =0 x−a a−c

o sea

y−b =0. x−a

De aqu´ı y − b = 0, por lo que

y=b. Todos los puntos de la recta tienen la misma ordenada y = b. Se observa que la pendiente de una recta paralela al eje x es cero. Ejemplo

4.2.6 Dar la ecuaci´on de la recta paralela al eje x que pasa por el punto (−1, −1). Soluci´ on. La ecuaci´on es de la forma y = b, en este caso y = −1.



Ejercicio 4.2.5 Dar la ecuaci´on de la recta horizontal y que pasa por el punto indicado. 1. (2, 3) ( 6.



1 2, 4

2. (−1, 6)

3. (−4, 7)

4. (π, 5)

5. (0, −5)

)

Soluciones. 1. y = 3 6. y =

1 4

2. y = 6

3. y = 7

4. y = 5

5. y = −5

258

4.2.6.

Cap´ıtulo 4.

Geometr´ıa anal´ıtica

Rectas verticales

Una recta es vertical si es paralela al eje y; sus puntos son de la forma (a, y). Se eligen dos puntos de la recta (a, b), (a, c) y se sustituyen en la ecuaci´on (4.2.1). As´ı c−b y−b = x−a a−a donde el cociente del lado derecho no tiene sentido, ya que hay una divisi´on entre cero. La pendiente de la recta no est´a definida. Al escribir la ecuaci´on (4.2.1) en la forma x − x1 x2 − x1 = y − y1 y2 − y1 y sustituir los puntos anteriores, se tiene x−a a−a = =0 y−b c−b

o sea

x−a=0 .

Por tanto la ecuaci´on de la recta es

x=a. Luego todos sus puntos tienen la misma abscisa y por lo tanto, se trata de una recta paralela al eje y. Ejemplo

4.2.7 Dar la ecuaci´on de la recta vertical que pasa por el punto

(3, 4). 

Soluci´on. La ecuaci´on es de la forma x = a, en este caso x = 3.

Ejercicio 4.2.6 Dar la ecuaci´on de la recta paralela al eje y y que pase por el punto dado. ( ) 1 , −1 1. (5, −3) 2. (−1, 0) 3. (−2, 4) 4. (−15, 8) 5. 2 √ 6. ( 3, 1) Soluciones. 1. x = 5 2. x = −1 √ 6. x = 3

3. x = −2

4. x = −15

5. x =

1 2

4.3.

Ecuaci´ on general de la recta

4.3.

259

Ecuaci´ on general de la recta

En esta secci´on se ve que la ecuaci´on ax+by +c = 0, con a y b no simult´aneamente cero, siempre es la ecuaci´on de una recta. Caso 1. a ̸= 0, b = 0. Ahora ax + by + c = 0 se reduce a: ax + c = 0 ax = −c c x=− a Se trata de una recta paralela al eje y, que pasa por los puntos con abscisa c x=− . a Caso 2. b ̸= 0. ax + by + c = 0 by = −ax − c a c y =− x− b b a c Ahora es una recta de pendiente m = − , con ordenada al origen y = − . b b La ecuaci´on

ax + by + c = 0 se conoce como la ecuaci´ on general de una recta. Ejemplo

4.3.1 Sea la recta definida por la ecuaci´on 2x − 3y + 6 = 0.

Obtener: a) Su pendiente e inclinaci´on. b) Sus puntos de intersecci´on con los ejes coordenados. c) Su gr´afica. Soluci´ on. Se compara la recta dada con la forma general ax + by + c = 0.

260

Cap´ıtulo 4.

Geometr´ıa anal´ıtica

2 2 a = − = . Como su signo b −3 3 es positivo la recta est´a inclinada a la derecha. El mismo resultado se obtiene despejando y de la ecuaci´on original.

a) La pendiente de la recta es m = −

b) La recta intersecta al eje x cuando y = 0, ya que los puntos del eje x son de la forma (x, 0). Luego, sustituyendo y = 0 en la ecuaci´on de la recta se tiene: 2x − 3(0) + 6 = 0 2x + 6 = 0 2x = −6 x = −3 As´ı, la recta intersecta al eje x en el punto (−3, 0). La recta intersecta al eje y cuando x = 0, ya que los puntos del eje y son de la forma (0, y). Sustituyendo x = 0 en la ecuaci´on de la recta se tiene 2(0) − 3y + 6 = 0 −3y + 6 = 0 −3y = −6 −6 y= =2 −3 Por lo tanto, la recta intersecta al eje y en (0, 2). La informaci´on anterior se presenta en la siguiente tabla x y −3 0 0 2

punto (−3, 0) (0, 2)

c) Para obtener la gr´afica se ubican sobre los ejes los dos puntos de intersecci´on con los ejes coordenados y se traza la recta que pasa por ellos. El dibujo de la recta aparece en la figura 4.7. Ejercicio 4.3.1 Graficar y dar la pendiente de las siguientes rectas. 1. x + 4y + 8 = 0 4. −5x + 3y = 0

2. 3x − 4y + 12 = 0

3. 4x + 2y + 3 = 0

4.4.

Intersecci´ on de rectas

261

2.5

2.0

H0,2L

1.5

2x-3y+6=0 1.0

0.5

H-3,0L -4

-3

-2

1

-1 -0.5

Figura 4.7: Gr´afica de la recta 2x − 3y + 6 = 0. 5. Hallar la ecuaci´on de la recta que pasa por el punto (3, 5) y cuya abscisa al origen (intersecci´on con el eje x) es el doble que la ordenada al origen. Graficar la recta. Soluciones. 1 4 5. x + 2y − 13 = 0 1. m = −

4.4.

2. m =

3 4

3. m = −2

4. m =

5 3

Intersecci´ on de rectas

Dos rectas no paralelas tienen un punto de intersecci´on. Para determinarlo se resuelve el sistema de ecuaciones lineales que forman las ecuaciones de dichas rectas. Ejemplo

4.4.1 Dadas las rectas 3x − y − 7 = 0, 2x + y − 8 = 0 hallar la intersecci´on de ´estas y su representaci´on gr´afica. Soluci´ on. Al resolver el sistema de ecuaciones que forman se tiene 3x − y = 7 2x + y = 8 5x = 15 x = 3 .

262

Cap´ıtulo 4.

Geometr´ıa anal´ıtica

Sustituyendo el valor de x en la segunda ecuaci´on del sistema: 2(3) + y 6+y y y

=8 =8 =8−6 =2.

Por lo tanto, las rectas se intersectan en el punto (3, 2), ver figura 4.8. Para graficar la recta 3x − y − 7 = 0 se considera −y = −3x + 7

y = 3x − 7

o sea

con la tabulaci´on x y punto 2 −1 (2, −1) 0 −7 (0, −7) Para la segunda recta 2x + y − 8 resulta x y 0 8 2 4

= 0 se tiene y = −2x + 8; tabulando punto (0, 8) (2, 4)



10

H0,8L

y=3x-7 H2,4L

5

H3,2L 1

2

3

4

5

6

H2,-1L

y=8-2x -5

H0,-7L

Figura 4.8: La intersecci´on de las rectas 3x − y − 7 = 0, 2x + y − 8 = 0. Si dos rectas son paralelas el sistema formado por sus ecuaciones no tiene soluci´on, como lo muestra el siguiente ejemplo.

4.4.

Intersecci´ on de rectas

263

Ejemplo

5 4.4.2 Dadas las rectas 5x + 2y − 8 = 0, y = − x hallar la 2 intersecci´on de ellas y la representaci´on gr´afica del sistema lineal. Soluci´ on. Se resuelve el sistema de ecuaciones 5x + 2y − 8 = 0 5 y=− x 2

5 sustituyendo y = − x en la primera ecuaci´on se tiene 2 ( ) 5 5x + 2 − x − 8 = 0 2 5x − 5x − 8 = 0 −8 = 0 , lo cual es absurdo. Por tanto las rectas no se intersectan, es decir, no hay un punto (x, y) que pertenezca a las dos rectas, luego las rectas son paralelas, ver figura 4.9. Para graficar la primera recta se tiene 5x + 2y − 8 = 0

o sea

5 y =− x+4 2

y se le asocia la tabla x 0 2 5 y para la recta y = − x se tiene 2 x 0 -2

y punto 4 (0, 4) −1 (2, −1) la tabla y 0 5

punto (0, 0) (−2, 5) 

264

Cap´ıtulo 4.

Geometr´ıa anal´ıtica

10 8 6

H-2,5L 4

H0,4L

5x y=4-

5x y=-

2

2

2 -3

-2

-1

H0,0L

1

2

H2,-1L

3

-2

5 2

Figura 4.9: Las rectas paralelas 5x + 2y − 8 = 0, y = − x. Ejercicio 4.4.1 Hallar la intersecci´on de las rectas en los siguientes incisos. Graficar ambas rectas. 1. x + y − 4 = 0; −x + 4y − 6 = 0

2. 2x − 3y − 12 = 0; 7x + 6y = 9

3. 5x − 2y = 3; 6x − 2y = 2

4. −2x + 5y = 4; 6x − 15y = 1

5. 2x + y − 5 = 0; y − 3 = 0

6. 5x − 3y = 4; x = −4

7. 3x − 7 = 0; 2y + 4 = 0

8. x = 2; y = 5

Soluciones. 1. (2, 2)

2. (3, −2)

5. (1, 3)

6. (−4, −8)

4.5.

3. (−1, −4) 7 7. ( , −2) 3

4. No se intersectan 8. (2, 5)

Rectas paralelas

En esta parte se obtiene una condici´on para que dos rectas sean paralelas. Sean Ax + By + C = 0 y ax + by + c = 0 dos rectas no verticales, as´ı B ̸= 0, b ̸= 0. Despejando y de ambas ecuaciones se tiene A C y =− x− B B

;

a c y =− x− , b b

4.5.

Rectas paralelas

265

al igualar C a c A − x− =− x− b b ( B )B A a c C − . x= − B b b B Las rectas se intersectan, si y s´olo si, es posible despejar el valor de x, para A a ello se requiere que, ̸= , es decir las pendientes de las rectas deben ser B b distintas. Por lo tanto las rectas no se intersectan, o sea son paralelas, y esto sucede, si y s´olo si, sus pendientes son iguales. Resumiendo Teorema 4.5.1 Dos rectas no verticales son paralelas, si y s´olo si, sus pendientes son iguales. Ejemplo

4.5.1 Obtener la ecuaci´on de la recta que pasa por el punto (3, −3) y es paralela a la recta x + 2y − 5 = 0. Soluci´ on. Para determinar la ecuaci´on de la recta se necesita conocer un punto de ´esta y su pendiente. El punto es (3, −3). Como las rectas deben ser paralelas las pendientes deben ser iguales. La pendiente de la recta dada 1 5 1 1 y = − x + es m1 = − , por lo tanto la pendiente buscada es m2 = − . 2 2 2 2 As´ı, la ecuaci´on pedida es: 1 y − (−3) = − (x − 3) 2 1 y + 3 = − (x − 3) 2 2(y + 3) = −x + 3 2y + 6 = −x + 3 x + 2y + 3 = 0 . Ejemplo



4.5.2 Verificar que las rectas −4x + 3y = 1, 8x − 6y + 7 = 0 son paralelas. 4 −4 = y la de la Soluci´ on. La pendiente de la primera recta es m1 = − 3 3 8 4 segunda recta es m2 = − = . Como las pendientes son iguales, las −6 3 rectas son paralelas. 

266

Cap´ıtulo 4.

Geometr´ıa anal´ıtica

Ejercicio 4.5.1 Obtener la ecuaci´on de la recta que pasa por el punto dado y es paralela a la recta indicada. 1. (2, −1); −3x + 4y − 1 = 0 3. (3, −4); x + 3y − 2 = 0 5. (−6, 1); y = 8

2. (0, 0); 2x − 5y + 6 = 0 4. (5, −2); 2x − 1 = 0

Soluciones. 1. 3x − 4y − 10 = 0 4. x = 5

2. 2x − 5y = 0 5. y = 1

3. x + 3y + 9 = 0

Ejercicio 4.5.2 De las siguientes rectas determinar las que son paralelas. 1. 2x − 5y + 1 = 0 2. 5x − 2y + 4 = 0 4. −2. 5x + y + 2 = 0 5. 4x − 10y + 3 = 0

3. −3x + 8y + 1 = 0

Soluci´on. 1 y 5 ; 2 y 4 son paralelas. Ejercicio 4.5.3 Dibujar el tri´angulo con v´ertices en A(−4, 3), B(−2, −7) y C(6, −3). Obtener: a) El punto medio B ′ del segmento BC. b) El punto medio C ′ del segmento CA. c) La ecuaci´on de la recta que pasa por B ′ C ′ y su gr´afica. d) La ecuaci´on de la recta que pasa por AB. e) ¿C´omo son las rectas obtenidas en los incisos c y d ? Soluciones. a) (2, −5), b) (1, 0), c) y = −5x + 5 d) 5x + y + 17 = 0 e) son paralelas.

4.6.

Rectas perpendiculares

267

y= y0 +m1 Hx-x0 L

RHx0 -1, y0 -m2 L

QHx0 +1, y0 +m1 L

PHx0 ,y0 L

y= y0 +m2 Hx-x0 L

Figura 4.10: La condici´on de perpendicularidad.

4.6.

Rectas perpendiculares

Se consideran las rectas que pasan por el punto P (x0 , y0 ) con pendientes m1 y m2 diferentes de cero, y se desea determinar la condici´on para que las rectas y = y0 + m1 (x − x0 ), y = y0 + m2 (x − x0 ) sean perpendiculares. Para esto se observa que el punto Q(x0 +1, y0 +m1 ) pertenece a la recta y = y0 +m1 (x−x0 ) y el punto R(x0 − 1, y0 − m2 ) pertenece a la recta y = y0 + m2 (x − x0 ), ve´ase la figura 4.10. Las rectas son perpendiculares si los puntos P, Q, R son los v´ertices de un tri´angulo rect´angulo, es decir cumplen P Q2 + P R2 = QR2 . (x0 + 1 − x0 )2 +(y0 + m1 − y0 )2 + (x0 − 1 − x0 )2 + (y0 − m2 − y0 )2 = (x0 + 1 − (x0 − 1))2 + (y0 + m1 − (y0 − m2 ))2 o sea 1 + m21 + 1 + m22 = 4 + m21 + 2m1 m2 + m22 −2 = 2m1 m2 −1 = m1 m2 Teorema 4.6.1 Dos rectas con pendientes m1 , m2 diferentes de cero son perpendiculares, si y solamente si, el producto de sus pendientes es −1. Es decir

m1 m2 = −1 . Equivalentemente

268

Cap´ıtulo 4.

m2 = −

Geometr´ıa anal´ıtica

1 . m1

Ejemplo

4.6.1 Dar la ecuaci´on de la recta que pasa por el punto (−1, 3) y es perpendicular a la recta 5x + 2y − 3 = 0. 5 a Soluci´on La pendiente de la recta dada 5x + 2y − 3 = 0 es m1 = − = − b 2 1 y cualquier recta perpendicular a ella tiene pendiente m2 = − . As´ı m1 m2 = −

1 2 = . 5 5 − 2

2 Luego, la recta buscada tiene pendiente m = y pasa por el punto (−1, 3), 5 entonces la ecuaci´on es: 2 y − 3 = (x + 1) 5 5(y − 3) = 2(x + 1) 5y − 15 = 2x + 2 −2x + 5y − 17 = 0 .



Ejemplo

4.6.2 Pruebe que el tri´angulo con v´ertices A(−4, 2), B(3, −3) y C(8, 4) es rect´angulo. Soluci´on. Se calculan las pendientes de sus lados para determinar si se cumple la condici´on de perpendicularidad para alg´ un par de lados. Las pendientes del segmento AB y del segmento BC son mAB =

−5 −3 − 2 5 = =− 3+4 7 7

mBC =

4+3 7 = . 8−3 5

4.6.

Rectas perpendiculares

269

Las pendientes son rec´ıprocas con signo contrario, por lo tanto los lados AB y BC son perpendiculares y el tri´angulo es rect´angulo, con su ´angulo recto en el punto B.  Ejercicio 4.6.1 Dibujar el tri´angulo con v´ertices en A(−4, 3), B(−2, −7) y C(6, −3). a) Determinar los puntos medios A′ , B ′ , C ′ de los segmentos AB, BC y CA respectivamente. b) Obtener la ecuaci´on de la recta que cumple las condiciones dadas. i) Pasa por A ′ y es perpendicular al segmento AB. ii) Pasa por B ′ y es perpendicular al segmento BC. iii) Pasa por C ′ y es perpendicular al segmento CA. c) Graficar las rectas de los incisos i), ii), iii). ¿Qu´e puede afirmar de estas rectas? Calcular la intersecci´on de las tres rectas. Soluci´ on. a) A′ (−3, −2), B ′ (2, −5), C ′ (1, 0). b) i)−x + 5y + 7 = 0, ii) 2x + y = −1 iii) −5x ( + 3y =)−5. 2 15 c) Las tres rectas se intersectan en el punto ,− 11 11 Ejercicio 4.6.2 Determinar si el tri´angulo con v´ertices A(0, 0), B(−2, 5), C(5, 2) es tri´angulo rect´angulo. Soluci´ on. Es un tri´angulo rect´angulo, el ´angulo recto est´a en el origen. Ejercicio 4.6.3 Determinar si las rectas 4y − 8x − 1 = 0, son perpendiculares.

2y + x − 5 = 0

Soluci´ on. Las rectas son perpendiculares. Ejercicio 4.6.4 Obtener la ecuaci´on de la recta que contiene al punto (−2, 5) y es perpendicular a la recta indicada: a) x = 3. b) y + 9 = 0 Soluciones. a) y = 5, b) x = −2

270

4.7.

Cap´ıtulo 4.

Geometr´ıa anal´ıtica

La circunferencia

Se recuerdan los siguientes conceptos. Al conjunto de puntos en el plano que distan de un punto fijo en una magnitud constante se le denomina circunferencia. El punto fijo es el centro de la circunferencia y la magnitud constante se llama radio de la circunferencia. Al segmento de recta que une dos puntos distintos de una circunferencia se le llama cuerda. Las cuerdas que pasan por el centro se llaman di´ ametros. Teorema 4.7.1 La ecuaci´ on de la circunferencia con centro C(h, k) y radio r es:

(x − h)2 + (y − k)2 = r2 .

(4.7.4)

Demostraci´on. Sea P (x, y) un punto de la circunferencia. Por definici´on de circunferencia la distancia del punto P al centro C de la circunferencia es igual a r. Por lo tanto √ (x − h)2 + (y − k)2 = r y al elevar al cuadrado ambos lados de la igualdad se tiene (x − h)2 + (y − k)2 = r2 . As´ı, cualquier punto de la circunferencia cumple la u ´ltima igualdad. Rec´ıprocamente, si un punto cumple la ecuaci´on 4.7.4 entonces el punto est´a en la circunferencia.  La ecuaci´on 4.7.4 se llama la ecuaci´ on can´ onica de la circunferencia. Corolario 4.7.1 La ecuaci´ on de la circunferencia con centro en el origen y radio r es: x2 + y 2 = r 2 . Ejemplo

4.7.1 Hallar la ecuaci´on de la circunferencia con centro en el origen y radio 2. Soluci´on. La ecuaci´on es de la forma x2 + y 2 = r2 , como r = 2 se tiene x2 + y 2 = 4 . 

4.7.

La circunferencia

271

Ejemplo

4.7.2 Hallar la ecuaci´on de la circunferencia con centro en el origen y que pasa por el punto (3, 4). Soluci´ on. El radio es la distancia del centro (0, 0) al punto (3, 4). As´ı √ √ √ r = (3 − 0)2 + (4 − 0)2 = 9 + 16 = 25 = 5 . La ecuaci´on de la circunfencia es x2 + y 2 = 25 .



Ejemplo

4.7.3 Obtenga la ecuaci´on de la circunferencia con centro en (3, 5) y radio 6. Soluci´ on. La ecuaci´on es de la forma (x − h)2 + (y − k)2 = r2 . Aqu´ı h = 3, k = 5 y r = 6, por lo tanto, la ecuaci´on es (x − 3)2 + (y − 5)2 = 36 .  Ejemplo

4.7.4 Los extremos de un di´ametro de una circunferencia son los puntos (2, 3) y (−4, 5). Hallar la ecuaci´on de la circunferencia. Soluci´ on. Un di´ametro de la circunferencia es una cuerda de magnitud 2r. Por lo tanto el centro (h, k) de la circunferencia es el punto medio del di´ametro dado. 2−4 −2 8 3+5 h= = = −1 , = =4. k= 2 2 2 2 As´ı, el centro de la circunferencia tiene coordenadas (−1, 4). El radio es la distancia del centro a cualquiera de los puntos dados √ √ √ √ r = (2 + 1)2 + (3 − 4)2 = 32 + (−1)2 = 9 + 1 = 10 . Luego, la ecuaci´on de la circunferencia es (x + 1)2 + (y − 4)2 = 10 .

4.7.1.

Ecuaci´ on general de la circunferencia

Desarrollando la ecuaci´on can´onica de la circunferencia se tiene (x − h)2 + (y − k)2 = r2 x2 − 2hx + h2 + y 2 − 2ky + k 2 − r2 = 0 x2 + y 2 + (−2h)x + (−2k)y + h2 + k 2 − r2 = 0 .



272

Cap´ıtulo 4.

Geometr´ıa anal´ıtica

Si se denota D = −2h ,

E = −2k ,

F = h2 + k 2 − r2

la ecuaci´on se escribe como

x2 + y 2 + Dx + Ey + F = 0 . As´ı, toda circunferencia tiene una ecuaci´on de la forma anterior, la cual es llamada ecuaci´ on general de la circunferencia. Rec´ıprocamente, el problema que se presenta ahora es averiguar si toda ecuaci´on de la forma x2 + y 2 + Dx + Ey + F = 0, representa una circunferencia. Para contestar esta pregunta se pasa de la forma general a la forma can´onica de la circunferencia. x2 + y 2 + Dx + Ey + F = 0 2 2 x + Dx + y + Ey = − F . Completando cuadrados ( )2 ( )2 D E D2 E 2 2 2 x + Dx + + y + Ey + = −F + + 2 2 4 4 )2 ( )2 ( E −4F + D2 + E 2 D + y+ = x+ 2 2 4 Luego, si −4F + D2 + E 2 > 0 la ecuaci´on corresponde a una circunferencia con centro √ ( ) −D −E D2 + E 2 − 4F C , y radio r= . 2 2 2 En caso que −4F + D2 + E 2 = 0 la ecuaci´on corresponde a un punto, y no representa circunferencia alguna en R2 si D2 + E 2 − 4F < 0. Ejemplo

4.7.5 Para cada una de las siguientes ecuaciones determine si la ecuaci´on representa o no una circunferencia. Si la respuesta es afirmativa, hallar su centro y radio. 1. x2 + y 2 + 4x + 6y − 12 = 0

4.7.

La circunferencia

273

2. 4x2 + 4y 2 + 24x − 8y − 16 = 0 3. x2 + y 2 − 3x + 5y + 21 = 0 Soluci´ on. 1. −4F + D2 + E 2 = −4(−12) + 42 + 62 = 100 > 0, por tanto la ecuaci´on si representa una circunferencia. Su centro y radio son ( C

−D −E , 2 2

√ r=

)

( =

4 6 − ,− 2 2

−4F + D2 + E 2 = 2



) = (−2, −3)

10 100 = =5. 2 2

2. Se repite expl´ıcitamente el procedimiento algebraico 4x2 + 4y 2 + 24x − 8y − 16 = 0 dividiendo entre 4 x2 + y 2 + 6x − 2y − 4 = 0 x2 + 6x + y 2 − 2y = 4 x2 + 6x + 32 + y 2 − 2y + 12 = 4 + 9 + 1 (x + 3)2 + (y − 1)2 = 14 . Por lo tanto, √ es la ecuaci´on de una circunferencia con centro en (−3, 1) y radio r = 14. 3. −4F + D2 + E 2 = −4(21) + (−3)2 + 52 = −50 < 0, por tanto, la ecuaci´on dada no representa una circunferencia.  Ejemplo

4.7.6 Hallar la intersecci´on de la recta x + 3y − 8 = 0 y de la circunferencia x2 + y 2 − 4x − 4y − 2 = 0. Graficar la recta y la circunferencia. Soluci´ on. Si un punto (x0 , y0 ) est´a en la intersecci´on de la recta y la circunferencia, entonces sus coordenadas deber´an satisfacer ambas ecuaciones. Luego se debe resolver el sistema de ecuaciones: x + 3y − 8 = 0 x + y − 4x − 4y − 2 = 0 . 2

2

274

Cap´ıtulo 4.

Geometr´ıa anal´ıtica

Despejando x de la primera ecuaci´on, se tiene x = 8 − 3y. Sustituyendo esta expresi´on de x en la segunda ecuaci´on resulta (8 − 3y)2 + y 2 − 4(8 − 3y) − 4y − 2 = 0 64 − 48y + 9y 2 + y 2 − 32 + 12y − 4y − 2 = 0 10y 2 − 40y + 30 = 0 y 2 − 4y + 3 = 0 (y − 1)(y − 3) = 0 . De aqu´ı: y1 = 1, y2 = 3. Sustituyendo y1 , y2 en x = 8 − 3y se tiene x1 = 8 − 3y1 = 8 − 3(1) = 8 − 3 = 5 x2 = 8 − 3y2 = 8 − 3(3) = 8 − 9 = −1 . As´ı, la recta x + 3y − 8 = 0 y la circunferencia (x − 2)2 + (y − 2)2 = 10 se intersectan en los puntos (−1, 3) y (5, 1), seg´ un se ilustra en la figura 4.11.  5

x+3y-8=0 H-1,3L

4

x2 +y2 -4x-4y=-2

3 2

H2,2L H5,1L

1

2

-2

4

6

-1

Figura 4.11: La intersecci´on de la circunferencia x2 + y 2 − 4x − 4y − 2 = 0 y la recta x + 3y − 8 = 0. Ejemplo

4.7.7 Hallar la intersecci´on de las circunferencias:

x2 + y 2 − 6x − 6y − 7 = 0;

x2 + y 2 + 2x + 2y − 15 = 0 .

Soluci´on. Si un punto (x0 , y0 ) est´a en la intersecci´on de las circunferencias sus coordenadas deben satisfacer ambas ecuaciones, luego se tiene que resolver el sistema de ecuaciones: x2 + y 2 − 6x − 6y − 7 = 0 x2 + y 2 + 2x + 2y − 15 = 0 .

4.7.

La circunferencia

275

Se multiplica la primera ecuaci´on por −1 y al sumar con la segunda se tiene − x2 − y 2 + 6x + 6y + 7 = 0 x2 + y 2 + 2x + 2y − 15 = 0 8x + 8y − 8 = 0 Se despeja y de la ecuaci´on resultante 8y = 8 − 8x y =1−x y esta expresi´on de y se sustituye en la primera ecuaci´on x2 + (1 − x)2 − 6x − 6(1 − x) − 7 = 0 x2 + 1 − 2x + x2 − 6x − 6 + 6x − 7 = 0 2x2 − 2x − 12 = 0 x2 − x − 6 = 0 (x − 3)(x + 2) = 0 . Por lo tanto las soluciones son x1 = 3, x2 = −2 y se sustituyen en la ecuaci´on y = 1 − x para obtener y1 = 1 − x1 = 1 − 3 = −2, y2 = 1 − x2 = 1 − (−2) = 1+2 = 3. Por lo tanto, las circunferencias se intersectan en los puntos (3, −2) y (−2, 3), como se ilustra en la figura 4.12.  Ejercicio 4.7.1 1. Para cada una de las siguientes ecuaciones determinar si la ecuaci´on representa o no a una circunferencia. Si la respuesta es afirmativa hallar el centro y el radio de la circunferencia. a) x2 + y 2 + 6x − 8y + 200 = 0 b) (x − 3)2 + (y + 1)2 = −4 c) x2 + y 2 − 8x + 15 = 0 2. Determinar la ecuaci´on de la circunferencia que pasa por los puntos (0, 0), (3, 3) y (4, 6). Dar el centro y el radio de la circunferencia. 3. Hallar la intersecci´on de la circunferencia x2 + y 2 + 4x − 6y + 8 = 0 con la recta x + 2y − 9 = 0. Graficar ambas figuras.

276

Cap´ıtulo 4.

Geometr´ıa anal´ıtica

8 6

H-2,3L

x2 +y2 -6x-6y-7=0

4

H3,3L 2

-4

2

-2

4

6

8

H-1,-1L -2

H3,-2L

-4 2

2

x +y +2x+2y-15=0

-6

Figura 4.12: Las circunferencias x2 +y 2 −6x−6y−7 = 0, x2 +y 2 +2x+2y−15 = 0. 4. Encontrar la intersecci´on de la recta 4x − 3y + 12 = 0 con la circunferencia x2 + y 2 − 10x + 4y + 21 = 0. Graficar ambas figuras. 5. Una cuerda de la circunferencia x2 + y 2 − 24x − 2y + 80 = 0 est´a sobre la recta con ecuaci´on 3x − 2y − 8 = 0. Hallar los extremos de la cuerda y su longitud. 6. Hallar los puntos de intersecci´on de las circunferencias x2 + y 2 + 8x − 6y − 6 = 0 y x2 + y 2 + 4x − 2y + 2 = 0 7. Encontrar los puntos de intersecci´on de las circunferencias x2 + y 2 − 6x + 5 = 0 y x2 + y 2 − 6x − 4y + 5 = 0. Graficar ambas curvas. 8. Una circunferencia pasa por los puntos (−5, 2) y (3, 6) y su centro est´a en la recta y = x + 2. Obtener la ecuaci´on de la circunferencia. 9. Una circunferencia tiene radio 5, centro en el eje x, y pasa por el punto (2, 4). Dar la ecuaci´on de la circunferencia. 10. Un punto P (x, y) se mueve en el plano de manera tal que el segmento P A siempre es perpendicular al segmento P B, con A(−2, 4) y B(2, −4). Determinar el lugar geom´etrico que describe el punto P (x, y). Sugerencia. Sean (x, y) las coordenadas del punto P , entonces la pendiente del segmento P A es...

4.7.

La circunferencia

277

Soluciones. 1. a) No

b) No

c) Si, C(4, 0), r = 1.

2. (x + 4)2 + (y − 7)2 = 65 ,

C(−4, 7), r =



65.

3. (−1, 5). 4. No se intersectan. 5. (4, 2) y (8, 8). Longitud de la cuerda



52.

6. No se intersectan. 7. (1, 0) y (5, 0). 8. x2 + (y − 2)2 = 52 . 9. (x + 1)2 + y 2 = 25 ´o (x − 5)2 + y 2 = 25. 10. x2 +y 2 =√20. El punto describe una circunferencia de centro en el origen y radio 20.

278

Cap´ıtulo 4.

4.8.

Geometr´ıa anal´ıtica

La par´ abola vertical

La gr´afica de la ecuaci´on cuadr´atica

y = ax2 + bx + c

con

a ̸= 0

se denomina par´ abola vertical . Para obtener algunas propiedades de la par´abola vertical se escribe la ecuaci´on y = ax2 + bx + c usando la relaci´on 2.5.4

[(

2

ax + bx + c = a

b x+ 2a

)2

b2 − 4ac − 4a2

] .

As´ı se tiene (

b y =a x+ 2a

)2 +

4ac − b2 . 4a

De esta expresi´on y de la figura 4.13 se obtienen las siguientes conclusiones. umero positivo arbitrario y sean x1 = − Sea t > 0 un n´

b − t, 2a

b b x2 = − + t dos puntos sim´etricos con respecto al punto − . El 2a 2a valor de la ordenada y de la par´abola en estos dos puntos es [( ] )2 b b b2 − 4ac b2 2 y =a − ±t+ − = at + c − . 2a 2a 4a2 4a Esto muestra que los valores de la variable y coinciden en puntos b b sim´etricos al punto − , en otras palabras, la recta vertical x = − 2a 2a es el eje de simetr´ıa vertical de la par´abola y = ax2 + bx + c. Sea a > 0. Si la variable t > 0 crece, entonces tambi´en la variable y crece, es decir la par´abola y = ax2 + bx + c abre hacia arriba y adem´as b2 b si t = 0, o sea x = − , se obtiene el valor m´ınimo de y = c − . 2a 4a

4.8.

La par´ abola vertical

279

Sea a < 0. Si la variable t > 0 crece, entonces la variable y decrece, es decir la par´abola y = ax2 + bx + c abre hacia abajo y si t = 0, o sea b b2 x = − , se obtiene el valor m´aximo de y = c − . 2a 4a ertice de la par´ abola y = ax2 + bx + c como el punto Se define el v´ ( ) b 4ac − b2 V − , . 2a 4a

Eje de simetría

V

b -

b -t

2a

x=-

b -

2a

+t 2a

Figura 4.13: La par´abola vertical y = ax2 + bx + c, con a > 0. Del Teorema 2.9.1 y de la gr´afica de una par´abola, se obtiene una interpretaci´on de las soluciones de la ecuaci´on cuadr´atica ax2 + bx + c = 0 . Teorema 4.8.1 La gr´afica de la par´ abola y = ax2 + bx + c intersecta al eje horizontal x en dos puntos distintos si b2 − 4ac > 0, es tangente al eje x si b2 − 4ac = 0 y no intersecta al eje x si b2 − 4ac < 0. A continuaci´on, se ilustra este resultado.

280

Cap´ıtulo 4.

Geometr´ıa anal´ıtica

Si b2 − 4ac > 0 las gr´aficas de la par´abola son de la forma

a>0 x

x

a0 x

x

a0 x

x

a
View more...

Comments

Copyright ©2017 KUPDF Inc.
SUPPORT KUPDF